Ihr wollt mir eine Frage stellen, möchtet dies öffentlich tun, damit andere Leute, die eine ähnliche Frage haben, dies später einfach hier nachlesen können und habt keinen Artikel gefunden, zu dem die Frage passen würde?

Dann benutzt einfach die Kommentar-Funktion unter diesem Artikel.

Anonyme Kommentare sind natürlich auch immer willkommen.

Falls ihr euch unsicher seid, ob eine Frage schon mal gestellt worden ist: Im Zweifelsfall einfach hier drunter schreiben.

Von mir aus auch persönliche Fragen (wie heißen deine Katzen?, welche Kleidergröße hast du? o.ä.). Wenn sie mir zu persönlich werden, dann sage ich euch das schon *g*.

Kommentare (348)

  1. #1 UMa
    15. September 2017

    Hallo Tobias, danke für den Fragenartikel.

    Ich möchte gleich eine erste Frage stellen.

    Kalium enthält ja das radioaktive Isotop K-40. Wenn man jetzt auf die (wahrscheinlich dumme) Idee käme auf Kalidüngersäcken (mehrere Zentner) zu schlafen, wie hoch wäre die Strahlenbelastung pro Stunde in etwa?
    Und wie hoch wäre er bei reinem K2O?

  2. #2 Tobias Cronert
    15. September 2017

    Ja, grundsätzlich hat Kaliumdünger schon einiges an Aktivität:

    16kg Kalium mit 20Bequerel pro g wären dann 320000 Bequerel, also 320tausend radioaktive Zerfälle pro Sekunde. Soweit so richtig. Wenn jetzt jeder Zerfall auf deinen Körper wirken würde und diese WIrkung für zwei Monate anhält, dann würdest du tatsächlich deine 4mSv an Dosis abbekommen.

    Jetzt kommt natürlich die Realität mit ins Spiel. Wenn diese 320kBq eine Punktquelle wären, dann würde die Strahlung mit dem Abstandsquadrat abnehmen. Das heißt, wenn du in 1cm Abstand 4mSv abbekommen würdest, dann würdest du in 2m Abstand nur noch 1/10000, also 0,4µSv, abbekommen.

    Dann gibt es noch die Selbstabschirmung des Materials. Aus der Hüfte geschossen sollten 2cm KOH reichen um das meiste der Strahlung abzuschirmen. Das heißt, dass du nur den Teil der 16kg abbekommst, der dir zugewandt ist. Das heißt von den eigentlichen 16kg “sehen” dich nur die Strahlen von … sagen wir mal 4kg des Stoffes. Dann bist du schon mal runter auf 0,1µSv pro zwei Monate. Also in zwei Meter bist du absolut sicher, egal, wie lange du dich aufhälst. Aber wenn du dich nackt in eine Badewanne voll mit KOH legst, von Kopf bis Fuß bedeckst und so zwei Monate liegen bleibst, dann bekommst du schon eine gehörige Menge an ionisierender Strahlung bzw. Dosis (mSv) ab.

    Die Frage hatten wir schon einmal in ähnlicher Ausführung (https://scienceblogs.de/nucular/2015/07/06/bananendosenaequivalent/#comment-9948) deswegen konnte ich jetzt hier ein wenig copy and pasten.

    Für die Zukunft: Ihr braucht nicht alle anderen Beiträge nach ähnlichen Fragen zu durchsuchen, denn das kann recht aufwendig werden. Ich habe da im Admin Backend bessere Möglichkeiten zu. Also auch auf die Gefahr hin, das wir uns widerholen… Immer hier fragen, dann antworte ich schon und poste ggf. einen Link.

  3. #3 UMa
    15. September 2017

    Danke. Diese Frage wollte ich schon immer mal stellen. Die ähnliche Frage hatte ich nicht gesehen.
    Ich hatte es versucht selbst auszurechnen, ohne Selbstabschirmung, weil ich davon keine Ahnung habe, und war auf ziemlich hohe Werte gekommen, ein vielfaches der natürlichen Strahlung.

  4. #4 Tobias Cronert
    15. September 2017

    Selbstabschirmung ist notorisch schwer auszurechnen. In der Anwendung schätzt man dann eher aus der Hüfte und macht eine “Worst-Case”-Abschätzung oder macht eine Messung.

    Wenn man es richtig haben möchte, dann müsste man eine Monte-Carlo Simulation machen, aber das ist dann schon recht aufwendig und bei den meisten Anwendungen viel zu teuer.

  5. #5 UMa
    15. September 2017

    Gut dann eine weitere Frage. Zur kosmischen Strahlung.

    Welche Komponenten verursachen die höchste Strahlendosis auf der Erde? Sekundärstrahlung wie Myonen und Neutronen?

    In den letzten 20 Jahren hat sich durch Funde von Fe-60 in den Sedimenten herausgestellt, dass in den letzten Jahrmillionen mehrere Supernovae in ca. 100 pc Entfernung explodiert sind, welche die lokale Blase erzeugten.
    Die Supernovae erzeugen an ihrer Stoßfront kosmische Strahlung in erheblichen Mengen.
    Die Erhöhung der kosmischen Strahlung hängt von der m.W. unbekannten Richtung des interstellaren Magnetfeldes ab. Nach den unten verlinkten Papern von einer geringer Erhöhung im günstigsten Fall zu mittleren Werten von 20 bis 150-fachen der heutigen Werte bis zum 10000-fachen im Worstcase. Und das über viele tausend Jahre.
    Was wären die Auswirkungen einer solchen Erhöhung der kosmischen Strahlung?
    https://arxiv.org/abs/1605.04926
    https://arxiv.org/abs/1702.04365

  6. #6 Tobias Cronert
    15. September 2017

    Uuufts, das ist schwer. Mit kosmischer Strahlung habe ich so gut wie gar nichts zu tun und daher auch keine gute Antwort parat. Ich gucke mal nach, ob mir da zufällig ein paar Infos auf die Füße fallen… aber keine Garantie.

  7. #7 werner
    15. September 2017

    @#2: “Aber wenn du dich nackt in eine Badewanne voll mit KOH legst, von Kopf bis Fuß bedeckst und so zwei Monate liegen bleibst,…” dann hast Du absolut kein Problem mehr mit der Strahlung (Stichwort “Verseifung”). Dann musst Du nur noch den Stöpsel ziehen, alles runterspülen und die Knochen separat versorgen. 😉

  8. #8 Tobias Cronert
    15. September 2017

    Tja, ich sag ja immer, gegen so manche biologischen oder chemischen Sachen ist Radioktivität eher schwach auf der Brust. *g*

  9. #9 tomtoo
    17. September 2017

    @ Tobias
    Was machen eigentlich ultrakalte Neutronen die zusammenkommen. Die reagieren über die starke WW oder ? Da könnte ich mir ja sozusagen eine ‘Erbse’ aus neutronen bauen ? Ok die teuerste Erbse der Welt.

  10. #10 Robert
    17. September 2017

    Tobias,
    Hat irgendein Mensch schon einmal ein Neutron gesehen?
    Ich meine jetzt nicht seine Spur sondern seine Umrisse?
    Ein Proton hat ja auch noch niemand gesehen.
    Ich denke da auch nicht an die Elektronenhülle!

    Wie stellst du dir die Kernbausteine vor? Als kleine Kügelchen? Erzähl mal aus dem Nähkästchen!

  11. #11 Tobias Cronert
    17. September 2017

    Wenn man ultrakalte Neutronen in einem Becher sammelt, dann “klumpen” sie theoretisch über die Graviation, starke und schwache WW zusammen. Aber alle Diese Kräfte sind sehr schwach oder kurzreichweitig, weswegen eher die minimale, aber immer noch vorhandene Kinetische Enegie dominiert.

    Aber das ist schwer zu beobachten, weil derzeit schafft man es ca. 10^3-5 Neutronen zu fangen. Das ist halt immer noch seeeeehr weit von einem Nanogramm entfert, mit dem man irgendwas makroskopisch anstellen könnte.

    Ja, theoretisch ist es ein Neutronenstern im Taschenformat…. aber

  12. #12 tomtoo
    17. September 2017

    @Tobis
    Das wäre richtig cool. Der Neutronenstern im Reagenzglass. Klasse Buchtitel. ; )

  13. #13 Tobias Cronert
    18. September 2017

    @Robert: Uuups habe deinen Beitrag zu spät gesehen. “Sehen” ist da ein sehr relativer Begriff. Ich würde ich meinem Sprachgebrauch schon sagen, dass ich Neutronen gesehen habe. Aber wenn du “sehen” jetzt als “Reflektion von sichtbarem Licht, dass dann ins Auge kommt” definierst …. dann ist es natürlich unmöglich ein Neutron zu sehen. Das Verhältnis von Elektronenhülle zu Kern ist in etwas das eines Menschen zur gesamten Erde. Selbst wenn alle Menschen der Erde auf einem Fleck wären könnte man sie warscheinlich aus dem Weltraum aus nicht sehen. Ähnlich ist es mit den Neutronen.

    Einen Apfel sehe ich ja, indem EM-Wellen mit einer Wellenlänge von ca 500nm reflektiert werden und von Detektoren in unserer Netzhaut in elektrische Signale verwandelt werden, die unser Gehirn interpretiert. Ein Neutronendetektor macht das gleiche, nur dass da noch ein Zwischenschritt vorgeschaltet ist. Also Signal -> visuelle Darstellung -> Auge -> Gehirn.

    Ich haben einen Aufbau, an dem ich 1 Neutron pro Minute messe. Das würde ich durchaus als “sehen” bezeichnen. Lass noch mal ein par Jahre ins Land gehen und dann kann ich bestimmt auch meinen Detektor direkt über eine neuronale Schnittstelle an mein Nervensystem anschließen … dann kann man Neutronen wirklich sehen.

    Als kleine Kügelchen stelle ich sie mir eigentlich nicht vor, sondern eher als einzelne Wasserwelle. OK, da ist kein “Wasser” Medium, aber da kann ich mir trotzdem gut verschiedene Wellenformen (spitz, lang breit) vorstellen und auch einen Aotmkern aus verscheidenen Wellen, die übereinander liegen.

  14. #14 gedankenknick
    18. September 2017

    Nur mal so allgemein in den Raum geworfen: Der Elektronikversender ELV hat jetzt auch einen eigenen Radioaktivitätsdetektor. https://www.elv.de/gamma-scout-strahlungsmessgeraet-geigerzaehler.html Für fast 400€ finde ich das allerdings schon recht heftig, zumal ich die Beschreibung so interpretiere, dass das gute Stück sowieso nur so Langzeitüberwachung und -auswertung gedacht ist – also explizit NICHT für eine “Kurzzeit-Detektion”. Allerdings soll das Teil kalibriert sein in einem Messbereich 0,01 bis 1.000 µSv/h.

    Aber bei DEM Preis frage ich mich tatsächlich nach einer sinnvollen Verwendung für den geneigten “Endverbraucher”, wenn man mal von Wohnbereichen gegenüber einem KKW oder einem (Braun)Kohlekraftwerk absieht…

  15. #15 Tobias Cronert
    18. September 2017

    Ja diese ganzen mittelpreisigen Strahlungsmesser waren einer der Hauptgründe, warum ich hier meine kleine Vergleichsserie von Detektoren angefangen habe. Die gehen mir nämlich ziemlich auf den Keks.

    Die meisten mittelpreisigen Geräte bieten gegenüber den billigen (60-180€) keinen Vorteil und sind offensichtlich dazu gedacht Menschen mit Angst vor Radioaktivität Geld aus der Tasche zu ziehen.
    Es gibt ein paar Geräte, bei denen man für die 200€ mehr auch mehr Leistung bekommt… aber die sind eher die Ausname (und weil ich nicht so viel Geld habe kann ich da auch nicht viel testen). Die, die sich lohnen haben aber schon andere Leute rezensiert.

    Daher mein TIp: Kauft die günstigsten oder investiert Zeit (und Geld) um euch etwas richtig gutes rauszusuchen.

  16. #16 Robert
    18. September 2017

    Tobias ’13,
    Danke für die ausführliche Antwort,
    wenn man allerdings den Gedanken von tomtoo noch hinzunimmt und einen Becher mit Neutronen füllt, dann könnte man die Oberfläche dieser “Energiewellen” irgendwie nachweisen.

  17. #17 Tobias Cronert
    19. September 2017

    Um sie makroskopisch nachzuweisen muss die Anzahl der Neutronen halt in der Nähe der Avogadro Konstante sein… also irgendwo bei 10^23

    Soweit ich weis kann man derzeit maximal 10^3-5 Neutronen gleichzeitig “fangen”.

  18. #18 tomtoo
    19. September 2017

    @Tobias
    Gibts bei WP keine möglichkeit das “Fragen an den Autor” oben festzupinnen ? Wäre ja sinvoll imho.

  19. #19 Tobias Cronert
    19. September 2017

    Die Möglichkeit gibt es und ich habe es gerade mal eingerichtet. Danke für die Idee.

    Aber ich bin mir noch nicht sicher, ob ich das auch permanent so lassen möchte. Ich muss, sowieso mal mehr Zeit in das Design hier investieren… ergo kommt auf die 2do Liste.

  20. #20 tomtoo
    19. September 2017

    @Tobias
    Ey ,super !

    Also ich find’s gut.

  21. #21 rolak
    19. September 2017

    bin mir noch nicht sicher

    Ein auf ewig als ‘neuester Artikel’ deklarierter Text ist ja auch ziemlich gewöhnungsbedürftig, Tobias. Doch es gibt zusätzlich die deutlich dezentere, das schnell&leicht-Finden aber kaum einschränkende Variante des Unterbringens im rechten Rand. So wie zB bei MartinB der link zu den Artikelserien unter ‘über das Blog’.

    • #22 Tobias Cronert
      19. September 2017

      Einen Link auf der rechten Seite habe ich schon gemacht. Ich werde ihn erst mal oben lassen und ein wenig darüber meditieren und ausprobieren.

  22. #23 tomtoo
    19. September 2017

    Mist, müsste PC anschmeisen. Aber auf dem Handy sehe ich gar keine Links rechts ?

  23. #24 rolak
    19. September 2017

    Sachma, Tobias, stand das mit ‘Fragenartikel’ da eben auch schon? Gruselige Vorstellung, mir ist so, als hätte ich eben mal ergebnislos nachgeguckt…

    keine Links rechts ?

    Evtl zwischen Artikel und Kommentaren, tomtoo, oder ganz ‘unten’ dran hängend. Diese Beistelltischchen oder sidebars finden sich manchmal an den urkomischsten Stellen wieder.

  24. #25 tomtoo
    19. September 2017

    Also auf dem Handy , find ichs praktisch.

    @Rolak Beistelltischchen ?? Ooo….kee… %-)

  25. #26 demolog
    20. September 2017

    Atommüllentsorgung:

    Warum, meinen sie, das, wenn alle doch so behutsam mit dem Problem umgehen wollen, niemand auf die Idee kommt, die wahrhaftig ultimative Endlagerung im Verbohren in Subduktionszonen zu erkennen?

    Also den Müll in in Subduktionszonen tief verbohren, damit er dann auch absehbar tatsächlich “weg” ist und nie wieder an die Oberfläche kommen kann!

    Bei allen anderen Endlager-Strategien scheinen (nicht nur mir) die Sicherheitsaspekte eher unvollständig erfüllbar zu sein.

  26. #27 roel
    no gods, no kings, no courts
    24. Oktober 2017

    @Tobias Cronert “Fragen an den Autor” gerade erst entdeckt.

    Ich denke es gibt Studien zur Auswirkung der Reaktorkatastrophe in Tschernobyl. Welche Auswirkungen hat die freigesetzte Radioaktivität auf die Tiere und Pflanzen rund um Tschernobyl? Vielleicht hast du einen guten Linktip oder noch besser vielleicht wäre das ein Thema für einen eigenen Beitrag.

  27. #28 Tobias Cronert
    24. Oktober 2017

    Also es gibt sehr viele Tschernobyl Experten da draußen im Internet, die sich mit dem Thema wesentlich mehr beschäftigt haben, als ich. Ich denke nicht, das ich mich so bald mit dem Thema beschäftigen werde, weil ich da erst mal viel Arbeit reinstecken müsste um auf einem durchschnittlichen “Internet” Niveau zu landen.

    Aber ein nettes Video bzgl. Tiere und Pflanzenwelt, ist mMn das hier: https://www.youtube.com/watch?v=-WhIDtP64aw

  28. #29 Tobias Cronert
    24. Oktober 2017

    @Demolog: Bislang schließt man alle geologisch irgendwie aktiven Zonen aus, weil man nich nicht sicher sein, kann, dass nichts negatives passiert. Bei einer Bohrung in einer Subduktionszohne ist es ja auch möglich, dass das Material dann über geologische Aktivitäten nach oben kommt. auch wenn die Wahrscheinlichkeit gering ist, so ist sie doch hoch genug, dass man es nicht riskieren möchte.

  29. #30 roel
    no gods, no kings, no courts
    24. Oktober 2017

    @Tobias Cronert Danke für den Link

  30. #31 Beobachter
    21. November 2017

    @ Tobias Cronert:

    Was ist davon zu halten?:

    https://www.sueddeutsche.de/panorama/ruthenium-erhoehte-radioaktivitaet-ueber-europa-russland-als-ursache-bestaetigt-1.3758770

    Zitate:

    “21. November 2017, 12:05 Uhr
    Ruthenium
    Erhöhte Radioaktivität über Europa kommt aus Russland

    Das deutsche Bundesamt für Strahlenschutz hatte erklärt, erhöhte Ruthenium-Werte seien seit Ende September aus Deutschland, Italien, Österreich, der Schweiz und Frankreich gemeldet worden.
    Die französische Atomaufsichtsbehörde vermutete einen Unfall in einer russischen Einrichtung.
    Russland dementierte die Berichte zunächst, bestätigte nun aber, dass in Teilen des Landes eine “äußerst hohe” Konzentration des Isotops Ruthenium-106 festgestellt wurde. … ”

    ” … Nach ersten Berichten über ein Ruthenium-Leck in einer Anlage im Südural hatte der staatliche russische Rosatom-Konzern im Oktober erklärt, die Substanz komme nicht aus seinen Anlagen. “Die Behauptung, dass die Kontamination russischen Ursprungs ist, ist unbegründet”, erklärte Rosatom.

    Nun hat der russische Wetterdienst bestätigt, dass in Teilen des Landes eine “äußerst hohe” Konzentration von radioaktivem Ruthenium-106 festgestellt wurde, berichtet die Nachrichtenagentur AFP. Die höchste Konzentration wurde demzufolge in der Messstation Argajasch registriert, einem Dorf in der Region Tscheljabinsk im südlichen Ural an der Grenze zu Kasachstan, 20 Kilometer von der Kerntechnischen Anlage Majak entfernt, wo sich 1957 der drittschwerste Atomunfall der Geschichte ereignet hatte. Dort sei in der Woche vom 25. September bis 7. Oktober eine Konzentration von Ruthenium-106 gemessen worden, die das 986-Fache des erlaubten Werts betragen habe. Heute dient die Anlage der Wiederaufbereitung abgebrannter nuklearer Brennstoffe…. “

  31. #32 Tobias Cronert
    21. November 2017

    Tjoa, also wenn das BfS und die anderen nationalen Behörden das so nachgemessen haben, dann wird das wohl so stimmen. Ru106 mit der charakteristischen weichen Betalinie und der harten + Gamma im Rhodium ist zwar nicht ganz einfach zu detektieren, aber recht eindeutig.

    Wenn man jetzt von Windrichtung etc. auf eine Ausbreitung aus Russland schließen kann ist das schon hübsch, aber da habe ich selber keine Expertise und diesen Punkt kann ich nicht nachvollziehen bzw. bestätigen.

    Ein Kumpel von mir arbeitet mit Ruthenium (also dem stabilen Isotop) und das ist halt ein rel. normales Metall. Das als Aerosol in die Luft zu bekommen ist schon eher schwierig. Da muss schon irgendein (hoher) thermischer Prozess hinter stecken.

    Ich habe jetzt auch keine bessere Idee, als das BfS, was da in Russland passiert sein könnte. Wie ja gesagt wurde ist es schon verwunderlich, dass da keine anderen Isotope unterwegs sind, was für eine sehr selektive Freisetzung spricht.

    Isotopenbatterien? Sattelit? Joa, hört sich plauisibel an. Alienraumschiff? Spricht auch nichts dagegen 😉

    Was ich schön finde, ist, dass alle Messtationen mittlerweile so fein sind, dass in der Welt quasi nichts mehr passieren kann, ohne dass es viele Leute mitbekommen … zumindest im Nukularen Bereich. Ein ernstzunehmender Unfall würde sofort überall die Alarmglocken schellen lassen und “das 1000fache des erlaubten Wertes” qualifiziert noch nicht mal im Ansatz als großer Unfall.

    Die Größe des Unfalls liegt irgendwo in der Ecke “Mitarbeiter aus ehem. Isotopenbatterieproduktion hat den Filter der Anlage auf das Lagerfeuer im Garten geworfen um es zu entsorgen”. Oder eben das Alienraumschiff oder die geheime Sovjetwaffe, die nun entsorgt wird. Ein bischen die Fantasie spielen lassen darf man ja schon *g*.

  32. #33 Beobachter
    21. November 2017

    Die Leute, die dort die fast 1000-fache Dosis des erlaubten Wertes abbekommen haben, finden das bestimmt nicht witzig und machen auch bestimmt keine Jokes über “Alienraumschiffe”, “Lagerfeuerentsorgung” o. ä. …
    Würden Sie auch so fantasievoll witzeln, wenn diese extrem hohen Werte hier in D bzw. bei Ihnen in der Nähe gemessen worden wären?

    Außerdem finde ich es immer wieder etwas befremdlich, dass bei Unfällen mit austretender radioaktiven Strahlung zuerst immer “dementiert” wird.
    Vielleicht würde es öfters beim “Dementieren” bleiben, wenn es keine oder weniger (feine) Messstationen gäbe.

  33. #34 Tobias Cronert
    21. November 2017

    Also ich würde tatsächlich auch so witzeln, wenn ich persönlich diese Dosis abbekommen würde.

    Die Freigrenze für Ru106 liegt bei 10Bq/cm^2. Das tausendfache wären dann ca. 10^4 Bq/cm^2, was dann sehr (sehr) grob 10mSv/cm entspricht. Das der Boden aber so eine Dosis hat, heißt noch nicht, dass auch ein Mensch diese Menge abbekommen würde.
    Gaaaaanz grob aus dem Bauch heraus würde ich schätzen, dass ein Mensch ein paar mSv mehr im Jahr durch die 1000fache Menge Ru106 abbekommen würde. Die Gesundheitsgefährdung würde ich ähnlich stark einschätzen, wie bei einem kleinen bis mittelmäßigen Chemieunfall mit dem wichtigen Unterschied, dass wir diese radioaktive Gesundheitsgefährdung aus mehreren tausend Kilometer Entfernung messen können im Gegensatz zu einer chemischen Gefährdung.

    Die “Lagerfeuerentsorgung” war von mir nicht als Witz gedacht, sondern bitterer Ernst. Die meisten Kontaminationsunfälle entstehen dadurch, dass belasteter Müll oder Schrott unsachgemäß entsorgt wird. Es gibt viele Fälle von Kobald auf der Mülldeponie oder eben Luftfiltern, die einfach weggeschmissen, woanders eingebaut oder eben verbrannt werden.

    Ich entschuldige mich für die Alienraumschiffe. Das war als Platzhalter für “kuriose Gründe der Kontamination” gedacht, wobei der Grund für den Ru106 Austritt natürlich schon recht interessant ist, trotz der Gefährdung.

    Bzgl. letzterer sei hier noch gesagt, dass die Grenzen in Deutschland sehr sehr konservativ verfasst sind, wie man an meinem kleinen Rechenbeispiel oben sieht. Tausendfach über dem Grenzwert heißt nicht viel, wenn der Grenzwert bei einer Banane (ca. 12Bq) liegt. Es heißt eben “nur” tausend Bananen pro Quadratzentimeter. JA, das ist viel, aber immer noch überschaubar.

    Bei Strahlung wird genauso dementiert, wie bei Asbest und anderen Giftstoffen auch. Nur dass man den Leute ihre Lügen bei Radioaktivität viel leichter nachweisen und sie damit konfrontieren kann.

  34. #35 Tobias Cronert
    22. November 2017

    PS: Ein Kolege von mir wurde auch zu dem Thema von RP-online befragt https://www.rp-online.de/leben/gesundheit/news/wahrscheinlich-ist-ein-unfall-in-der-atomanlage-passiert-aid-1.7217396

    Kommt zu einem ähnlichen Ergebnis, wie ich, macht aber noch weniger Spekulationen.

  35. #36 UMa
    4. Dezember 2017

    Sind neben dem Ru106 noch andere radioaktive Isotope in erhöhten Konzentrationen festgestellt worden? Falls ja, welche?

  36. #37 Tobias Cronert
    4. Dezember 2017

    Also so weit ich weiß sind keine weiteren Isotope frei geworden. Aber das ist mit der großen Einschränkung “soweit ich weiß”. Denn obwohl das Thema sehr interessant ist habe ich mich selber recht wenig drum kümmern können, da ich zur Zeit recht wenig von selbiger habe.

    Falls bei der ersten Messung von Ru106 auch noch andere Isotope dabei gewesen wären, dann hätte man es sicher detektiert und wahrscheinlich auch veröffentlicht. Wenn es im Nachinein noch mal eine Kontamination mit anderen Substanzen gegeben hätte, dann hätte man sie vielleicht auch detektiert, aber es wäre nicht mehr davon berichtet worden. Letzteres kann man nicht wirklich ausschließen.

  37. #38 Heart of the Atom
    6. Dezember 2017

    Im einem Buch wird die Formel T = (tau * S) / (f(E) * P)
    für die Halbwertszeit von radioaktiven Kernen angegeben.
    Somit hat die Zeitdilatation der speziellen Relativitätstheorie keinen Einfluss auf die Kernzerfalls-Zeit.

    Zeigt eine Kernzerfall-Uhr auf einem nuklear betriebenen Nazi-Raumschiff, welches Passagiere mit halber Lichtgeschwindigkeit zu Urlaubs-Planeten bringt, eine andere Zeit als Einsteins mechanische Taschenuhr an?

  38. #39 Tobias Cronert
    17. Dezember 2017

    Hab die Frage gerade erst gesehene sorry. Ist aber ein sehr toller Gedanke, den ich aus dem Stehgreif auch nicht beantworten kann. Ich muss da mal drüber nachmeditieren und melde mich, wenn ich eine Antwort habe.

  39. #40 Detlef
    Erde
    29. Dezember 2017

    Hallo Tobias
    Keine Ahnung ob das in Deinen Blog passt, aber ich versuche es mal. Es stellt sich mir die Frage nach dem Verhalten eine ‘geteilten’ Elektron in einer Verbindung. So ‘teilen’ sich ja ein Sauerstoff und ein Wasserstoffatom ein Elektron, wenn sich beide verbinden. Was sagt das ‘teilen’ nun über das Verhalten / Wahrscheinlichkeitsort des geteilten Elektrons aus ?
    Bleibt das Elektron trotzdem bei ‘seinem’ Atom, oder wechselt es auch zum verbundenen Atom und damit auf eine höhere Schale ?
    Detlef

  40. #41 Tobias Cronert
    30. Dezember 2017

    Hallo Detlef,

    ganz klassische Fragestellung in der Festkörperphysik, also meinem Heimatgebiet *g*.
    Also grundsätzlich ändert sich durch Bindungen die Aufenthaltswarscheinlichkeiten und die Elektronen sind “woanders”, als sie es bei einem nicht gebundenen Atom wären. Wo sie genau sind und was sie genau machen ist EXTREM davon abhängig um was für ein System es sich handelt. Von einfachen kovalenten Bindungen, bis hin zu frustrierten Spinsystemen, in denen die Elektronen sog. Spin-Dichte Wellen bilden, mit denen man Q-Bits herstellen kann ist da eigentlich alles dabei

    Dabei müssen die Elektronen nicht unbedingt bei “Ihrem” Atom bleiben. Denk da am ehesten an die delokalisierten Bindungselektronen beim Benzolring.

    Eine Standartübung, die jeder Physiker im Studium in der Quantenmeschanik machen muss ist das rel. simple Wasserstoffatom. https://hydrogen.physik.uni-wuppertal.de/hyperphysics/hyperphysics/hbase/molecule/hmol.html

    von da an wird es mitunter wesentlich komplizierter, macht dann aber auch echt coole Effekte, wie z.B. Supraleitung im delokalisierten “Elektronengas” in einem Festkörper.

    Das “wechseln in eine höhere Schale” ist grundsätzlich möglich, aber gerade Energieniveaus sind da auch noch mal stark von den entsprechenden Substanzen abhängig. Aber grundsätzlich kommt da ja die chemische Bindungsenergie her, dass Elektronen im gebundenen Zustand “woanders” sind, als im “alleinstehenden” Atom.

    Gruß
    Tobi

  41. #42 Detlef
    30. Dezember 2017

    Hallo Tobias
    Vielen Dank für die ausführliche Erklärung !
    Gruß
    Detlef

  42. #43 tomtoo
    27. Februar 2018

    @Tobias
    Wenn du mal Zeit und natürlich auch Lust hättest, könntest du etwas über den Spinn erzählen. Das ist vollkommen verwirrend. Mal wir etwas über Drehimpuls, mal als pure Eigenschaft erzählt. Also wie kommt der Spinn eines elektrisch neutralen Teilchen zustande ? Warum gibt es zwei Neutronen die sich im Magnetfeld(ist doch Spinn oder?) dennoch unterschiedlich verhalten ?

  43. #44 tomtoo
    27. Februar 2018

    @Heart of a Atom
    Klar hat die SRT Einfluss auf die Kernzerfallszeit. Tau ist ja abhängig von der Zeit und somit vom Betrachter. Anders ausgedrückt 1Gramm Radium das sich relativ zu dir Bewegt zerfällt langsamer als ein Gramm Radium das neben dir steht.(Relativ zu dir Ruht).Oder habe ich da etwas falsch verstanden ?

  44. #45 Tobias Cronert
    28. Februar 2018

    Also über den Spin kann man sicher seeehr viel schreiben und vor allem Richtung normalen QM haben das eine Menetge Leute schon wesentlich besser gemacht, als ich das in absehbarer Zeit selber machen könnte. Spin bei Neutronen und Polarisation steht bei mir aber hoch oben auf der 2do Liste… aber für einen richtigen Artikel halt und nicht in den Kommentaren.

    @SRT und Kernzerfall: Achtung Achtung. Kernzerfälle werden maßgeblich durch die QCD bestimmt, wenn man nicht “nur einfache Tunneleffekte” betrachtet und die ist noch nicht vernünftig mit der QED, geschweige denn ART verheiratet worden. Ich bin da erst mal echt vorsichtig und müsste da noch mal viel nachlesen, bevor ich was definitives sage.

  45. #46 UMa
    19. März 2018

    Hallo Tobias, ich habe eine weitere Frage:

    Heute beträgt der U-235 Anteil 0,72%. Früher war er wegen der kürzeren Halbwertszeit von U-235 gegenüber U-238 höher.
    Z.B. gab es vor etwa 1.7 Milliarden Jahren den
    https://en.wikipedia.org/wiki/Oklo_Fossil_Reactors
    bei 3.1% U-235. Noch früher war der Anteil noch höher z.B. 8% vor 3 Milliarden Jahren 16,6% vor 4 Milliarden Jahren und 24% zur Zeit der Entstehung der Erde.

    Müssten sich dann nicht noch früher noch leichter natürliche Reaktoren gebildet haben? Oder war Sauerstoff für die Wasserlöslichkeit erforderlich?

    Kann der hohe U-235 Anteil auch zu natürlichen nuklearen Explosionen geführt haben?

    Was ist mit der Mondentstehungshypothese (siehe Wikipedia) durch eine gigantische nukleare Explosion an der Mantelkern/grenze. Ist das prinzipiell denkbar und wie könnte man das prüfen?

    Was ist mit anderen Planeten, wie z.B. Mars. Wenn dort eher freier Sauerstoff als auf der Erde vorhanden war?!

    Oder extrasolare Planeten mit anderer U-235 Anfangskonzentration, oder frühem Sauerstoffüberschuss?
    Könnte es dort öfter natürlich nukleare Explosionen geben?

  46. #47 Tobias Cronert
    19. März 2018

    Also den Sauerstoff zur Wasserlöslichkeit braucht es ja “nur” um das U235 nahe zusammen zu bekommen. Wenn das durch andere Mechanismen gelöst werden kann, dann bekommt man auch ohne Sauerstoff einen Naturreaktor hin. Man braucht ja auch nicht präzise 3% U235, sondern wieviel man braucht hängt stark von der Geometrie ab und wenn es klassische Reflektormaterialien, wie Graphit oder Beryllium in dem Boden gibt, dann kann der benötigte U235 Teil wesentlich reduziert werden. Man braucht halt auch noch disen periodischen An/Aus Schalter, damit die thermische Leistung nicht zu viel aufgestauter Wärme führt.

    Daher wird das auch mit den Explosionen schwierig, Bei der Atombombe muss man ja viel Aufwand betreiben um das U235 mit Spengstoff so lange so nahe zusammen zu bekommen, dass es Zeit hat Kernspaltung zu machen und nicht durch die produzierte Wärme direkt wieder auseinander fliegt. Das wäre halt auch das Problem, bei der natürlichen Explosion. Wenn zu viel Wärme sich aufstaut und das Gestein ein paar hundert Grad heiß wird, dann passieren grundsätzlich zwei Sachen. 1.) verbreitern sich die Neutronen absorbtionsresonanzen im epithermischen Bereich, absorbieren damit mehr Neutronen und schalten damit die Reaktion wieder aus (dauert lange (sekunden)) 2.) bewegt sich durch die Erwärmung die ganze Zusammensetzung und vielleicht liegt das U235 dann nicht mehr nah genug beieinander und der Reaktor geht aus (bzw. Wasser verdampft, wenn das zur Moderation gebraucht wird).

    Dadurch wäre mMn eine “Explosion” eher so ein Geysierausbruch und weniger ein Vulkanausbruch. Also irgendwas, bei dem nicht so viel festes Material bewegt wird, weil das der Todesstoß für die sehr besondere Geometrie wäre, die es braucht um einen Naturreaktor am laufen zu halten.

    Eine Explosion, die nur einmal funktioniert, bräuchte ja auch einen An-Knopf. Also irgendwie müsste in kurzer Zeit viel Spaltbares Material, viel Moderator (Wasser) oder viel Refflektor (Graphit, Beryllium) an die richtige Stelle kommen, an der es gerade noch dieses bischen mehr Neutronen braucht um alles in die Luift fleigen zu lassen. Also theoretisch möglich, aber unwahrscheinlich. ein unterirdischer Fluss, bei dem plötzlich ein natürlicher Damm bricht und der dann Beryllium Uran Wasser in eine poröse U235Höle spült??? Wiso nicht?

  47. #48 UMa
    23. März 2018

    Hallo Tobias, danke.
    Ok, ich bin noch nicht so ganz überzeugt, dass es so schwierig ist.
    Es gibt z.b. diese Uranvorkommen mit ca 20% Uranoxid, ca 1,3 Milliarden Jahre alt.
    https://de.wikipedia.org/wiki/Cigar_Lake
    https://de.wikipedia.org/wiki/McArthur-River-Uranmine
    Es soll keine natürliche Kernspaltung aufgetreten sein, angeblich wegen Substanzen die die Neutronen absorbieren. Kannst du dazu etwas sagen?

    Wenn sich derart große und hohe Urankonzentrationen in einer Zeit gebildet hätten, in der der U-235 Anteil deutlich über der vor 1,3 Milliarden Jahren, oder der in Oklo gelegen hat, sagen wir vor 3,5 Milliarden Jahren, müsste das dann nicht zwangsläufig überkritisch werden?

    Außerdem ist mir nicht klar, warum sich die Geometrie durch Bewegung unbedingt verschlechtern muss, sie könnte sich, wenn nicht optimal, genau so gut verbessern.
    Oder ob das Wasser tatsächlich verdampft und entweicht wenn die Tiefe und damit der Druck nur hoch genug sind?!
    Denkbare Auslöser wären auch Erdbeben, die die Geometrie plötzlich ‘verbessern’ könnten, oder?

    Was ist bei sehr großen Vorkommen mit Druckwellen durch Asteroideneinschläge? Z.B. in dem hypothetischen Mondentstehungszenario?

    Wie könnte man solche Explosionen in der Frühzeit des Sonnensystems nachweisen?

    Oder kann man natürlich Kernexplosionen irgendwie ausschließen?

  48. #49 Tobias Cronert
    23. März 2018

    Also Substanzen, die freie Neutronen absorbieren gibt es in der Erde genug. Das häufigste wird wohl Wasserstoff in Form von Wasser sein. Das hat eine ca. 20fach so hohe WK Neutronen zu absorbieren, wie Si oder so andere Materialien, aus denen “Stein” besteht. Sprich ein wenig Wasser wird gebraucht um zu moderieren, aber bitte nicht zu viel, sonst werden die Neutronen absorbiert. Wenn jetzt zufällig sowas, wie Bor vorhanden ist, dann wäre das ein ziemliches Todesurteil für jegliche Kernspaltung.

    Natürlich kann eine Geometrie durch Bewegung auch verbessert werden, aber wenn die Bewegung durch den Thermischen Output der Kernspaltung ausgelöst wird, dann ist der geometrisch ungünstige Zustand ja stabil, während der geometrisch günstige (erzeugt Kernspaltung) Zustand so lange thermische Leistung produziert, bis der Zustand wieder ungünstig wird.

    Natürliche “Kernexplosionen” kann man nicht ausschließen, aber aus dem Bauch heraus würde ich sie schon eher als unwahrscheinlich einschätzen (ohne da jetzt wirklich tiefer in die Materie eingestiegen zu sein). Nachweisen könnte man die, wie heutige Atomwaffentests auch, an ihren Abbauprodukten. Allerdings wird da bei den meisten Annahmen die lange geologische Zeit in die Quere kommen. Je länger das Ereignis her gewesen sein soll, desto schlechter lassen sich die Abbauprodukte nachweisen bzw. von ähnlichen Prozessen, die auch noch da irgendwo ablaufen unterscheiden.

  49. #50 zimtspinne
    30. März 2018

    Da du ja die Tage vielleicht ein etwas größeres Zeitfenster hast, von mir nun auch mal eine Frage, Thema Skelettszintigraphie.

    Ich frage für eine Bekannte, die von ihren Ärzten keine wirklich gute Antwort auf die Frage bekam, ob, wie und wie lange sich Angehörige nach solcher Diagnostik räumlich/körperlich von den Angehörigen, insbesondere Schwangere oder kleine Kinder, kranke Kinder u.ä. fernhalten sollten.

    Immerhin ist die “Ausstrahlung” 😉 ja stark genug, um eine verwertbare Bildgebung zu erzeugen, da muss doch nachher noch reichlich nachgestrahlt werden, so meine Gedanken dazu…..
    Ich habe mir gerade mal die physikalische Halbwertszeit angeschaut, komme dort aber nicht wirklich weiter.

    Ich würde aber mal sagen, die schwangere Person sollte sich sicherheitshalber einen Tag lang fernhalten vom Patienten, was Hautkontakt angeht sowieso… und kleine Kinder, kranke Personen ebenso.

    Finde es echt nicht schön, dass Ärzte dort nur gelangweilt abwinken, nach dem Motto: vernachlässigbar im Angesicht der Diagnosenschwere. Als ob es den Gesundheitsschutz der Angehörigen nicht gäbe…

  50. #51 Tobias Cronert
    30. März 2018

    Tolle Frage:

    Kurze Antwort: TC99 hat eine Halbwertszeit von 6h und wenn ich hier meine Faustregel anwende, dann ist alles nach 10 Halbwertzeiten (also 60h nach Herstellung des Wirkstoffes (nicht Einsatz)) weg.

    Lange Antwort: Die Aktivität bei einer Skelettszintigrafie beträgt 600MBq. Das ist eine ziemliche Menge und auch nach 10 Halbwertzeiten nachweisbar. Mit den Konversionsfaktoren entspricht die Dosis, die der Patient abbekommt ca. 4,8mSv. Das ist wesentlich weniger, als die 600MBq vermuten lassen würden, aber sicherlich richtig, da eine ausgereifte medizinische Prozedur dahinter steht. Das heißt, dass Personen, die sich viel in der Nähe des Patienten aufhalten ein-zweistellige µSv Dosen abbekommen (im schlimmsten Fall). Das ist ungefähr so viel, wie bei einer Röntgenaufnahme oder so viel, wie ich bei einem meiner Experimente abbekommen. Also schon erhöht, aber noch OK.

    Die Strahlung ist am stärksten, wenn der Patient direkt aus der Behandlung kommt und wird dann immer besser. Also den Patienten nach Hause holen ins Bett stecken oder auf die Coach vor den Fernseher und am nächsten Tag ist das meiste schon vorbei.

    Ansonsten gilt, wie bei allem mit Strahlung, dass sie gefährlicher ist, je jünger die Ziele sind. Das heißt, wenn die Oma eine Skelettszintigraphie bekommen hat, sollte sie erst mal bis am nächsten Morgen nicht mit den Enkeln spielen. Schwangere genauso (jünger geht nicht).

    Bei älteren Personen ist es egal und das Immunsystem spielt auch keine Rolle. Für Leute mit angeschlagenem Immunsystem ist ionisierende Strahlung genauso schädlich, wie für Leute mit einem einem super Immunsystem. Kein Unterschied, es erhöht halt nur das Krebsrisiko, weswegen es bei älteren Leuten halt auch mit dem Alter immer “egaler” wird.

  51. #52 Tobias Cronert
    30. März 2018

    Ansonsten komme ich irgendwie immer wieder in die Situation “medizinische” Strahlenschutzberatung zu machen. Irgendwie sind die lieben Mediziner da recht faul oder wiederholen immer nur den Standartsatz ohne sich mit dem Thema wirklich auseinander zu setzen.

  52. #53 Tobias Cronert
    31. März 2018

    PS: Sorry ich habe wieder zu sehr als Physiker gedacht. Also angeblich ist die biologische Halbwertzeit und effektive Halbwertzeit 4h bzw. 2-3 h je nach Nierenaktivität (wird über den Urin ausgeschieden). Das ändert in meiner Antwort von oben aber nur die “kurze Antwort”, weil ich in der langen Antwort mit der Dosis gerechnet habe und bei der steckt die Halbwertszeit ja schon mit drin. Außerdem stimmt die ALARA, also “Worst Case” Betrachtuntg immer noch. 😉

  53. #54 zimtspinne
    31. März 2018

    Ich hatte die Info schon weitergeleitet und soll dir ein herzliches Dankeschön übergeben.

    Mit der Halbwertszeit hatte ich mich glaub ich auch vertan.
    Hatte es mal einfach so abgeleitet, unter biol. HWZ fällt der Zerfall innerhalb des Organismus (des Patienten) durch Stoffwechselvorgänge, und die andere bezieht sich dann “logischerweise” (ähem) auf das, was außerhalb zerfällt und das Umfeld mitbetrifft.

    Jetzt hab ich doch mal nachgeschaut und gesehen, dass man beide HWZ zusammen nimmt und als effektive HWZ bezeichnet.
    Dann kommt mein Denkfehler mit innerhalb und außerhalb… ich glaube, ich hab es aber jetzt verstanden.
    Das ist aber nicht dramatisch, wichtig war ja vor allem die ungefähre Zeitspanne, auf die Stunde kommt es nicht an.
    Die Zerfalls-/Abbau-Zeiten im Körper sind ja auch interessant für diejenige…. allerdings hängt das auch vom Alter, Ernährungsweise etc ab und kann für die einzlenen Organe unterschiedlich sein.

    Die Docs sind vermutlich genervt, das alles immer wieder zu erklären und dann sehr wahrscheinlich noch mit extra Fragen “belästigt” zu werden (geht ja immerhin um Strahlung) und versuchen dem zu entgehen, indem sie das Thema kurz halten oder gleich ganz mit einem Satz abwürgen.
    Ein ausfürhliches Infoblatt vorab fände ich nicht schlecht, das die Patienten sich dann in Ruhe zu Hause durchlesen können, was eh besser ist als es unter Stressreaktion nur zu hören.
    Meine Mom hatte bei einer Schilddrüsen-Szini mal so ein Blättchen erhalten, was sie ganz gut fand.

    Hast du dir das Wissen bezügl. medizinische Be/-strahlung selbst angeeignet oder war das auch Teil deines Studiums? Interessiert mich jetzt nur mal so am Rande… 😉

  54. #55 zimtspinne
    31. März 2018

    edit/Szinti

    ach so, da fällt mir auch gerade noch auf, von Abbau (im Organismus) spricht man bei radioaktiven Stoffen ja gar nicht, bzw die werden nicht abgebaut, sondern lagern sich im Gewebe ein oder zerfallen.

    Wenn ich einlagern höre, frage ich mich ja nun schon wieder, verbleiben da eigentlich längerfristig irgendwelche Überreste in (manchen?) Organen/Geweben?
    Betrifft jetzt auch eine PET/CT etc.

  55. #56 Tobias Cronert
    31. März 2018

    Immer gerne. Es freut mich immer, wenn mein komiges Hobby zu was gut ist.

    Halbwertszeiten von Isotopen kann man extrem genau vorhersagen. Biologische Halbwertzeiten sind aus Sicht eines Physikers nur marginal zuverlässiger, als das nächstbeste Horrorskop … aber das das gilt für die meisten medizinischen Sachen. Hat wahrscheinlich mit den “menschlichen Faktoren” zu tun, die wir Physiker soweit wie möglich zu vermeiden versuchen. *g*

    Ich bin Rettungssanitäter und hatte daher Medizin als nichtphysikalisches Nebenfach gewählt (statt Mathe, Chemie oder so). Der Rest ist einfach nur Interesse.

    Radioaktive Isotope verhalten sich chemisch genauso, wie ihre nichtradioaktiven Kumpel. Das kommt daher sehr stark darauf an über welches Element wir sprechen. Exemplarisch für ungewollte Aufnahme ist immer die Iod 131 Geschichte mit den entsprechenden Iodtabletten. Aber schon für Tritium gilt z.B. das es genauso, wie normaler Wasserstoff schnell in den Körper rein und ebenso schnell wieder heraus kommt. Strontium und Cäsium lagern sich dagegen gerne langfristig in Knochen an.

    Bei nucularmedizinischen Verfahren versucht man natürlich die biologische Positionierung so zu wählen, dass es für den Patienten positiv ist. In den meisten Fällen bedeutet es, dass die Strahlung so schnell wie möglich aus dem Körper heraus soll, aber manchmal bedeutet es eben auch (z.B. wenn man mit der Strahlung einen Tumor zerstören will), dass die Strahlung lange an einer Stelle einwirken soll und darauf wird die Behandlung dann ausgelegt.

    Beim PET ist die Strahlung schnell weg und CT Kontrastmittel funktioniert anders, weil das eine Röntgentechnik ist. Außerdem wandeln sich durch den radioaktiven Zerfall die Isotope ja auch meist noch in eine anderes Element um und man muss aufpassen, dass sich das schädliche radioaktive Isotop nicht z.B. durch den Zerfall in das nicht radioaktive Arsen umwandelt. 😉

  56. #57 Trifon
    1. April 2018

    Hallo Tobias,

    deine Beschreibungen der Tests zu Geigerzähler sind hilfreich, besten Dank.
    Hast du auch weitere, qualitativ bessere Geräte getestet, oder ev. vor diese zu testen?
    Ich denke da an GQ gmc-600, bzw. GQ gmc-600 Plus.

    Oder hast du ev. eine Empfehlung?
    Ich arbeite als Ingenieur für Arbeitssicherheit und brauche einen halbwegs vernünftigen Detektor. Er muss nicht kalibriert, oder kalibrierbar sein, aber so detektieren, dass wenn ich eine offizielle Messung von behördlicher Seite anordne, ich mich anschliessend nicht blamieren muss.

    Vorab vielen Dank für deine Rückmeldung

    Gruss Trifon

  57. #58 Tobias Cronert
    1. April 2018

    Hallo Trifon,

    sorry, aber genau diese Preisspanne ist mein blinder Fleck. Ich habe halt einfach privat kein Geld um mir zum Spaß eins, geschweige denn mehrere von den “400€” Geräten zum Testen zu kaufen. Daher gibt es hier leider nur Testberichte von den billigen Geräten. Alternativ könnte ich auch mal einen Testbericht von den “Profi”-Sachen von der Arbeit machen, aber das ist 1. problematisch wg. Bildrechten etc. und 2. wahrscheinlich uninteressant, weil die Leute mit dem Budget für solche Geräte sowieso wissen, was sie kaufen.

    Ich habe auch schon öfter mal mittelpreisige Geräte, wie den Gamma Scout in der Hand gehabt und auch bei Firmen und ähnlichen Einrichtungen gesehen. Ich denke blamieren wirst du dich damit nicht. Irgendwas halbwegs vernünftigen messen die alle, ich bin für den Privatgebraucht jetzt halt nur nicht von dem Preis/Leistungsverhältnis überzeugt … und habe auch keine ausführlichen Tests gemacht. Tip: Bei der Meldung an die Behörden immer den Detektor und die Countrate mit angeben, nicht nur die Dosis. Besser wäre dann nur noch die Messung beschreiben und ein Handybild mitschicken. Die werden dann schon die richtigen Schlüsse draus ziehen.

    Wenn du dir Arbeit machen möchtest, kannst du unter den Detektoren Artikeln mal die Kommentare durchforsten. Da sind recht viele Strahlungsmesser Enthusiasten, die privat auch recht gute Detektoren verwenden und mitunter auch gute Empfehlungen aussprechen. Ggf. sind die auch mit E-Mail Adresse angemeldet und gerne bereit dir eine persönliche Stellungnahme zu Detektor XY zu geben. Gerade was Geigerzählen angeht, gibt es da eine kompetente Bastlerszene.

    liebe Grüße
    Tobi

  58. #59 m
    20. April 2018

    Besteht Gefahr, dass Fossilien (Ammoniten usw.), welche man auch zur Verzierung der eigenen vier Wände kaufen kann, verstrahlt sind? Wie hoch ist das Risiko? Und wenn sie strahlen, sind sie dann gefährlich?

  59. #60 Tobias Cronert
    22. April 2018

    Eigentlich nicht wirklich. Also die Fossilien selber sollten nicht mehr radioaktiv sein, weil z.B. alles organische C14 oder so schon lange zerfallen ist. Aber die Fossilien sind ja meist irgendwie in Steinplatten eingebaut und letztere können schon strahlen. Das ist halt bei allen älteren Gesteinsarten so, da gibt es jetzt immer irgendwelche bestimmten Steinarten, die halt natürlich vor sich hin strahlen (mitunter sehr verschiedenen Stärkegraden). Welche das genau sind kann ich dir leider nicht sagen, weil ich mit Geologie so ziemlich gar nichts am Hut habe. Eine Google Suche wird da viel auftreiben, aber mMn brauchst du dir da keine Gedanken zu machen. Das wäre halt im schlimmsten Fall so, als würde man sich irgendeine Platte exotischen designer Marmor aus Italien ins Wohnzimmer stellen. Das ist auch nicht so wild, auch wenn man mit einem Geigerzähler definitiv was messen würde.

  60. #61 m
    23. April 2018

    Habe herzlichen Dank für deine Info!
    Kurze Nachfrage: Kann natürlich strahlendes Gestein überhaupt gesundheitsschädigende Dosen aufweisen? (Wäre es z.B. bedenklich, wenn Kinder beim Spielen derartige Gesteine in den Mund nehmen?)
    Für eine erneute Beantwortung danke ich vielmals im Voraus!

  61. #62 Tobias Cronert
    24. April 2018

    Also natürliches Gestein kann durchaus erhebliche Dosen aufweisen, die im Strahlenschutz als gesundheitsschädlich gelten. Als konkretes Beispiel habe ich oft künstlich bestrahlte Proben, die immer verschlossen/einegschweißt sehr vorsichtig gehandhabt werden und nur aus dm Bleischrank herauskommen, wenn man sie unbedingt braucht … aber durchaus eine geringere Aktivität haben, als manche natürlichen Steine.

    Die Grenze “Gesundheitsgefährlich” ist natürlich von Menschen definiert worden und dementsprechend willkürlich. Aber manche natürliche Gesteine würe ich in den Händen von Kindern als ähnlich gefährlich ansehen, wie einen Brocken Asbest/Eternit oder sonstwelchen krebserregenden Stoffen.

  62. #63 UMa
    24. April 2018

    Hallo Tobias,

    ich bin noch nicht ganz so überzeugt, dass es wirklich so schwierig ist mit den natürlichen nuklearen Explosion. Insbesondere bei hohem Druck im Erdinneren oder plötzlichen Verschiebungen durch Erdbeben oder die Entstehung des Erdkerns.

    Aber ich habe noch eine weitere Frage. Wie lange hält sich das Material aus dem Satelliten bestehen in der kosmischen Strahlung? Asteroiden verändern sich ja auch durch die kosmische Strahlung. Wäre nach einer oder hundert Millionen Jahren noch etwas übrig? Noch so gut wie neu oder völlig zerbröselt?

  63. #64 Jule
    29. April 2018

    Hi Tobias,

    folgende Frage: Ist Gartenerde (gekaufte und die, die man schon vorfindet) eigentlich radioaktiv strahlend? Und wenn ja, ist sie dann vom Niveau her gefährlich?
    Wenn du antworten könntest, wäre ich dir sehr verbunden. Vielen Dank!

    MfG
    Jule

  64. #65 Tobias Cronert
    30. April 2018

    Hallo Jule,

    also grundsätzlich ist viel Erde in Deutschland noch sehr leicht von Tschenobyl belastet. Das ist zwar messbar, aber harmlos.
    Blumenerde ist im Allgemeinen weniger radioaktiv, als normale Erde aus dem Garten, weil da weniger “Gestein” drin ist … also vor allem noch harmloser, als eine Schüppe Erde aus dem Wald.

    Also alles kein Problem, wenn man mal von total hypothetischen, theoretisch konstruierten Unfallszenarien absieht.

    liebe Grüße
    Tobi

  65. #66 UMa
    1. Mai 2018

    Nachtrag zu meiner Frage: Offenbar wird Mondgestein auf der Oberfläche um etwa 1mm pro Million Jahren durch Mikroimpakte erodiert. Das dürfte ähnlich auch auf Satelliten zutreffen. Das könnte die Auswirkungen kosmischer Strahlung übertreffen.

  66. #67 Tobias Cronert
    1. Mai 2018

    Hallo UMa,

    also grundsätzlich verursachen Sonnenwinde und andere kosmische Strahlung hauptsächlich Strukturschäden durch Implantation von zusätzlichen Teilchen. Wenn man ein schönes Stück Stahl (oder Microcontroller) hat und einfach wild Protonen da reingebuttert werden, dann stört das die Funktionalität meist enorm.

    Aber das ist wahrscheinlich nicht, was du denkst. Also richtig Material “wegsandstrahlen” kann man mit kosmischer Strahlung nur sehr bedingt (eigentlich nicht), weil der Teilchenfluss sehr klein gegenüber der Avogadro-Konstanten ist und es lange dauert, bis etwas makroskopisch passiert. Die meisten Effekte sind wie oben schon angedeutet mikroskopisch.

    Mikroimpakte ist mWn noch mal was ganz anderes, als kosmische Strahlung. Also echter kosmischer Staub im µm Bereich vs. Protonen (Strahlung) ganz andere Liga. Aber da habe ich schon zu wenig Ahnung von Astrophysik für.

    Über natürlich nukleare Explosionen könnte man sicher auch mal einen eigenen Artikel zusammenphilosophieren. Ich fürchte, dass wir da in den Kommentarzeilen nicht all zu weit kommen *g*

  67. #68 Jule
    9. Mai 2018

    Hi Tobias,

    ich bins nochmal: Jule. Zuerst möchte ich dir vielmals danken für deine Nachricht!
    Dann hätte ich noch eine Frage:
    Sind Partikel aus Kamin(gängen) theoretisch eine radioaktive Gefahrenquelle (ob nun mit Kohle, Holz oder Gas geheizt wird)?
    Und falls man radioaktive Partikel in den Körper aufgenommen haben sollte, wie gefährlich ist so etwas? Wird bei niedriger Strahlendosis alles ausgeschieden? Oder bleibt alles drin? (Wenn ich mich z.B. an deinen interessanten Bananen-Artikel richtig erinnere, steht dort, dass die aufgenommene niedrige Strahlung der Bananen auch wieder ausgeschieden wird, richtig? Niedrige Dosen summieren sich nicht über Tage, wenn ich beispielsweise jeden Tag Bananen essen würde?)

    Wenn du antworten könntest, wäre ich dir sehr dankbar!

    MfG
    Jule

  68. #69 Tobias Cronert
    10. Mai 2018

    Hallo Jule,

    also grundsätzlich hat alles, was mit Kohle oder Gas zu tun hat einen messbaren radioaktiven Anteil. Das ist normal (bei Erdgas z.B. hier) bei allem “was aus der Erde kommt”.

    Ob das jetzt in den Körper eingebaut oder wieder ausgeschieden wird hängt von der entsprechenden Substanz ab. (Radioaktives) Kalium (aus der Banane) z.B. wird bis zu einer bestimmten Schwelle in den Körper eingebaut und alles, was darüber hinaus geht wird ausgeschieden. Das heißt jeder Mensch ist durch das Kalium ein kleines wenig Radioaktiv, aber eben nur ein kleines bischen, weil es diese obere Grenze gibt. Für Caesium gilt mehr oder weniger das gleiche.

    (Radioaktives) Strontium dagegen wird wegen der chemischen Ähnlichkeit mit Calcium in Knochen eingebaut und kommt da auch nicht mehr raus. Das sammelt sich da “für immer” an und summiert sich auf.

    Das ist eine Eigenschaft, die man bei Fossilen Brennstoffen “auf dem Schirm” hat. Sprich man vermeidet in der Praxis, dass sich zu viele Schlacke und feste Ablagerungen aus Kohle oder Erdgas irgendwo sammeln. Wenn also im Privathaushalt der Schornsteinfeger alles vernünftig abgenommen hat, dann ist aus Strahlenschutzsicht alles OK. Im Industriellen Maßstab muss man da zwar schon etwas mehr Aktrobatik betreiben, aber grundsätzlich läuft das genauso. Radioaktivität ist da eben wie jeder andere chemische Gefahrenstoff (Asbest, Kohlenmonoxid etc. pp.). Man muss halt aufpassen und sich drum kümmern.

    Gruß
    Tobi

  69. #70 zimtspinne
    10. Mai 2018

    Was Kamine, Holzöfen und ähnlichen Krams angeht, sollte man sich besser Gedanken um die Feinstaubbelastung machen.
    Es gibt (privat) kaum größere Dreckschleudern als holz- und müllbefeuerte Kaminöfen – Kachelmännchen hatte mal einen Vergleich mit Dieselautos gebracht, allerdings ist er eh ein großer Kritiker oder besser gesagt Wetterer (hehe schön doppeldeutig) der neuen Kaminöfenkultur. Er setzt auch Messgeräte in seiner Privatforschung ein und wird sicher inzwischen von allen Nachbarn gehasst^^

    Strontium – interessant zu wissen.
    Ich denke aber, wenn das Zeug schon immer oder sehr lange in Nahrung und Trinkwasser vorkommt, wird der Mensch es in den üblichen Mengen auch gut kompensieren können. Da sehe ich eher eine Gefahr, falls die Mengen sich plötzlich stark erhöhen…. warum auch immer.
    Vielleicht hat das Zeug in den Knochen ja sogar eine sinnvolle Aufgabe zugewiesen bekommen, wenn es eh schon mal da ist… es gibt ja unzählige Spurenelemente, die ihre Funktion im menschlichen Organismus haben bzw sogar essentiell sind.
    ups, nun komme ich aber gerade ins Schleudern…. ist (radioaktives) Strontium nochmal was gesondertes? Oder ist Strontium immer radioaktiv? Oder mal mehr und mal weniger, je nach Zustand? Fragen über Fragen….

  70. #71 Tobias Cronert
    10. Mai 2018

    Strontium88 ist das normale, nicht radioaktive Strontium und kommt auch ganz normal im menschlichen Körper vor. Ich habe keine Ahnung ob das da jetzt irgendeine sinnvolle Funktion hat, aber aus dem Bauch hätte ich jetzt mal eher Nein gesagt.

    Darüber hinaus gibt es halt noch das radioaktive Strotium90. Das ist ein Spaltprodukt von Uran und tritt eben überall dort auf, wo es auch Uran gibt, von Tschernobyl bis zu natürlichen Gesteinen. Es hat eine unbequeme Halbwertszeit von 30 Jahren, weshalb halt immer noch viel von Tschernobyl davon im bayrischen Wald übrig ist und dort vom Hobbyschtrahlenschützer nachgewiesen werden kann.

  71. #72 zimtspinne
    10. Mai 2018

    Doch, Strontium ist (wie ich vermutete) ein Spurenelement, das der Körper beispw. in der Entwicklung zum Aufbau des Knochengewebes benötigt. Als Medikament gibts das auch… (das hat mir jemand vorhin mitgeteilt, der sich damit etwas besser auskennt, den ich gerade zufällig zur Hand hatte, also an der Strippe).
    ok, das ist also der Mist aus Tschernobyl mit dieser monströsen Halbwertszeit.
    Bedeutet das, nach 30 Jahren ist es verschwunden, abgebaut, wie auch immer? Kann man sich auf diese Halbwertszeit verlassen? Ich hab da gerade so leichte Zweifel, andererseits sammle ich Pilze seit Jahren, mit oder ohne Strontium90.
    Stundenlang in der Mittagssonne braten, ist vermutlich gefährlicher…

  72. #73 zimtspinne
    10. Mai 2018

    Korrektur:
    ich weiß natürlich, was Halbwertszeit bedeutet, ich meinte es eher so, ist dann soviel des Stoffes abgebaut, dass es langsam aber sicher in Richtung Unbedenklichkeit geht?

  73. #74 Tobias Cronert
    10. Mai 2018

    Naja es gibt halt die Faustregel im Strahlenschutz, dass nach 10 Halbwertszeiten “alles weg” ist. https://scienceblogs.de/nucular/2016/04/19/kurz-notiert-faustregel-10-halbwertszeiten/

    Richtig heißt das ja nur, dass es auf 1/1000 abgesunken ist, aber hier sieht man halt gut, dass vor allem die mittleren Halbwertszeiten problematisch sind, denn 300 Jahre “bis alles wieder harmlos ist” ist für Strontium90 halt schon eine lange Zeit.

  74. #75 Jule
    12. Mai 2018

    Hi Tobias,

    ich hätte folgende Nachfrage zum Themenkomplex rund um Kamin etc.: Das Haus, in dem ich aktuell wohne, hat eine Vorratskammer, hinter deren Wand ein Kaminschacht verläuft. In der Wand (des Vorratszimmers) ist eine ca. DINA4-große Öffnung (wohl irgendwelche Belüftungslöcher!?). Ist davon auszugehen, dass von dort (zuviel) radioaktiv belastete Partikel aus dem Kamin in die Wohnung dringen?

    Und generell: ab welcher Größe bzw. Ungröße sind radioaktive Partikel (k)eine Gefahr? Muss man sie sehen können? Sind mikroskopisch kleine Partikel immer noch schädlich?

    Bin schon gespannt auf deine Antwort und danke dir vorab!

    LG
    Jule

  75. #76 Tobias Cronert
    12. Mai 2018

    Hi Jule,

    wie schon gesagt, ein vernünftiger deutscher Kamin, der von einem regulären Schornsteinfeger abgenommen wurde ist auch in Hinsicht auf Radioaktivität in Ordnung. Es muss schon echt viel Kaminschlacke zusammen kommen, damit die Radioaktivität auch nur ansatzweise gefährlich wird. In Deutschland passiert sowas in der Regel nur in irgendwelchen Erdgasverteilerzentren, Kohlenkraftwerken etc. und auch da wird es dann meist entsprechend überwacht und kontrolliert.

    Die physikalische Größe ist bei radioaktiven Partikeln egal. Auch die kleinsten Partikel können je nach Aktivität gefährlich werden. Man kann verdammt viel “Strahlungsenergie” in verdammt kleine Teilchen quetschen, die man nur noch unter dem Mikroskop sehen kann. Eine tödliche Dosis Polonium210, wie sie bei dem Mord an dem ehem. Agenten Litvinienko benutzt wurde war z.B. nur 50ng. Das kann man mit dem bloßen Augen schon nicht mehr sehen. 1ng ist ungefähr eine menschliche Zelle.

    beste Grüße
    Tobi

  76. #77 Renée
    13. Mai 2018

    Wenn selbst mikroskopisch kleine Partikel gefährlich sein können, was ist dann von einem japanischen Touristen zu halten, der mit seinem Rucksack und/oder Kleidung ständig in meinen Rücken an meinem eigenen Rucksack drückte (weil so viele Touristen auf engem Raum rumstanden)? Kann der Abrieb eventuell etwas verschleppen? (Stichwort Fukushima)
    Was sagt der Fachmann?

  77. #78 Tobias Cronert
    13. Mai 2018

    Klar kann der jap. Tourist Tourist Kontamination verschleppen … ist nur die Frage, wieviel.

    Der Tourist aus Bayern, dessen Rucksack auf einem Bayrischen Waldboden lag verschleppt von diesem Waldboden auch Tschernobylkontamination zu mir nach Köln. Das ist aber erstens so wenig, dass ich es kaum noch nachmessen kann (also nur, wenn ich mir viel Mühe gebe) und zweitens harmlos im Vergleich mit anderen Umweltgiften/Gefahrstoffen.

    Um jetzt mehr Kontamination, als der Bayern Tourist zu haben müsste der Japaner:
    1.) Aus der Nähe von Fukushima kommen.
    2.) Außerhalb der kontrollierten Gebiete Bodendreck aufgenommen haben …
    3.) … der dann über den ganzen Flug bis Deutschland an dem Rucksack geblieben ist, bis er an einem anderen Rucksack abreibt.

    Das ist zwar möglich, aber unwahrscheinlich. Schon wenn der Japaner aus Oasaka und nicht aus Sendai kommt, dann hat er weniger Kontamination, als der Bayer.

    Boden-Kontamination besteht meist aus Sand und anderen ähnlichen Sedimenten, die sich ebensoleicht vertielen, wie Sand eben. Dadurch verdünnt sich eine Kontamination ziemlich schnell zu harmlosen, kaum noch nachweisbaren Mengen. Vor allem, wenn so viel Entfernung, wie bei Japan im Spiel ist. Wir bekommen hier in Deutschland von den Resten von Tschernobyl viel mehr Strahlung ab, als alle japanischen Touristen zusammen einschleppen könnten.

  78. #79 Jule
    14. Mai 2018

    Hi Tobias,

    ich möchte dir nur nochmals vielmals danken für deine Hilfe und deinen Blog!
    Deine Artikel sind stets sehr interessant wie informativ – deinem Blog bleibe ich auf jeden Fall treu, und freue mich auf zukünftige Artikel!

    LG
    Jule

    • #80 Tobias Cronert
      14. Mai 2018

      es freut mich von Nutzen sein zu können. Immer wieder gerne und danke für die netten Worte.

  79. #81 zimtspinne
    15. Mai 2018

    Huhu Tobias,

    für dich heute mal ein Schmakerl aus einem Tatort, bevor ich den lösche. Ich mach ja bei solchen Sprüchen immer gerne mal einen Faktencheck 😉
    Sonst eher für Forensiker, heute mal was für dich:

    Es lässt sich tatsächlich wissenschaftlich ermitteln, ob ein Wein vor oder nach 1945 gekeltert wurde. Alle Weine, die nach Hiroshima gekeltert wurden, weisen Spuren erhöhter radioaktiver Strahlung auf.

  80. #82 Tobias Cronert
    15. Mai 2018

    Danke für die Vorlage: Etwas pauschal ausgedrückt, aber grundsätzlich haben die großen deutschen Entertainer da schon Recht.

    Wir hatten Schulklassen immer Erde aus dem Garten mitbringen lassen und da kann man rel. leicht am Iod und Cs herausfinden, ob sie aus einem Neubaugebiet (Garten nach 1986 umgegraben) oder einem älteren Garten stammen.

    Das gleiche Spiel lässt sich auch mit Vor- und nach der Atombombe betreiben, wobei die ganzen militärischen Tests da wesentlich mehr Isotope hinterlassen haben, als Hiroshima und Nagasaki.

    Es kommt halt weniger auf die “Radiaoktivität” an, als auf die speziellen Isotope, aber grundsätzlich kann man mit dem ganzen künstlichen Krempel, den das Atomzeitalter in die Luft gejagt hat eine zienlich präzise altersbestimmung machen.

    Das gilt allerdings auch für geologische Zeiträume und Perioden, wo ich anhand von Isotopenkonentration local oder global datieren kann. Da ist dank der hohen Präzision der Isotopenmessungen mittlerweile sehr viel möglich.

  81. #83 Renée
    15. Mai 2018

    Birgt eigentlich der Gebrauch von Unterstellern/Untersetzern (für Gläser, Tassen etc.), bestehend aus Schiefer, eine Gefahr (im radioaktiven Lichte betrachtet)?

    Gruß

  82. #84 Tobias Cronert
    15. Mai 2018

    Nein. Untersetzer aus egal welchem Gestein sind keine wirkliche Gefahr.

  83. #85 Renée
    5. Juni 2018

    Hallo,
    bei Feuerbestattungen werden ja gerne Schamottsteine (mit Kennnummer zur Identifizierung des Verstorbenen) verwendet – können diese Steine auch (allzu) radioaktiv sein, und falls ja, könnte dann von dieser Strahlung irgendetwas auf die Asche übergehen? (Schamott ist doch mit Ton verwandt und Ton vermag Radioaktivität zu binden??)
    Vorab vielen Dank für die Beantwortung!

  84. #86 Tobias Cronert
    6. Juni 2018

    Also ich habe wenig Ahnung von Steinen und/oder Geologie, aber wenn Schamott (wie Wikipedia behauptet) fast ausschließlich aus SiO2 und Al2O3 besteht, dann wäre es absolut ungefährlich.

    Grundsätzlich gilt die Faustregel: “je älter und massiver das Gestein, desto mehr Radioaktivität”. Granit, Marmor, etc. haben mehr Radioaktivität als Sand- oder Kalkstein.

    Selbst wenn bei der Feuerbestattung Gestein mit natürlichem Urangehalt benutzt werden würde, würde nur minimal Radioaktivität auf die Asche übergehen. Radioaktivität ist nicht ansteckend, sondern um übertragen zu werden müssten Teile von dem Stein abbrechen und in die Asche gelangen. Das passiert zwar in geringen Maße, aber die Radioaktivität ist proportional zu der Menge an Gestein, die von dem Stein in die Asche glangt. Wenn also bei jedem Verbrennungsvorgang 1g Steinstaub in die Asche kommt, dann kommt auch max. 0,001% der Radioaktivität des Staubes in die Asche und das wäre verschwindend gering.

    Ansonsten noch als Nebenbemerkung: Ton kann keine Radioaktivität binden. Ton kann nur radioaktive Partikel binden, so wie kontaminiertes Wasser etc. pp.

  85. #87 Halo
    8. Juni 2018

    Hallo,

    Eine Frage an den Strahlen-Physiker:
    Eine Frage beschäftigt mich schon seit vielen Monaten. Es geht um eine Reise zum Mars. Kann eine Besatzung überhaupt unter günstigen Weltraumbedingungen eine etwa 9 monatige Reise zum Mars überstehen oder ist generell davon auszugehen, daß diese verunglücken, also der gesamte Verlust der Besatzung zu beklagen ist.

    Laut Wikipedia:
    https://de.wikipedia.org/wiki/Strahlenexposition#%C3%9Cbersicht:_Kosmische_Strahlung
    ist die “kosmische Strahlung im interplanetaren Raum” bei etwa ~200mSv pro Jahr.

    Wie lange würde ein Mensch unter derartigen Bedingungen überleben können und
    was wären die Symptome mit denen ab wann zu rechnen ist ?

    Ich versuche nämlich zZt. für mich abzuschätzen wieviele Personen (Es sollen ja einige Dutzend zum Mars gebracht werden) hier vorab verunglücken noch bevor diese den Mars erreichen.

    Gute Besserung und mit freundlichen Grüßen,
    Halo

  86. #88 Tobias Cronert
    8. Juni 2018

    Hallo Halo,

    Also 200 mSv verursacht “nur” stochastischen (also Wahrscheinlichkeit Krebs zu bekommen) und keinen deterministischen (direkte Zellschäden) Effekt. Das heißt, dass höchstwahrscheinlich alle die Reise zum Mars und zurück überleben (aus Strahlenschutzsicht). Weil sie die erhöhte Strahlung abbekommen haben, haben sie ein stark erhöhtes Risiko später Krebs zu bekommen, aber der wird in der Regel erst 10 oder 20 Jahre nach Strahlenexposition auftreten. Das ist wie beim Rauchen, der Krebs kommt erst Jahre später.

    Direkte Schäden (in dem Bild die blaue Kurve) treten erst ab 1 Sv innerhalb kurzer Zeit (Stunden max. Tagen) auf. Das ist dann die sog. Strahlenkrankheit. Die 200mSv / Jahr kosmische Strahlung sind im Bereich des grünen Kreises und dort ist die Wahrscheinlichkeit für direkte Schäden verschwindend gering.

    Siehe dazu auch https://scienceblogs.de/nucular/2015/02/11/strahlungsschaeden-iii-eindeutigkeit-bestrahlungkrankheit/

    liebe Grüße und Danke für die guten Wünsche
    Tobi

  87. #89 Halo
    15. Juni 2018

    Wie ist das eigentlich mit Pflanzen, Reptilien, Insekten.
    Sind primitive Lebensformen anfälliger für erhöhte Strahlenbelastung ?

  88. #90 Tobias Cronert
    15. Juni 2018

    Das kann man nie so pauschal sagen. Die Resistenz von manschen primitiven Lebwesen gegenüber Strahlung ist ja schon fast legendär (Kakerlaken etc. pp.), währen manche seh sensitiv darauf reagieren.

    Das gilt auch für den Pflanzenbereich, in denen manche schon bei den kleinsten Veränderungen in die Knie gehen und anderen die extremsten Situationen überleben können. Es gibt Moose und Flechten, die Strahlendosen aushalten, die Stahln und andere unbelebte Dinge in die Knie zwingen.

  89. #91 Renée
    17. Juni 2018

    Hallo Tobias,

    zunächst das Wichtigste: ich wünsche dir gute Besserung!

    Sodann das Zweitwichigste: vielen Dank für deine steten wie hilfreichen Erläuterungen (die du trotz deiner Erkrankung nicht unterlässt) – danke!

    Für den Fall, dass du Zeit für eine Beantwortung findest, würde ich dir folgende Fragen stellen:
    -Speckstein ist aufgrund seiner Porosität wohl eher ungefährlich (kann man daher sicherlich auch als Buch-Beschwerer benutzen)?
    -Granit soll ja sehr anfällig sein für Radioaktivität. In meinem Keller soll eine ganze Wand daraus bestehen (ansonsten ist im Haus viel Sandstein verbaut) – ergeben sich hierfür Dinge, die man beachten sollte (z.B. hinsichtlich der Lagerung von Lebensmitteln, Strahlung ins Erdgeschoss hinein usw.)? Was würdest du als Fachmann zu Bedenken geben?

    Vorab wie immer herzlichen Dank!
    Grüße
    R.

  90. #92 Tobias Cronert
    17. Juni 2018

    Solange ich es schaffe, werde ich auf jeden Fall hier weiter machen und sobald ich kann alles beantworten, zu dem ich eine Antwort kenne. Das macht Spaß und wenn ich jemandem helfen kann, um so besser. Ich freue mich immer, wenn das Angebot angenommen wird und für andere Leute nützlich ist.

    Aber mal konkret zum Stein. Speckstein ist von der Radioaktivität her soweitich weis total harmlos. Granit ist dagenen das “Problemkind” ´, aber vielleicht ein wenig anders, als du gerade denkst. Also Lebensmittel irekt davon stellen ist vollkommen OK. Radioaktivität ist NICHT ansteckend, sondern wirkt nur direkt. Also die Sitzecke lieber an die andere siete des Raumes, aber die Lenesmittel ist OK. Personnen “Nein”, unbelebte Materia “Ja”..

    “Altes Getein”, wie Granit gibt auch noch das radioaktive Gas Radon ab. Also wenn man viel altes Gestein im Keller hat öfter mal lüften mit Durchzug. Aber das reicht im Alltag.

  91. #93 Renée
    18. Juni 2018

    Tobias, lieben Dank!
    -Rätst du auch, im Erdgeschoss (wo angeblich keine Granit-Wände mehr sein sollen) die Sitzecke dort anzusiedeln, wo sie möglichst fern ist von der Granit-Kellerwand? (Wieviele Meter sollte man von der Granit-Wand entfert sitzen?)
    -Sicherlich gibt es auch im Einzelfall Granit, der kaum (oder gar nicht) strahlt?
    -Was weißt du über Dachziegel? Bestehen hier etwaige Risiken (z.B. im Dachboden)?

    Ich hoffe, ich nerve nicht allzusehr mit meinen Fragen…
    Ein großes Dankeschön wieder im Voraus!

    Viele Grüße
    R.

  92. #94 Tobias Cronert
    18. Juni 2018

    Wenn noch eine Deckenwand dazwischen ist, ist alles gut. Granit ist im Reglfall schon relativ harmlos. Eiegntlich sind es beim Granit selbst schon nur die Sonderfällte, die ernstahaft strahlen. Faustregel: Je exotischer der Granit (mehr Adern und Wehnen und natürliches buntes Zeug im Stein) desto mehr strahlt es. Je homogener, desto weniger.

    Dachziegel: Ton -> gar nicht. Schiefer -> ein bichen, je nachdem welcher konkret, aber auch nicht ersthaft, das man was tun müsste.

  93. #95 Renée
    26. Juni 2018

    Hallo Tobias,

    gut, dass deine Behandlung nach Plan verläuft. Weiterhin gute Besserung! Eine völlige Genesung ist (mittel- bzw. langfristig) greifbar?

    Ich hätte noch einige abschließende Fragen zum Themenkomplex Gestein und Radioaktivität:
    -Die Granit-Wand meines Kellers ist mit einer Gips-Schicht verputzt, die, wenn ich es richtig überblicke, zumindest leicht porös ist und leicht bröckelt. Gehen von dieser Schicht etwaige Gefahren aus?
    -Wie verhält es sich diesbezüglich mit Beton (bzw. Beton-Partikeln)?
    -Granit bröckelt nicht, oder (weil es sehr massiv ist)? Ich frage mich, ob hiervon Gefahren ausgehen könnten z.B. hinsichtlich der Lebensmittellagerung und potentieller Verschleppung von Granit-Partikeln…
    -Wie du schon in jüngster Vergangenheit mitteiltest, lassen sich vor der Granitwand problemlos Lebensmittel bzw. Objekte – nur keine Menschen! – lagern. Gibt es theoretisch irgendwelche Objekte, bei denen es doch Probleme geben könnten (die also doch zu Strahlern werden könnten, z.B. Metall)?

    Für die schnelle und kompetente Beantwortung danke ich wie immer vielmals im Voraus!

    LG
    Renée

  94. #96 Tobias Cronert
    27. Juni 2018

    -Nein

    -Nein

    -Vielleicht, aber nicht mehr als von den Granit Partikeln in der Hauseinfacht, am Supermarktparkplatz etc. pp. Normalen Granit können wir Menschen ab, das passt schon.

    – Nein. Mit der Ausnahme von Mikroelektronik. Die können zwar nicht zu Strahlern werden, aber beschädigt werden. Je sensibler die Elektronik, desto anfälliger. Die Warscheinlichkeit ist aber sehr sehr gering. Wenn du Mikroelektrinok im Keller lagers, dann wird durch die Abschirmung des Kellers schon mehr gewonnen, als du durch die Granitwand wieder verlierst.

    https://scienceblogs.de/nucular/2017/10/18/wohngebaeudeversicherung-vor-radioaktivitaet/

  95. #97 Renée
    28. Juni 2018

    Danke!
    Letzte Fragen:
    -Falls Gesteins-Partikel (Granit usw.) doch irgendwie in den menschlichen Körper verschleppt werden, werden dann diese (mit ihrer Strahlung) ausgeschieden oder eingelagert?
    -Sind eigentlich Bergwanderer bzw. -steiger einer erhöhten (gar zu hohen) Strahlung ausgesetzt?

  96. #98 Tobias Cronert
    29. Juni 2018

    -Granitpartiekl werden einfach wieder ausgeschieden und sind dann weg.

    -Bergwanderer etc. sind schon erhöhter Strahlung ausgsetzt. Aber das ist OK so. Der menschliche Körper ist durchaus dafür ausgelegt in den Alpen zu wohnen und kommt mit der natürlichen Radioaktivität, die dort ein gutes Stück höher ist, als z.B. in Hannover, auch gut klar. Da haben wir uns evolutionär dran angepasst.

  97. #99 Jule
    6. Juli 2018

    Hey Tobias,

    ich wünsche dir auf jeden Fall gute Besserung!
    Wie deinem Blog zu entnehmen ist, ist Radioaktivität ja nicht ansteckend, nur radioaktive Partikel sind auch weiterhin gefährlich… Aber hinterlässt die Strahlung selbst (ohne Partikel) nicht auch gewissermaßen “Partikel”, nur im Nano- bzw. Mikro-Bereich? Woraus besteht die Strahlung (Teilchen oder Wellen…)?

    Liebe Grüße
    Jule

  98. #100 Tobias Cronert
    6. Juli 2018

    Hi Jule Danke,

    die Strahlung besteht auch aus Partikeln (Quantenmechanik Partikel = Welle), die sind aber noch mal 1 Tausend mal kleiner als der Nano Bereich. Das heißt wir reden hier über einzelne Atome (Alpha- oder Neutronenstrahlung), Elektronen oder Photonen (Lichtteilchen). Das Leben dieser Teilchen wird hauptsächlich durch die Gesetze der Quantenmechanik bestimmt, daher funktionieren sie nicht wie “Staubteilchen”, dass sie von einem Gegenstand auf einen anderen übertragen werden würden.

    Meine kalten Neutronen z.B. sind gleichzeitig ein Teilchen (ein Atombaustein mit einer Masse von 1 U und einer Geschwindigkeit von 200 m/s) und eine Welle mit einer Wellenlänge von 5 Angström (wie Licht z.B.). Quantenmechanik live sozusagen. 😉

  99. #101 Jule
    7. Juli 2018

    Hi Tobias,

    das heißt, an den Objekten (in der Nähe eines Strahlers) sammeln sich keine radioaktiven Mini-Partikel an? Oder bekomme ich als Person doch einige Mini-Partikel ab, falls ich ein solches Objekt in die Hand nehmen würde?

    Nochmals vielen Dank!

    Liebe Grüße
    Jule

  100. #102 Tobias Cronert
    7. Juli 2018

    Genau. An einem Strahler sammeln sich keine radioaktiven Mini-Partikel an und eine Person bekommt auch keine Mini-Partikel ab, wenn sie es in die Hand nimmt.

    Es sei denn von dem Strahler bröckelt etwas ab.

    Kleiner Disclaimer: Das gilt für ca. 95% aller Strahlung. Einige sehr spezielle Strahlung kann kleine Partikel erzeugen. Aber das ist nur seh sehr spezielle Strahlung, der man im Alltag eigentlich nicht über den Weg läuft.

  101. #103 Jule
    7. Juli 2018

    Der “kleine Disclaimer” ist ja interessant. Was für Strahlung wäre denn das genau (oder beispielsweise)?

  102. #104 Tobias Cronert
    8. Juli 2018

    Zum Beispiel Neutronenstrahlung oder hoch energetische Schwerionenstrahlung aus einem großen Teilchenbeschleuniger. Wenn die Strahlung z.B. schon selber aus Kohlestoffatomen besteht, dann ist die Wahrscheinlichkeit recht hoch, dass sie das ziel, auf dass sie triff aktiviert. Das heißt ein Zielatom in ein anderes “verwandelt” (zum Beispiel Beryllium in Lithium … oder so). Die Atome in die das Material verwandelt worden ist, die sind dann oft radioaktiv, weil sie stark angeregt sind (aber in der Regel mit einer sehr kurzen Halbwertszeit).

    Aber für solche Späßchen braucht man eben meist einen großen Teilchenbeschleuniger oder kosmische Strahlung und bei beiden ist die Menge der erzeugten neuen (radioaktiven) Teilchen sehr sehr gering.

  103. #105 Jule
    10. Juli 2018

    Nochmals vielen Dank, Tobias!
    Dein letzter Kommentar ist ja wirklich sehr interessant – die sich verwandelnden Atome erinnern ja fast ein bisschen an die Alchemie (ein bisschen Umwandlung in Gold wäre ja gar nicht schlecht 🙂
    Zum vorletzten Kommentar habe ich nur zur Sicherheit eine kleine Nachfrage: An einem Strahler sammeln sich keine radiaoktiven Mini-Partikel an – dies gilt aber auch für Gegenstände, die selbst nicht strahlen und die in Reichweite des Strahlers (für lange Zeit) liegen?

    Liebe Grüße
    Jule

  104. #106 Jule
    10. Juli 2018

    Ach ja, ich vergaß: Wie ist es eigentlich um Holz bestellt? Kann es radioaktiv sein/werden? Ich frage natürlich auch mitunter wegen alltäglicher Gebrauchsgegenstände: Regale, Möbel, Holz-Kisten etc. (gibt es Holzarten und Orte, die eher mit Radioaktivität zu tun haben?)

    Danke!
    Jule

  105. #107 zimtspinne
    11. Juli 2018

    ist jetzt keine Frage, aber ich wusste eben nicht wohin damit, also hier…..
    vielleicht interessiert dich ja dieses neue schicke Gerät:
    Interventions-CT
    https://www.mittelbayerische.de/region/regensburg-stadt-nachrichten/das-geraet-das-den-krebs-gart-21179-art1668615.html

    es hat ja als CT in gewissem Umfang auch mit Strahlen zu tun….

  106. #108 Tobias Cronert
    12. Juli 2018

    Hi Jule,

    auch bei langer Zeit nehmen sammeln sich keine radioaktiven Partikel an. Die Zeit ist egal, das einzige, was zählt ist die Energie.

    Holz sammelt idR keine radioaktiven Partikel auf. Zumindest nicht in größerem Umfang als alles andere auch. Natürlich ist alles aus Kohlenstoff radioaktiv wegen dem C14 Gehalt. Aber das gilt ja für alle Lebewesen, Pflanzen, etc. pp.

  107. #109 Jule
    13. Juli 2018

    Lieber Tobias, vielen lieben Dank!
    Ich wünsche dir weiterhin gute Besserung – und ich freue mich auch auf neue wissenswerte Inhalte in deinem Blog!
    Jule

  108. #110 Renée
    18. Juli 2018

    Hi Tobias,
    ich möchte nur noch mal kurz Bezug nehmen zu #77 + #78 (hier bei diesen Fragenartikeln):
    Der Rucksack des Japaners vermag quasi keine Strahlung zu verschleppen (wie du überzeugend darlegtest). Ich habe mich nur noch gefragt, ob der Rucksack selbst radioaktiv sein könnte, denn vielleicht ist von seinen Bauteilen (Kunststoffe, Wasser etc.) etwas durch die Fukushima-Katastrophe radioaktiv geworden (Strahlung ist ja auch schon in der Lebensmittelkette Japans entdeckt worden). Und wenn dergestalt der Rucksack selbst quasi ein Strahler sein sollte, könnte dieser dann durch stärkere Reibung an meinem Rucksack Strahlung (in Form von kleinen Rucksack-Partikeln) hinterlassen!?

    Ich weiß, das ist jetzt sehr theoretisch – aber als Rätselfreundin fand ich dieses Gedankenexperiment interessant… Was sagt der Fachmann dazu?

    Beste Grüße
    Renée

  109. #111 Tobias Cronert
    21. Juli 2018

    Hallo Renée,

    ich finde es auch immer interessant zu gucken, in welchen Alltagsgegenständen Radioaktivität versteckt sein könnte. Also Plastik sind ja grundsätzlich Kohlenhydrate, das heißt entweder der Kohlenstoff oder der Wasserstoff könnten Radioaktiv sein. Beim Kohlenstoff wäre das das berühmte C14 Isotop, dass man zur Altersbestimmung benutzt. Da das meiste Plastik aber aus Erdöl gewonnen wird, ist da kaum noch C14 drin. Das Erdöl stammt aus Lebewesen, die vor so langer Zeit gelebt haben, dass das C14 idR schon vollkommen zerfallen ist. Beim Wasserstoff wäre es das radioaktive Tritium, über das ich auch schon mal einen längeren Artikel geschrieben hatte.
    https://scienceblogs.de/nucular/2015/02/17/tritium-was-sie-schon-immer-wissen-wollten/
    Also da besteht idR auch keine Gefahr.

    Die gesundheitlich bedenkenswerten Sachen, sind dieselben, die es auch hier nach Tschernobyl gegeben hat und die teilweise heute noch mit Cs, I, Co, Ca oder ähnlichem beslastet sind. Also bayrische Wildschweine, Moos, Pilze und dergleichen. Da gibt es auch in Japan (also in der Provinz Sendai) belastete Lebensmittel, die dann (hoffentlich alle) aussortiert werden, bevor sie in die Nahrungskette gelangen. Das liegt aber vor allem daran, dass die Grenzwerte für Nahrung wesentlich niedriger sind, als für Gebrauchsgegenstände. Sprich was für eine Küchenfliese als Strahlung vollkommen OK ist würde als Nahrung aussortiert werden müssen.

    Sprich wen ein Plastikrucksack so radioaktiv ist, wie 10 Bananen, dann ist das immer noch vollkommen OK.

  110. #112 Renée
    22. Juli 2018

    Habe wie immer vielen Dank!

  111. #113 Anouk
    Stuttgart
    1. August 2018

    Hey,

    ein sehr interessanter und spannender Blog !!
    Also meine Frage:
    Ich möchte mir von einem Bekannten interessehalber einen Geigerzähler ausleihen.Dieser verleiht den selber sehr oft, auch an Leute die zB. Exkursionen nach Tschernobyl machen.
    Würde dieser anzeigen, wenn er mit radioaktivem Staub von dort kontaminiert wäre ?
    Es ist irgendwie eine blöde Frage schließlich ist es ein Strahlenmessgerät und dafür gedacht, aber ich guckte mir neulich ein Video über diese Geräte an und da erklärten zwei junge( ich nehme an Physiker ), dass die Strahlung nicht immer erfasst wird.

    Danke und alles Gute für Dich !!

    Liebe Grüße,

    Anouk

  112. #114 Tobias Cronert
    2. August 2018

    Danke!

    Das ist eine sehr gute und interessante Frage. Jeder Detektor hat einen Untergrund und wenn jetzt radioaktiver Staub an einem Detektor hängt, dann generiert das zusätzlichen Untergrund, der nicht von dem bauartbedingten elektronischen Untergrund unterschieden werden kann. Bei prof. Geräten ist das auch eine potentielle Fehlerquelle und daher wird vor einer Messung das Gerät immer mit einer Eichquelle überprüft.

    Auf der anderen Seite braucht man sich da bei normalen Geigerzählern eigentlich keine Gedanken zu machen. Denn die haben eine fest einprogrammierte Eichkurve (Umrechnung von Zählrate in Dosis), die unter normalen Umständen nicht verstellt wird. Das heißt wenn nun radioaktiver Staub an dem Detektor kleben würde, dann würde er eine recht hohe Dosis anzeigen, die über dem normalen Untergrund liegt, den man erwarten würde. Das heißt man würde recht schnell merken, dass etwas nicht ganz korrekt ist.

    Lange Rede kurzer Sinn: Ich denke du kannst dir den Geigerzähler ohne größere Bedenken ausleihen.

    beste Grüße
    Tobi

  113. #115 Celine
    Heidesheim
    2. August 2018

    Hi Tobi,

    mir sind noch ein paar Fragen eingefallen.

    Also ich habe ja auch überlegt wegen einem Geigerzähler wie du weißt..
    Ich habe einiges darüber recherchiert und bin dabei immer wieder auf Kommentare im Internet gestoßen, in denen gesagt wird: “dass Laien mit Geigerzählern nichts vernünftiges messen können und das sozusagen rausgeschmissenes Geld sei”
    Die Begründung ist soweit ich es verstanden habe, dass man ohne das Hintergrundwissen die schwächeren radioaktiven Quellen wie sie zB. von einer Granitfensterbank ausgehen nicht aus der Hintergrundstrahlung herauslesen kann.

    Das verstehe ich nicht. Wenn (bleiben wir beim Bps. Fensterbank), diese Strahlung abgibt, dann muss so ein Strahlenmessgerät diese doch einfach anzeigen ?

    Bzw. wenn die Strahlung so schwach ist, dass sie wirklich nicht von der Hintergrundstrahlung zu unterscheiden ist, dann entspricht sie doch dieser oder ist evtl. sogar niedriger also dann vom Strahlenschutzstandpunkt aus auch zu vernachlässigen und nicht schädlicher als das was man ohnehin abbgekommt ?

    Einige andere schreiben, man kann damit schon Fliesen oder Fensterbänke und dergleichen messen.
    Man soll dazu eine Langzeitmessung machen.

    Heißt das ich lege das Gerät dann einfach ab und lass es mehrere Stunden laufen und schaue mir dann den Durchschnittswert an und ziehe davon die Hintergrundstrahlung ab ? Weißt du wie ich es meine ?

    Das interessiert mich wirklich sehr, nicht nur weil ich sichergehen möchte, dass meine Wohnung ok ist..
    Es ist ein sehr faszinierendes Thema aber es ist nicht einfach, sich durch das Internet oder Bücher fundiertes Wissen darüber anzulesen, einfach weil es soviele verschiedene, teils Widersprüchliche Informationen dazu gibt.

    Deswegen vielen Dank, dass du uns hier weiterhilfst 🙂

    und ganz Liebe Grüße,

    Celine

  114. #116 Celine
    2. August 2018

    Jetzt fällt mir noch etwas ein..

    Du hast gesagt, man sollte sich einen Prüfstrahler besorgen, wenn man messen möchte, da man ohne Teststrahler im Alltag kaum etwas misst und dann in
    die statistische Schwankung des Untergrundes irgendwas hinein interpretiert , was da eigentlich gar nicht zu sehen ist.

    Das verstehe ich auch nicht. In der normalen Umgebung taucht ja nicht plötzlich ein Wert von sagen wir 2,0 µSv/h auf und ist dann eigentlich gar nicht da..oder doch ? :-/

  115. #117 Tobias Cronert
    3. August 2018

    Wow viele gute Fragen auf einmal, die auch eine lange Antwort brauchen.

    Also

    Wenn irgendwo eine Atombombe explodiert oder ein AKW im Stil von Tschernobyl oder Fukushima “in die Luft fliegt”, dann kann das auch ein absoluter Laie mit einem günstigen Zähler messen. Kein Problem.

    Bei einer Granitfernsterplatte unserscheiden, ob die nun “ein wenig” oder “sehr wenig” strahlt ist schon wesentlich komplizierter und dafür muss man sich gut mit Messmethode und Wirkungsweisen vertraut machen.

    Diese Frage ist auch normale und ich hatte das schon mal nach einer Leserfrage zusammengefasst. https://scienceblogs.de/nucular/2016/11/24/wieso-zeigt-mein-geigerzaehler-so-unterschiedliche-ergebnisse-leserfrage/

    Grundsätzlich ist das Problem, dass die Dosis (z.B. µSv/h) keine direkte Messgröße ist, sondern nur eine Näherung, die errechnet bzw. abgeschätzt wird. Die eigentliche Messgröße ist die Zählrate bzw. die Counts und die errechnete Dosis wird umso ungenauer, je niedriger die Zählrate. Vereinfacht kann man sagen, dass die Ungenauigkeit mit der Wurzel der Counts angegeben werden kann. Also wenn ich meine Dosis aus 100 Counts berechne ist mein Fehler +-10 Counts oder 10%. Wenn ich dagegen meine Dosis nur aus 9 Counts berechne, dann ist mein Fehler +-3 oder 33%. Das kann man durch Langzeitmessungen etwas kompensieren, wie das genau funktioniert hatte ich auch mal in dem Artikel mit dem Link oben beschrieben.

    Das heißt, dass ich z.B. die Radioaktivität einer Banane (ca. 4 CPM) nicht messen kann, weil (wenn) mein Untergrund eine Schwankung von +-10 CPM hat. Das ist das gleiche, wie mit einer Granitplatte. Da geht das Messergebnis einfach im Untergrund unter.

    Die gute Nachricht ist aber (wie immer): Hohe Strahlung kann einfach gemessen werden. Also alles, was richtig gefährlich wird kann man ohne Probleme messen und nachweisen. Je höher die Strahlung, desto einfacher zu messen, je niedriger die Strahlung, desto komplizierter wird es.

    Ein Prüfstrahler ist da mehr oder weniger an der Grenze. Der strahlt zwar genug, dass man ihn messen kann, aber immer noch so wenig, dass er aus Strahlenschutzsicht harmlos ist. Das heißt wenn man mit dem Prüfstrahler überprüft hat, ob der Detektor vernünftig funktioniert kann man mit fug und Recht behaupten, dass alles, was der Detektor nicht messen kann harmlos ist

    solange man es sich nicht in den Mund stopft.
    https://scienceblogs.de/nucular/2018/01/04/wie-kann-ich-radioaktivitaet-in-nahrung-messen/

    War das jetzt hilfreich? Wenn nicht, bitte weiterfragen. Ich (und eine Menge anderer “Profis” aus dem Internet) bin bei sowas manchmal schon ziemlich Betriebsblind.

  116. #118 Celine
    3. August 2018

    Hey,

    ja ich denke, dass sich die meisten Menschen eben dafür interessieren die Strahlung in ihren Wohnungen und Häusern zu messen, also die eher niederen Werte von Bedeutung sind.

    Wenn eine Atombombe explodiert, dann ist es vielleicht nett, wenn man das noch messen kann 😉 aber im Grunde ist dann ja eh alles zu spät und man kann nicht viel machen..

    Im Gegensatz dazu kann ich es ja schon ein wenig beeinflussen, welche Baumaterialien ich mir ins Haus hole..daher ist es interessanter diese so einigermaßen bestimmen zu können.

    Der GammaScout wirbt ja damit auch sehr niedere Werte anzuzeigen..ich glaube 0,01MicroSievert/h..
    Das ist doch schon eine sehr geringe Dosis und wenn dieser dann bei einer Fensterbank oder Fliese auch über längeren Zeitraum keinen Wert anzeigt, der über das Hintergrundrauschen hinausgeht kann man ja vielleicht etwas beruhigt sein ??

    Weisst du was ich probiere das einfach aus..

    Ich leihe mir den GammaScout aus und messe ein paar Sachen und dann wenn man praktische Werte hat kann man sich das ja nochmal angucken ..
    Ist dann vielleicht auch für andere Menschen einfacher zu verstehen wenn man ein konkretes Beispiel hat 🙂

    LG

  117. #119 Celine
    3. August 2018

    in ca einer Woche ist das Gerät da 😀

  118. #120 Tobias Cronert
    3. August 2018

    “einfach mal ausprobieren” ist auf jeden Fall eine tolle Idee und hat meine vollste Sympathie.

    Wenn der Gammascout aber damit wirbt vernünftig unter Alltagssituationen auf +- 0,01µSv/h genau messen zu können ist das eine glatte Lüge … oder eben so eine Werbemaßnahme die nur unter idealen Bedingungen im Labor mit geeichter Quelle etc. pp.

    Der Gammascount ist aber trotzdem ein gutes Gerät, nur der Vollständigkeit halber.

    Baumaterialien zu untersuchen ist eine sehr mühsame Sache, weil in der Realität nur die aller, allerwenigsten Sachen eine erhöhte Radioaktivität aufweisen. Es gibt ja genug Leute, die gezielt danach suchen und sich total freuen, wenn sie die tollen Italienischen Designerfliesen mit der Uranglasur finden.

    Immer dran denken alles unter Alles unter 3mSv/Jahr / 365Tage / 24h = 0,35µSv/h als Erwartungswert ist in Deutschland normal.

  119. #121 Doro
    9. September 2018

    Hallo Tobias,

    bei meinen Eltern im Wohnzimmer fand ich kürzlich eine Steinplatte vor (ca. 30x30x3 cm), von der ich vermute, dass sie aus Granit besteht. Sollte man sie aus dem Wohnzimmer verbannen?
    Im Muster sind auch ein paare buntere Stellen (überwiegen aber nicht).
    Eine Antwort von dir als Experten könnte vielleicht helfen.

    Viele Grüße
    Doro

  120. #122 Tobias Cronert
    10. September 2018

    Nein, die eine Steinplatte kannst du getrost dalassen. Die Radioaktivität wird idR nur dann relevant, wenn wirklich eine ganze Wand oder so damit gefliest ist.

  121. #123 Doro
    11. September 2018

    Danke!
    Die Platte verliert, glaub ich, ab und zu einige Partikel (ist wohl ein bisschen porös). Ist sie vor diesem Hintergrund immer noch harmlos?

    Doro

  122. #124 Doro
    11. September 2018

    Ach ja: Meine Eltern kauften sich kürzlich auch einen neuen DVD-Rekorder von Panasonic (japanische Firma).
    Könnte es diesbezüglich Bedenken geben (hinsichtlich Japan-Fukushima)??
    Danke im Vorfeld!

  123. #125 tomtoo
    27. September 2018

    Hey was mach eigentlich mein Lieblingsneutronenphysiker? Bist sehr still im Moment. Hoffe es geht dir nicht allzu mies?

  124. #126 Frantischek
    29. September 2018

    Da will ich mich anschließen.
    Meld dich doch wieder einmal!

    Weiterhin alles Gute, ich freu mich schon auf gute Nachrichten!

  125. #127 Tobias Cronert
    30. September 2018

    Hi Leute,

    die letzten Wochen waren recht hart und außgrund der Medikamenten Nebenwirkungen zittern meine Hände recht viel, so dass das Tippen problematisch ist. Aber das Gröbste ist jetzt erst mal vorbei und ich melde mich bestimmt in den nächsten Wochen wieder.

    @Doro: Wenn etwas abblättert von der Granitplatte ist das natürlich nciht so toll. Ich persönlich würde mir zwar weiterhin keine Sorgen machen, aber das ist schon so ein Grenzfall, wo ich es verstehen kann, wenn jemand das nicht im Wohnzimmer haben möchte. Der DVD-Rekorder ist in jedem Fall ungefährlich, das ist kein Problem.

  126. #128 Doro
    30. September 2018

    Lieber Tobias, ich wünsche dir zuallererst gute Besserung!
    Und vielen Dank auch für deine Nachricht!
    Ich habe mich zwischenzeitlich auch gefragt, ob die Platte doch aus Marmor besteht, sicher bin ich mir aber nicht. Marmor ist jedoch harmloser als Granit, oder? Falls es doch der (schlimmere) Granit ist, frage ich mich natürlich, ob verschleppte Granit-Partikel (durch Abbröckeln) gefährlich werden können? Was können die eigentlich schlimmstenfalls anrichten (falls sie irgendwo landen wie z.B. an Alltagsgegenstände usw.)?

    Dann noch eine andere Frage: in der Küche verwendete (hellblaue) Fliesen an der Wand (das Haus ist ziemlich alt) lassen mich fragen, ob die auch strahlend sein könnten. Allerdings ist in dieser Fliesen-Wand auch der Lichtschalter integriert. Und wenn die Fliesen strahlen würden, dann würden sie doch im Laufe der Zeit den Lichtschalter beschädigen? Da der Lichtschalter jedoch tadellos funktioniert, könnte man doch daraus folgern, dass die Fliesen eben nicht strahlen?

    Ich danke dir vorab vielmals!

  127. #129 tomtoo
    30. September 2018

    @Tobias
    Sau gute Nachricht! Das freut mich!

  128. #130 tomtoo
    30. September 2018

    @Doro
    Mit dem Lichtschalter hat das nix zu tun. Das ist so ein grobes Bauteil, den würde Strahlung nicht so schnell klein bekommen.

  129. #131 tomtoo
    30. September 2018

    @Doro
    Das soll natürlich nicht heissen die strahlen oder nicht. Sondern wirklich nur an einem Lichtschalter kann man sowas nicht feststellen.

  130. #132 zimtspinne
    1. Oktober 2018

    @ Tobias
    argh, hast du es etwa versäumt, dir einen Schreiberling/Tippse(r) zu organisieren? Eine old school-Sekräterin quasi?

    Ich bin gerade bisschen “gespannt” zu wissen, ob du deine Stammzelltransfusion schon hattest oder steht das noch bevor?
    Ab dann werden ja Tage gezählt, 100 glaube ich, bis das Gröbste vorbei ist… da muss deine Gemeinde doch wissen, dass Daumen gedrückt werden … (jawohl).
    Ich wünsche dir alles Gute und bleib so unverwüstlich wie du bist 🙂

  131. #133 zimtspinne
    1. Oktober 2018

    *Ergänzung
    bitte das “gespannt” nicht missverstehen, aber ich weiß von meinen Verwandten, dass die Infusion an sich total unspektukalär und …eigentlich ziemlich öde ist…. kein dramatischer Knall oder so. und auch keine dramatischen Nebenwirkugen wie bei Chemotherapie.

  132. #134 Tobias Cronert
    1. Oktober 2018

    @Doro: Wie tomtoo schon gesagt hat braucht es sehr sehr viel Strahlung um einen Lichtschalter zu Beschädigen. Eine, für einene Menschen tödliche, Dosis juckt den Lichtschalter nicht die Bohne. Wenn das normale Fliesen sind brauchst du dir keine Gedanken zu machen. Problematisch sind maximal irgendwelche Natursteinfliesen oder so. Als Faustformel gilt immer je älter und natürlicher das Gestein, desto potentiell radioaktiver. An gefährlisten ist es, wie immer, natürlich, wenn man die Partikel in den Körper ausfnimmt (einatmet oder isst), wenn sie einfach nur herumliegen sind sie rel. harmlos.

    Ansonsten: Nein, eine Sekretärin habe ich leider nicht und selbst wenn war das Sprechen eine zeitlang auch nicht so dolle *g*. Mittlerweile bin ich aber eigentlich wieder auf dem Dampfer und nur noch dabei liegengebliebene Arbeit nachzuholen. Die nächsten Artikel kommen sicher in den kommendenTagen und Wochen … ein bischen Gedult noch.

  133. #135 Doro
    2. Oktober 2018

    Vielen Dank, Tobias!
    Ein weiteres Dankeschön auch an tomtoo.

    Das ist ja ein wirklich sehr interessantes Themengebiet. Ich studiere ja in einer völlig anderen Richtung, aber ich muss wirklich sagen, dass diese Wissenschaft es wirklich in sich hat!

    An meine letzten Fragen anknüpfend fallen mir noch folgende ein (um den Sack zuzumachen 😉
    -Granitpartikel: hier greift doch bestimmt auch (wie auch bei Keimen) das Dosis-Prinzip (nur viele derartiger Partikel sind gefährlich)? Wieviele müssten es denn eigentlich sein, bis sie zur echten Gefahr werden würden?
    -Fliesen: falls die Fliesen eine Uranglasur besitzen würden, würde dann auch Drüberwischen (mit einem feuchten Küchenlumpen) winzige Uran-Teilchen absorbieren) Wäre dann also der Lumpen auch eine radioaktive Quelle? Und wenn ja, wäre diese Quelle hinsichtlich ihrer Dosis gefährlich?

    Wenn du hierzu noch Bescheid wüsstest, wäre ich sehr dankbar!
    Liebe Grüße und weiterhin gute Besserung!

  134. #136 Tobias Cronert
    2. Oktober 2018

    -Granitpartikel: hier greift doch bestimmt auch (wie auch bei Keimen) das Dosis-Prinzip (nur viele derartiger Partikel sind gefährlich)? Wieviele müssten es denn eigentlich sein, bis sie zur echten Gefahr werden würden?

    Ja, grundsätzlich stimmt das. Um zu bestimmen, wann sie zur Gefahr werden muss man die Anzahl Atome des Radioaktiven Elementes zählen. Da aber jetzt in einem Staubkorn sehr sehr sehr viele Atome drin sind macht das von einem praktischen Standpunkt aus keinen Sinn und man guckt sich wieder nur die Dosis an.

    Bei einer Uranglausur würden auch ganz ganz ganz wenige Atome abreiben beim Drüberwischen, aber das wären so wenige, dass der Lappen selber wahrscheinlich mehr natürliche Radioaktivität hat, als er von der Fliese aufnehmen würde … und alles ist so wenig, dass es harmlos ist.

  135. #137 Doro
    2. Oktober 2018

    Und aber welcher Dosis (an Granitparkeln) wäre es dann bedenklich?

    Vorab herzlichen Dank und gute Nacht! 🙂

  136. #138 wereatheist
    3. Oktober 2018

    Ich freue mich, dass es bisher wohl ganz gut für Dich gelaufen ist, Tobias (man macht sich ja Sorgen…).
    @Doro: Das hängt doch davon ab, wie hoch der Gehalt an U/Th im Granit ist.
    Einer meiner Ururgroßväter war Steinmetz.
    Er starb, wie es sich so gehört, an Silikose (aka ‘Staublunge’).
    Falls Du Bildhauerin wärest, solltest Du immer eine Staubmaske tragen, nicht wegen der Radioaktivität in den Gesteinen, sondern wegen der Silikose-Gefahr.

  137. #139 Tobias Cronert
    11. Oktober 2018

    Dosis an Grannitpartikeln … ja so um die 1mSv an Dosis wäre schon eher Schlecht.

    Das kann man aber nicht in “Staubkörner” umrechnen. Im Extremfall könnte man sehr viel Radioaktivität (eine mehr als tödliche Dosis) in weniger als einem Sandkorn unterbringen.

    Aber das gibt es bei natürlichem Granit nicht. Bei natürlichem Granit brauchst du sehr sehr viel, bevor es gesundheitsschädlich wird.

  138. #140 Doro
    11. Oktober 2018

    Lieben Dank!

  139. #141 Doro
    12. Oktober 2018

    Können vom eigenen Garten geerntete Früchte (Trauben, Äpfel etc.) zum Verzehr (rein theoretisch) noch Spuren von der Tschernobyl-Katastrophe in sich bergen? Oder ist dies abwegig?
    Wenn du hierzu noch etwas wüsstest, lieber Tobias, wäre ich sehr dankbar – vielen Dank!

  140. #142 Tobias Cronert
    12. Oktober 2018

    Ja, alle Pflanzen und Gartenerde enthält auch heute noch Spuren der Radioaktivität von Tschernobyl … wie gesagt, man kann Radioaktivität sehr sehr sehr präzise messen.

    Aber das wird in Deutschland immer noch regelmäßig kontrolliert und ist so gering, dass es für den Menschen auch zum Verzehr harmlos ist.

    … das gilt mit ein paar kleinen Ausnahmen. Pilze, Moos, Rehe und Wildschweine aus den bayrischen Wäldern können noch Radioaktivität über den Grenzwerten ausweisen. Davor wird dann aber auch in der Regel vom Bundesumweltamt in den entsprechenden Gebieten gewarnt.

  141. #143 Doro
    13. Oktober 2018

    Das heißt, wenn mein Garten nicht in Bayern ist (was der Fall ist), dann ist alles harmlos?
    Danke nochmal!

  142. #144 Tobias Cronert
    19. Oktober 2018

    Neben Bayern gibt es auch noch ein paar andere Gegenden von Deutschland, die belastet sind, aber generell …. Ja!

  143. #145 Doro
    21. Oktober 2018

    Großen Dank!

  144. #146 Bene
    Bendorf bei Koblenz
    12. November 2018

    Hallo Tobias!
    Über Facebook bin ich auf dein Tagebuch über deinen Kampf mit der Leukämie gestoßen. Ich wünsche dir alles alles Gute dabei, wieder richtige Stammzellen produzieren zu können und drücke dir ganz kollegial die Daumen.
    Jedenfalls halb kollegial: Ich bin Physiklehrer am Gymnasium und hätte eine blöde Bitte, vor allem vor dem Hintergrund deines anstehenden Kampfs mit den Folgen der Strahlentherapie. Die Dosis ist ja echt heftig. Im Strahlenschutzkurs wird vor sowas immer gewarnt und man liest ja die krassesten Geschichten und sieht die erschreckendsten Bilder von (sehr) hohen Strahlenbelastungen.

    Hier meine Bitte:
    Ich würde gerne mit meinem Abiturjahrgang in Physik zum Thema Strahlenschutz mal was anderes machen: Deinen Blog lesen (und nebenbei Veritasium schauen[incl. seiner Doku]). Wäre es für dich möglich, mir die Texte, falls du sie hast und es dir keine allzu große Mühe bereitet, als Rohfassung zukommen zu lassen, so dass ich sie nicht alle von Werbung bereinigen muss und handlich formatieren muss?
    Darf ich die überhaupt nutzen?
    Im Anschluss wollen wir vielleicht auch ins Bundeswehrzentralkrankenhaus nach Koblenz, die haben da einen Monster-Ganzkörperscanner.

    Ich fühl mich echt schlecht dich jetzt darum zu bitten, aber zumindest ein Einverständnis zur Nutzung wäre mir sehr lieb.

    Vielen Dank und teu teu teu,
    Bene

  145. #147 Tobias Cronert
    12. November 2018

    Klar kannst du die Texte gerne benutzen. Ich fühle mich sehr geehrt und für alle Sachen in der Ausbildung mit Physik bin ich eh immer zu haben.

    Als “Rohfassung” kann ich dir leider nur das XML-wordpress Format anbieten. Das kann man sich dann entweder als HTML angucken oder mit einem anderen wordpress Format importieren.
    Für alles andere müsste ich auch copy&pasten und da habe ich ehrlich gesagt gerade nicht den Nerv zu.

  146. #148 Bene
    13. November 2018

    Sehr cool, vielen Dank fürs Einverständnis 🙂
    Ich werde mir die Tete dann selbst zusammenbasteln aus dem Blog, das ist denke ich am einfachsten und ich habe die Möglichkeit gewisse Dinge zu kürzen oder an manchen Stellen Kommentare einzufügen.

  147. #149 Tobias Cronert
    14. November 2018

    OK, praktische Office oder .pdf Dokuente habe ich leider nicht von den Artikeln. Sorry

  148. #150 Till
    18. Januar 2019

    Ich habe eben diesen SPON Artikel zu Radionuklidbatterien für Herzschrittmacher gelesen. Das klingt ja auf den ersten Blick nicht nach einer guten Idee. Das Kredo lautet doch sonst Radioaktives Material wenn irgend möglich aus dem Körper fernzuhalten.

    In dem Artikel steht nun: “Zum Abschirmen dieser Strahlen reiche “schon eine einfache Plastikverpackung” [da es sich um einen Betastrahler handelt]”. Mir ist bewusst, dass man Betastrahler für den sicheren Gebrauch ausserhalb des Körpers sehr einfach abschirmen kann. Wenn ich so ein Ding aber 50 Jahre im Körper habe, dann stellt das doch ganz andere Anforderungen an die Abschirmung.

    Hier also meine eigentliche Frage:
    Aus meiner Sicht müsste die Abschirmung sicher stellen, dass die Strahlung die es durch die Abschirmung schafft nicht höher ist als die normale Strahlung im Gewebe des Herzbeutels. Kannst Du einschätzen ob das mit der “normalen Plastikverpackung” unter diesen Bedingungen stimmt oder ob man doch 10 mm Titan braucht? Oder noch einfacher: Denkst Du, dass solche Radionuklidbatterien für den Medizinischen Gebrauch prinzipiell machbar sind?

    Das wäre schon ziemlich cool – erinnert mich irgendwie an Iron Man. Ich hätte dann auch gleich eine passende Idee für ein Tattoo auf der Brust.

  149. #151 Tobias Cronert
    20. Januar 2019

    Diese Batterien sind eine super tolle Idee , vorallem, wenn sie praktikabel werden.

    Ni 63 ist ein reiner Beta Strahler mit 17keV das ist nahezu gar nichts. Es ist zwar nicht ganz so wenig, wie der total weich Betastrahler Tritum mit 5keV, aber “normale Betastrahlung” schlägt immer mit ein paar hundert keV zu Buche.

    Erfahrungsgemäß kann ich 17keV locker mit 0,1mm Alufolie abschirmen. Dabei ist es ziemlich egal ob es 1 oder 50 Jahre lang abgeschirmt werden muss. Solange man den Behälter nicht aufbricht, sollte das keine Probleme geben … so wie beim Tritium *g*

    also auf zum eigenen arc reactor

  150. #152 UMa
    22. Januar 2019

    @Tobias:
    Das mit dem Ni63 klingt interessant, auch weil es keine radioaktiven Folgeprodukte gibt.
    Früher gab es Herzschrittmacher auch mit Plutonium.
    https://de.wikipedia.org/wiki/Herzschrittmacher#Plutonium-Herzschrittmacher
    https://web.archive.org/web/20150913030541/https://www.bfs.de/DE/themen/ion/anwendung-medizin/mehr-infos/herzschrittmacher.html
    Dabei sollen lokal nach dem 2. Link bis zu 50µSv/h Strahlendosis aufgetreten sein. Die Alphastrahlung sollte sich trotz über 5 MeV aber eigentlich besser abschirmen lassen, oder? Welche Strahlung ist dann für diese Dosis verantwortlich (Gamma?) und wie sieht das in größerer Entfernung im Körper aus?

  151. #153 Tobias Cronert
    22. Januar 2019

    Das Alpha-Teilchen lässt sich ziemlich leicht aufhalten, aber wenn es in der Abschirmung landet, dass löst es dort Ereignisse aus.

    Das Beta Elektron wird von der Abschirmung einfach nur ohne Rückstände absorbiert, aber das Alpha Teilchen löst beim Absorbtionsprozess Gammas aus und/oder aktiviert sogar noch Dinge … daher kommt dann die Dosis.

  152. #154 UMa
    5. Februar 2019

    @Tobias: Aha, danke. Die Aktivierung geht also über den Umweg gamma.

    Ich hoffe, Du bist weiterhin auf dem Wege der Besserung. Weiter alle guten Wünsche von mir.

    Ich habe noch eine weitere Frage. Es geht um die Rohstoffvorräte für die Kernfusion.

    Bei möglichen Kernfusionsreaktoren die D+T verbrennen, soll das T aus Li6 mittels Neutronen erzeugt werden. Da Neutronen verloren gehen, wird wohl Be9 oder ein Bleiisotop als Neutronenverdoppler eingesetzt werden sollen.
    Meine Frage: Wie viel Be9, relativ zum Li6 würde denn benötigt?
    Dazu finde ich im Netz keine vernünftigen Aussagen. Von sehr wenig bis zu so viel, dass nicht das Lithium sondern Beryllium der knappste Rohstoff für die Tritium Erzeugung ist habe ich schon gelesen, nur keine konkreten Mengen.
    Und warum nimmt man kein Li7, da dann das Neutron ja wieder frei wird?

  153. #155 Tobias Cronert
    5. Februar 2019

    Sowohl Be als auch T sind jetzt zwar nicht die billigsten Dinge, aber nicht sooo schwer zu bekommen, wenn man ein Budget hat. 200l Be kosten ca. 2Mio € … ist halt die Frage, wie groß du Bauen musst.

    Eine Atombombe kommt mit 30L Be aus. Bei den Reaktoren ist auf der Innenseite der Kacheln ein paar mm Be. Keine Ahnung, wieviel Masse oder Volumen (ca. 2kg/L) das so sind. Aber es gibt genug Be aus alten Kernreaktoren, dass immernoch so strahlt, das es nirgendwo anders, als wieder in einem Reaktor eingesetzt werden kann. Das kriegt man quasi kostenlos *g*

  154. #156 Tobias Cronert
    5. Februar 2019

    Ach ja, man benutzt Be, weil die Neutronenenergienen aus dem Be im MeV bereich sind und die im Li in 100keV Bereich. Dadurch sind die Be neutronen wesentlich effektiver, weiter zu reagieren.

  155. #157 nihil jie
    24. April 2019

    Fragen habe ich schon. Beispielsweise würde mich interessieren wie Du zu den Bemühungen um den Bau von Kernfusionsreaktoren stehst. Lohnt sich deiner Meinung nach die Mühe so ein Reaktor in Betrieb zu nehmen ? Oder werden wir das vorerst nicht bewältigen können ? Interessant wäre auch die Frage bezüglich eines Thoriumreaktors.
    Diese Themen interessieren mich auch ziemlich stark. Deswegen auch meine Fragen zu dem Thema.

    Meine persönliche Haltung zu den Themen ist schon noch recht optimistisch. Noch. Habe durchaus einige Hoffnungen bezüglich dieser Bemühungen.

  156. #158 Tobias Cronert
    24. April 2019

    Also die Fuionsleute haben chon lange viel Hoffnungen gemaht, aber untrm Strich ist nicht viel bei herausgekommen. Irgendwann mauss man sich eigenstehen, dass man ein totes Pferd reitet. Sobald dannneue Konzepte, wie Laserfusion oder Pyrofusion um die Ecke kommen, weil es auch entsprechenden anderen Gebieten urchbrüche gamcht hat. Dann kann man gerne noch mal diealten Ideen aus dem Keller holen, aber bis dahin?
    Es gibt ja noch jede Menge andere Kernreaktoren. Dual-Fluid, unterkritisch Beschleunigerbetriebener HTR oder eben Thorium. Das man da nicht wirklich forscht halte ich für grob Fahrlässig und kreide ich der Atomkraftwerkslobby an … das haben die echt verbockt.

  157. #159 nihil jie
    25. April 2019

    @Tobias Cronert

    Danke für die Antwort.

    “Irgendwann mauss man sich eigenstehen, dass man ein totes Pferd reitet.”

    Dem stimme ich uneingeschränkt zu. Manchmal fällt einem aber schon etwas schwer sich ein zu gestehen dass man ein totes Pferd reitet *g 😉

    “Es gibt ja noch jede Menge andere Kernreaktoren. Dual-Fluid, unterkritisch Beschleunigerbetriebener HTR oder eben Thorium. Das man da nicht wirklich forscht halte ich für grob Fahrlässig und kreide ich der Atomkraftwerkslobby an”

    Ja, da sehe ich auch ein Problem. Fahrlässig ? Das kann ich eben nicht so gut beurteilen. Da fehlen mir dafür die nötigen Infos. Diese Möglichkeiten auszuklammern halte ich aber auch für ziemlich “unglücklich”

  158. #160 Miri
    Eningen
    27. Mai 2019

    Hallo,

    ich habe auch eine Frage..Neulich las ich zufällig, dass Gartendüngergranulat oft mit Uran beastet sei.
    Der Garten unserer Wohnung wurde von einer Gartenbaufirma mit Blaukorn gedüngt. Jetzt stelle ich mir die Frage ob dies gefährlich ist, da ich auch Kinder habe die im Garten spielen und falls sich darin Uran befindet, strahlt das dann von meiner Terasse ins Innere meiner Wohnung oder halten die Wände das ab ?
    Ein Teil des Rasens befindet sich direkt vor meinem Wohnzimmer, also eigentlich nicht weit entfernt.
    Bzw. wie ist denn da die Reichweite ? Uran ist doch ein Alpha-Strahler, aber wie sieht es mit den Zerfallsprodukten aus ?

    Über eine Antwort freue ich mich und vielen Dank,

    LG Miri

  159. #161 rolak
    27. Mai 2019

    moin Miri, sämtliche Mineraldünger aus Sedimentgestein sind unweigerlich mit Uran belastet – weil der UranAnteil an der Erdkruste im Mittel 3.2ppm beträgt. Doch das gilt selbstverständlich ebenfalls für den Boden um Deine Wohnung herum, bereits ohne Dünger.

    Mal abgesehen davon, daß in diesen Dosen die Giftigkeit des Urans wesentlich relevanter sein dürfte als seine Radioaktivität, ist beides völlig vernachlässigbar gegenüber der Giftigkeit des Nitrates, das einen Teil der Funktionalität des NPK-Düngers ausmacht. Solange die farblich so interessanten Körner obenauf liegen, sollten daher weder naschfreudige Kinder noch pfotenleckende Haustiere auf den Rasen.

  160. #162 Tobias Cronert
    27. Mai 2019

    Ja, grundsätzlich, was rolak schon gesagt hat. Die Gifttigkeit des Urans, als Schwermetall (also wie Blei) ist beim Düngen im Garten etc. gefährlicher, als die Radioaktivität. Aber der Dünger wird halt auch nicht gefährlicher sein, als Erde aus dem Erzgebirge oder den Alpen.

    Die Radioaktivität ist zwwar auch Alpha Strahlung, aber die ist nur gefährlich, wenn sie in den Körper aufgenommen wird. Trotzdem gibt es halt durch Zerfallsprodukte etc. genug Gamma-Strahlung und die hat grundsätzlich eine große Reichweite. Aber selbst bei der Gamma-Strahlung braucht man sich bei ein paar Metern und einer Hauswand dazwischen keine Gedanken mehr zu machen.

  161. #163 Miri
    Eningen
    28. Mai 2019

    Danke für eure Antworten.

    Jetzt bin ich aber trotzdem noch etwas mehr beunruhigt.
    So viele Meter Abstand sind das nämlich nicht.
    Ein Teil des Rasens ist direkt vor der Hauswand an der auch mein Sofa steht. Also das sind keine zwei Meter :-/
    Könnte man das jetzt mit einem Geigerzähler nachmessen ob da überhaupt Strahlung vorhanden ist ?
    Danke euch und LG
    Miri

  162. #164 Miri
    Eningen
    28. Mai 2019

    Wenn man sich die Zerfallsreihe von Uran anschaut, also Uran-Radon-Reihe sind da aber nur Alpha-und Betra-Strahende Zerfallsprodukte aufgeführt oder gibt es noch andere Reihen ? Ich finde auf die Schnelle nur diese eine und ich kenne mich ja auch so gar nicht aus 🙁

  163. #165 Tobias Cronert
    28. Mai 2019

    Uran hört sich immer böse an, weil man es eben von Atomkraftwerken und Atombomben kennt. Aber die Radioaktivität von Uran ist recht harmlos, solange man eben den Kern nicht spaltet. Uran ist ja ein ganz normales Mineral, das in der Natur überall da vorkommt, wo es auch Granit gibt. Denk immer an die Fausregel je höher die Halbwertszeit, desto geringer ist die Strahlung eines Isotopes und Uran hat halt eine riesig lange Halbwertszeit (die Sache mit dem Atommüll) und daher eben auch nur recht wenig Strahlung. Für das kleine bischen Strahlung reicht ein Meter und/oder eine Wand (oder eine Coach *g*) locker aus.

    Natürlich kann man das nachmessen, aber ich gehe mal stark davon aus, dass der Geigerzähler eben nichts anzeigen wird, weil es einfach zu wenig ist um mit einem handelsüblichen Gerät gemessen zu werden. Aber da ich immer ein großer Fan von Nachmessen bin und du die 60-100 Euro für ein Gerät aus dem Internet übrig hast. …
    Wie schwer es ist Uran zu messen, habe ich hier mal beschrieben https://scienceblogs.de/nucular/2017/09/12/wie-spuere-ich-uranmunition-mit-einem-geigerzaehler-von-amazon-auf-eine-anleitung/

    Ansonsten bzgl. Zerfallsreihe gibt es halt überall, wo es Alpha-Strahlung gibt auch Aktivierung und damit eine ganze Reihe von Isotopen, die man nur vom Angucken der Zerfallsreihe nicht direkt auf dem Schirm hat. Aber das ist dann irgendwann echt nur noch akademisch, denn die Mengen sind soooooo furchtbar gering, dass sie kaum noch messbar sind. Selbst mit absolutem Profi-Equipment.

  164. #166 Miri
    28. Mai 2019

    Also kurz gesagt, ich muß mir da jetzt keine Sorgen machen 🙂 Danke !! Du hast mir echt geholfen und du hast einen neuen Fan für deinen Blog 😉

    Aber neugierig bin ich jetzt irgendwie schon.
    Man kann sich ja auch Strahlenmeßgeräte ausleihen, z.B. den Gammascout, der ja damit wirbt angeblich auch Alphastrahlung erfassen zu können.
    Aber wenn ich messe, dann interessiert mich eh nur die Gammastrahlung.

  165. #167 Tobias Cronert
    28. Mai 2019

    Gerne und Danke für die netten Worte.

    Bevor du dir einen Gammascout ausleihst guck dir am besten erst mal die YouTube Videos dazu an. Da gibt es viele Hobbyiisten, die viel ins Netz gestellt haben.

  166. #168 Miri
    28. Mai 2019

    Ja das mache ich, danke..

    Ich hab hier noch etwas interessantes zu dem Thema gefunden.
    https://www.opengeiger.de/RadonDuenger/RadonDuenger.html

    Leider sagen mir die Zahlen der Exhalations-Messungen nichts..heißt das die Dünger wären alle-bis auf den einen, der als “nicht empfehlenswert”eingestuft wird ok ?

    Falls das Thema nervt bitte sagen 😉

    LG

  167. #169 Tobias Cronert
    29. Mai 2019

    Ja, Radon ist auch immer ein durchaus sinnvolles Thema bei allem, was mit Mineralien, Kellern, Bohrungen etc. zu tun hat.

    Aber das ist immer nur in geschlossenen Räumen, wie in einem Keller z.B. relevant, wo die Luft steht und nicht ausgetauscht wird. Draußen auf dem Rasen ist das Radon-Gas direkt weg und absolut kein Problem.

    Wird halt relevant, wenn man einen Sack Dünger in der Garage lagert.

  168. #170 Roland
    30. Mai 2019

    Bekannte von mir wollen ein Haus kaufen und setzen sich zur Zeit daher auch mit dem Thema Radon auseinander.
    Das ist schon auch erschreckend, denn das ist ja praktisch überall und wenn man Pech hat auch in der eigenen Wohnung in höheren Mengen.
    Zerfällt Radon dann auch in Isotope die ɣ-Strahlung aussenden ?. Dann ist man dieser Art Strahlung ja eh immer augesetzt und man kann halt eh nichts dagege machen.Dann muss man sich auch wegen Dünger keine Gedanken machen.

  169. #171 rolak
    30. Mai 2019

    Radon → <Zerfallsprodukt> → ɣ ?

    moin Roland, falls ich es komplett überblicke, entstehen in der ersten Stufe ausschließlich (At, Po, Fr) alpha- und betaStrahler. Das ist aber in diesem Falle (insbesondere bei Polonium) wesentlich riskanter, weil Radon als Gas eingeatmet werden kann und dann der effektive Abstand auf Null schrumpft.

    Dann .. immer augesetzt

    Sowieso, dürfte doch im Normalfall der größte Teil der ɣStrahlung ursächlich von der kosmischen Strahlung kommen (→Belastung durch Flüge).

  170. #172 Tobias Cronert
    30. Mai 2019

    Naja Radon ist erst mal ein ganz normaler Begleiter aus dem Alltag. Da kommt der menschliche Körper auch durchaus gut mit klar. Das wird erst ein Problem, wenn es sich eben in Kellerräumen oder so sammelt.

    Gamma ist dabei aber das kleinste Problem, weil man es eben einatmet. Dadurch kommt die Alpha Strahlung nah genug ran um Schaden anrichten zu können und die ist halt eben eine Kanonenkuegl in der Zelle im Vergleich zu der 9mm Photonenstrahlung oder den elektronen, die irgendwo dazwischen liegen.

  171. #173 Peter
    12. Juni 2019

    Hallo Tobias,

    vorab vielen Dank für Dein Engagement in diesem informativen Blog!
    Zu meiner Frage: Granit ist doch ein Alpha-Strahler und als solcher vergleichsweise harmlos? Wieviel cm Reichweite hat diese Strahlung von einer Granit-Wand? Durchdringt Alpha-Strahlung die Haut? Würdest du vor einer Granit-Wand schlafen?
    (Gegen die Radon-Belastung reicht ja Lüften, oder?)

    Und: wenn auf meinem Fahrradweg quasi daneben eine Entsorgungsfirma für Abfall, Metalle und Schrott befindet, stellt dies ein Risiko dar aus radioaktiver Sicht? Würdest du den Fahrradweg wechseln?

    Danke!
    Peter

  172. #174 Tobias Cronert
    13. Juni 2019

    Hallo Peter,

    also da wo natürliches Granit-Gestein ist, ist auch meist natürliches Uran-Gestein. Da dieses schon lange in der Erde ist/war sind auch die ganzen Zerfallsprodukte in der Regel mit dabei. Es gibt Alpha Strahlung, aber auch immer noch ein bischen Beta und Gamma. Die Alpha Strahlung kommt an Luft keinen Zentimeter weit und auch nicht durch die menschliche Haut durch. Das ist eigentlich nur gefährlich, wenn man es verschluckt und in den Körper aufnimmt. Gegen Radon reicht einmal die Woche Lüften vollkommen.

    Vor einer Granit-Wand schlafen? Tja, also ein paar Tage oder ein-zwei Wochen pro Jahr … kein Problem. Das ganze Leben lang? Das würde ich mir schon überlegen. Weil wenn man pro Tag 8 Stunden im Bett verbringt, dann ist das ja der Ort, wo man die meiste Zeit seines Lebens verbringt. Wenn jetzt ausgerechnet da eine (auch nur leicht) erhöhte Strahlung zu finden ist … da kann man schon überlegen, ob man das nicht irgendwie anders machen kann.

    Auf der anderen Seite gibt es halt Granit, wo überhaupt kein Radioaktives Zeug drin ist und es gibt Granit, der schon ordentlich in der Gegend herumstrahlt. Ein Geologe kann dir durch angucken vielleicht schon sagen welcher welcher ist, aber ich müsste das immer nachmessen, weil ich von natürlichem Gestein annähernd keine Ahnung habe.

    Der Fahrradweg ist mMn egal. Da bist du ja innerhalb von ein paar Sekunden dran vorbei geradelt. Wenn der Schrottplatz jetzt neben deinem Schlafzimmer wäre, dann würde ich etwas anderes empfehlen, aber nur ein paar Sekunden auf dem Radweg sind kein Problem.

    Beste Grüße
    Tobi

  173. #175 Peter
    14. Juni 2019

    Hallo Tobias,

    zunächst vielen Dank für deine ausführliche Antwort!
    Das mit dem “bisschen Beta- und Gamma-Strahlung” war auf jeden Fall neu für mich. Wie gefährlich ist dieses “Bisschen” und: wie viele cm/m Reichweite haben sie? Ist Beta und Gamma bei Granit hinsichtlich der Dosierung vernachlässigbar oder eben nicht?
    Wenn du hierauf noch Bescheid wüsstest, wäre super!

    Danke,
    Peter

  174. #176 Tobias Cronert
    15. Juni 2019

    Hallo Peter. Beta und Gamma ist bei Granit ein Faktor und nicht vernachlässigbar. Beta hat eine Reichweite von cm und Gamma von ca. einem Meter (also in diesem Fall, theoretisch ist die Reichweite unbegrenzt nimmt aber stark mit dem Abstand (Distanz^2) ab).

    In der Regel ist das aber harmlos. 95% aller Granitanwendungen sind bzgl. Radioaktivität völlig irrelevant und von den restlichen 5% ist nur 0.01% wirklich gesundheitsgefährlich und das auch nur, wenn man sich lange und oft in der Nähe aufhält.

    Als Faustregel denk immer daran, dass es natürliches Gestein ist und der menschliche Körper ein paar Millionen Jahre Zeit hatte sich auf die Radioaktivität von diesem Gestein einzustellen und anzupassen. Ja, es gibt es paar Ausnahmen, aber die sind sehr selten und man kann sie leicht nachmessen.

  175. #177 Peter
    15. Juni 2019

    Hallo Tobias,
    erneut besten Dank für deine ausführliche Antwort!
    Kurze Nachfrage: fehlt vor dem “cm” in puncto Beta-Strahlung eine Zahl? Oder lässt sich dies nicht allzu genau einkreisen? Und stimmt es, dass Gamma-Strahlung nicht allzusehr gefährlich ist, wenn sie in den Körper tritt? Beta wäre dann viel gefährlicher?
    Sodann: ist es nicht auch so, dass Strahlung (in einem bestimmten Dosen-Verhältnis) nicht auch förderlich für den Körper sein kann? Das Wachstum von Pflanzen und Insekten kann unter Umständen profitieren? In der Therapie wird Strahlung auch eingesetzt… Ich meine, auch mal gelesen zu haben, dass es tatsächlich Menschen gibt, die im Vergleich zu anderen kaum bzw. sehr viel schwieriger Probleme durch Strahlung bekommen… aber wie genau erfortscht das alles ist, weiß ich natürlich nicht.
    Oder aber ist es ausnahmslos so, dass Strahlung eigentlich immer schädlich ist?
    Zum Schluss noch die Frage, dass das Nachmessen interassant klingt. Aber verhält es sich nicht so, dass ich als Laie diesbezüglich absolut überdordert wäre (mit der Handhabhung eines geeeigneten Geigerzählers)?

    Nochmals danke,
    Peter

  176. #178 Tobias Cronert
    16. Juni 2019

    Also es gibt Beta Strahlung mit sehr unterschiedlichen Energien. 15 keV Beta Strahlung kommt keine 5mm weit, während 2 MeV Betastrahlung schon 5 cm weit an Luft entfernt noch wirken kann.

    Beta ist gefährlicher, als Gamma … pro Teilchen. Also pro Strahlungseinheit. Aber auch nicht wirklich viel. So groß ist der Unterschied nicht unbedingt, im Alltag.
    Es kommt halt auch drauf an, wieviele Teilchen da ankommen. 10 Beta Teilchen sind gefährlicher, als 10 Gamma-Teilchen. Aber 100 Gamma-Teilchen sind gefährlicher als 10 Beta Teilchen. Gamma Teilchen haben die Tenden in größeren Rudeln aufzutreten, während Beta, geschweige denn Alpha eher seltener sind.

    Also es gibt Leute, die behaupten, das gesunge Strahlung sogar forderlich für den Körper ist, weil es die Regeneration ankurbelt. … Das glaube ich nicht. Ich teile da die offizielle Meinung, dass jedwede Art von Strahlung schädlich ist. Je mehr, desto schädlicher.

    Als wir evolutionstechnisch noch Bakterien und Einzeller gewesen sind haben wir von der erhöhten Mutationsrate durch Strahlung evolutionstechnisch profitiert. Aber sobald wir entwickelt genug waren haben wir meschanismen entwickelt um uns vor der Strahlung zu schützen und das ist auch gut so.

    In der Medizin wird Strahlung eingesetzt um Zellen zu zerstören. Also wie ein Skalpel oder ein Laser. Ja, aber da sitzt ein Arzt hinter, der die richtigen (Tumor) Zellen zerstört und die gesunden weitgehend gesund lässt. Da wird viel Forshcung betrieben und das ist eine etablierte Methode in der Medizin und wird in Zukunft wenn die Menschehit älter wird und mehr Krebs bekommt auch immer mehr eingesetzt werden.

    Nachmessen geht mit dem 60€ Gerät von ebay mit dem Smartphone auch für Laien. Radioaktivität messen ist sehr einfach. Ich habe da schon mal öfter was zu geschrieben. https://scienceblogs.de/nucular/2016/04/21/detektoren-iii-strahlungsmesser-fuer-den-hausgebrauch-testbericht-smart-geiger-pro-sgp-001/

  177. #179 Miri
    16. Juni 2019

    Zitat von Peter:Sodann: ist es nicht auch so, dass Strahlung (in einem bestimmten Dosen-Verhältnis) nicht auch förderlich für den Körper sein kann?

    Dazu habe ich heute einen interessanten Artikel gelesen:

    https://www.bazonline.ch/wissen/natur/laenger-leben-dank-radioaktivitaet/story/22689973

    was haltet ihr davon ?

  178. #180 Peter
    16. Juni 2019

    @Tobias: Für deine wiederholt ausführlichen Ausführungen besten Dank! Gut, dass du deine Mühen nicht in Rechnung stellst 🙂

    @Miri: In der Tat ein interessanter Artikel. Bei Forschern/Studien muss man allerdings leider immer darauf achten, auf welcher Lohnliste sie stehen. In der Regel sind sie nicht primär der Wahrheitssuche verpflichtet, sondern ihrem Geldgeber. Der Text erinnert aber an das berühmte Zitat von Paracelsus: allein die Dosis machts, dass ein Gift kein Gift sei. Wie von Tobias anempfohlen würde ich jedoch radioaktive Strahlung eher meiden wollen und nicht aufgrund einiger statstischer Winkelzüge darauf vertrauen, dass Radioaktivität gesund ist. Gegenstudien sind bestimmt schon im Anmarsch, wenn sie nicht bereits existieren. (Ob die Studienleiter tatsächlich ihre eigene Medizin kosten würden, fragt man sich.) Panikmache vor allem, was Strahlung betrifft, halte ich dagegen auch für ungesund.

  179. #181 Tobias Cronert
    16. Juni 2019

    @Peter: Gerne. Da jeder deutsche Steuerzahler (naja aktuell Mitglied der Krankenversicherung *g*) für meinen Unterhalt aufkommt geht das eben statt per Rechnung über ne Pauschale 😉

    @Miri: Diese Idee gab es immer wieder und mikroskopisch gesehen gibt es auch Anhaltpunkte die eben für einen solchen Wirkmeschanismus sprechen. Grundsätzlich geht es um oxidativen Stress in der Zelle. Darüber habe ich hier auch schon sehr viel geschrieben. Der springende Punkt ist, das oxidatiiver Stress aktuell sehr sehr stark erforscht wird. Auch ganz unabhängig von ionisierender Strahlung. Es gibt z.B. auch die Theorie, dass Vitamin C als Antioxidanz den oxidativen Stress senk und damit das Leben verlängert, die Gesundehit fördert. Das ist mehr oder weniger der gleiche Wirkmeschanismus, der mit der ionisierenden Strahlung im Gespräch ist.

    Trotz der Forschung auf dem Gebiet gibt es noch keine eindeutigen Ergebnisse. Aber damit ist in den nächsten Jahrzehnten zu rechnen. Sobald dem so ist, wird damit auch diese Diskussion bzgl. ionisierender Niedrigdosen entschieden werden. Da können wir einfach drauf warten und bis dahin weiterhin von der “jede Strahlung ist schädlich”-Prämisse ausgehen. Dafür gibt es übrigends auch die stärksten Studien. Die INWORKS-Studie z.B. ist sehr viel stärker, als alle, die in dem Artikel erwähnt wurden.

  180. #182 Miri
    Eningen
    17. Juni 2019

    Jetzt hab ich auch noch eine Frage..ich war gestern mit meinem kleinen Kind in einer unterirdischen Schauhöhle auf der schwäbischen Alb und bin auch im 6. Monat schwanger und heute ist mir so eingefallen, dass das vielleicht gar nicht so gut war wegen der erhöhten Strahlung in so einem Ding ? Oder macht das nichts ? :-/
    Wir waren so ca 30 Min. drin.

    Danke Tobias für Deine Bereitschaft und Geduld unsere ganzen Fragen zu beantworten 🙂

  181. #183 Tobias Cronert
    17. Juni 2019

    Das ist kein Problem. Wie gesagt denk immer dran, dass der menschliche Körper evolutionstechnisch genug Zeit hatte Abwehrmechanismen gegen Strahlung zu entwickeln und diese Abwehr funktioniert bei natürlicher Strahlung ausreichend gut.

    Die Abwehrmechanismen sind nur überfordert, wenn der Mensch auf die dumme Idee kommt den natürlichen Zustand zu verändern, indem er z.B. den Stein abbaut und das Wohnzimmer nur mit denen plastert, die die tollen Farben haben und damit dann die geringe natürliche Radioaktivität konzentriert.

    Da hat die Evolution dann nicht mehr genug Zeit den Körper dran anzupassen und der dumme Mensch muss sein Gehirn benutzen 😉

  182. #184 UMa
    18. Juni 2019

    Hallo Tobias,

    Du schreibst gerade relativ viel.
    Vielleicht/Hoffentlich geht es Dir soweit ganz gut, dass Du eine meiner Fragen beantworten kannst?

    Ich habe eine Frage zu kosmischer Strahlung und deren Sekundärstrahlung (hauptsächlich Myonen und Neutronen).
    Bei dieser ist die Jahresdosis im freien Weltraum ca. 600 mSv, nahe einem Planeten ca. 300 mSv (der halbe Raum ist vom Planeten verdeckt), an der Erdoberfläche (Meereshöhe) noch 0,3 mSv, dann hauptsächlich Sekundärstrahlung. Also verringert die Erdatmosphäre die Strahlung auf etwa 1/1000.

    Jetzt hat Alderamin diesen Artikel geschrieben
    https://scienceblogs.de/alpha-cephei/2019/06/17/die-sonne-ein-superflare-stern/

    Bei einem möglichen solaren Flare im Jahre 774 wurde der C-14 Gehalt der Atmosphäre um 1,2% erhöht. Das entspricht etwa einer C-14 erzeugenden Strahlung wie sonst in 100 Jahren.

    Falls die am Boden ankommende Strahlung des Flares proportional zu der das C-14 erzeugenden Strahlung ist, wäre die Strahlendosis auf Meereshöhe immerhin 30 mSv gewesen, was sehr hoch erscheint.

    Andererseits wird in diesem Paper ein viel geringerer am Boden ankommender Anteil an Strahlung bei Superflares berechnet, was auf sehr viel weniger als 30 mSv am Boden deuten würde:
    https://arxiv.org/abs/1906.06797

    Fragen:
    Woran könnte die unterschiedliche Strahlung am Boden liegen? Warum bekommen wir überhaupt noch 0,3 mSv aus der kosmischen Strahlung ab, aber kaum was bei sehr starken solaren Flares, die außerhalb der Atmosphäre viel stärker sind?

    Liegt es vielleicht daran, dass die Energie der galaktischen kosmischen Strahlung sehr viel höher ist als die bei solaren Flares?
    Oder verstehe ich hier etwas nicht richtig?

  183. #185 Peter
    21. Juni 2019

    Stellen eigentlich (radioaktiv strahlende) Rauchmelder ein Risiko da? Oder sondern diese zu wenig ab?
    Was passiert eigentlich mit einer kleinen Spinne, die da an der Deckenwand hineinkriecht? Und kann die was verschleppen?

  184. #186 Tobias Cronert
    21. Juni 2019

    OK, erst mal kurz für Peter, dann lang für UMa.

    @Peter: Rauchmelder mit Ionenkammer sind alleine harmlos. Da bräuchte man schon mehr als 10 aus denen man die Kammern ausbaut und auf einen Haufen legt um eine signifikante Strahlung zu erzeugen. Das kann man natürlich machen, aber das muss man bewusst tun. So wie sie normalerweise verbaut sind ist alles gut.
    Das Material ist auch immer verkapselt, da kann keine Spinne etwas heraustragen. Wieso muss ich jetzt an Spiderman denken *g*

    @UMa: Also erstmal muss ich sagen, dass kosmische Strahlung absolut nicht mein Gebiet ist und ich zwar des öfteren schon Lust habe mich da vernünftig reinzuarbeiten, aber das dann auch vernünftig machen wollen würde und das viel Zeit kostet. Daher sind meine Aussagen da alle mit viel Vorsicht zu genießen.

    So, nach dem Disclaimer aber zu den wilden Spekulationen 😉 Also die ganzen Primärteilechen sind ja geladen und treffen auf das EM-Feld der Erde, wodurch dann die ganze Sekundärstrahlung entsteht. Das Magnetfeld ist ein Dipolfeld mit der entsprechenden Charakteristik und dann so tollen Effekten, wie der Aurora an den Polen. Das EM-Feld ist aber sehr sehr schwach und hat nur eine so große Auswirkungen, weil es langreichweitig ist und weit in den Raum erstreckt. Diese ganzen Bilder, die man im Kopf hat, von der Erde und einem Magnetfeld, dass dann den doppelten Radius der Erde hat sind von den Größenverhältnissen total falsch und führen zu einen fehlerhaften Verständnis. Die richtige Skala ist eher Erde=Kirschkerngröße und effektives EM-Feld=Kölner Dom. Die geladenen relativistischen Teilchen brauchen viel viel Wegstrecke in dem EN-Feld um einen Effekt zu spüren, aber es reicht eben schon aus um das Dichtefeld der Sonnenstrahlung stark zu beeinflussen.

    Die Athmosphäre / Exophhäre ist dagegen winzig, aber so dicht, dass geladenen Teilchen echte Probleme bekommen und ungeladenen Sekundärteilchen bevorzugt werden. Sonst hätten Neutronen z.B. keine Chance und auch Myonen schaffen es ja nur mit relativistischen Tricks auf die Erde zu kommen.

    Wenn jetzt die verhältnissmäßig winzige Zone unter der Karman Linie so eine große Auswirkung auf die Primär und Sekundärstrahlung hat, dann muss ich vermuten, dass ihre Zusammensetzung eine ebenso (wenn nicht noch größere) gewaltige Auswirkung auf die Strahlung hat … und davon habe ich jetzt schlicht gar keine Ahnung mehr. Aber aus meinen “Was ist Was”-Büchern im Kinderzimmer weis ich noch, dass die Zusammensetzung unserer Luftkugel sich in der Geschichte extrem stark geändert hat und aus dem Funk weiß ich, dass ein einzelner Vulkanausbruch auf der Erde die Effektivität von Funkwellen mal locker um 20-50% hoch oder runter manipulieren kann. Der Effekt für geladenen Teilchen wird da noch dramatischer sein.

    Ob die geladenen Teilchen jetzt eine Größenordnung mehr oder weniger Energie haben (Also Flare vs. galaktische Strahlung etc. pp.) sollte mMn keinen Unterschied machen. Je höher die Energie, desto schneller fallen sie 😉
    Die meisten geladenen Teilchen verlieren bei der ersten interaktion die Hälfte ihrer Energie, bei der zweiten noch mal die Hälfte etc. pp. und das bei mehreren hundert Interaktionen. Auch werden sie früher vom EM-Feld abgelenkt und da braucht es halt nur einen kleinen Raumwinkel um an der Erde vorbeizukommen. Wenn ich also raten müsste, dann wäre meine Erklärung einfach und simpel “Zeug in der Luft”. Auch wenn sich das jetzt leider etwas banal für einen faszinierenden Zusammenhang anhört.

  185. #187 Peter
    21. Juni 2019

    Danke!
    Ich musste auch an Spiderman denken! Fast schon schade, dass die Spinne nichts heraustragen kann, ich hätte mich schon so auf Spiderman 2.0 gefreut. 🙂

  186. #188 UMa
    22. Juni 2019

    @Tobias:
    Ich hätte jetzt vermutet, dass im Magnetfeld bestenfalls Bremsstrahlung entsteht, während die Sekundärstrahlung erst mit der Kollision der Teilchen der kosmischen Strahlung mit Teilchen der Atmosphäre entsteht.
    Das Magnetfeld scheint eine große Wirkung auf Teilchen niedriger Energie zu haben, während Teilchen hoher Energie durch gehen und erst in der Atmosphäre reagieren.

  187. #189 Tobias Cronert
    22. Juni 2019

    Nein, ich denke Sekundärsttrahlung wird im Magnetfeld auch nicht viel erzeugt, aber Ablenkung aus dem Raumwinkel, der die Erde treffen würde … naja OK, macht dann wohl auch keinen Unterschied, weil die nie ankommen oder bestenfalls auf eine Bahn mit Dichtegewinn an den Polen abgelenkt werden *g*

    Wie gesagt ich rate hier nur. Ich fühle mich dabei nicht ansatzweise kompetent genug um eine qualifizierte Aussage zu treffen.

    Vom Teilchenbeschleuniger her weis ich halt, dass die Qualität des Vakuums einen riesigen Unterschied bzgl. der Strahlqualität macht. Weis jemand, wie stark das Magnetfeld die Dichte der Primärstrahlung verändert? Vielleicht hat das ja eine Relevanz … oder auch nicht ich hab gehört das Vakuum im Weltraum soll ganz gut sein *g*

  188. #190 rolak
    9. Juli 2019

    (Bitte in Form einer Frage!)

    Kann es sein, daß Dein Beharren auf ‘c’ Auswirkungen auf den Rest der Gesellschaft hat?

    Oder: Der Nuckel von King Lear ward vom Medicus verordnet.

  189. #191 Tobias Cronert
    11. Juli 2019

    ?

  190. #192 zimtspinne
    11. Juli 2019

    Wie heißen denn nun eigentlich deine Katzen?

  191. #193 Peter
    11. August 2019

    Als wie gefährlich ist eigentlich der neueste Atomunfall am Weißen Meer einzuschätzen?

  192. #194 Beobachter
    12. August 2019

    zu # 193:

    Aktuelle Meldungen dazu – z. B.:

    https://www.dw.com/de/immer-mehr-details-zu-atomunfall-in-sewerodwinsk-russland/a-49987172

    Vermutlich kann man es noch gar nicht einschätzen, weil es dazu keine oder zu wenig verlässliche und unabhängige Informationen gibt.
    Wie die “Risiko-Kommunikation” bei Atomunfällen aussieht, ist ja mittlerweile bekannt …

  193. #195 Tobias Cronert
    12. August 2019

    Joa, ohne mehr Informationen kann man da nur wild spekulieren.

    Wenn dieser Marschflugkörper “auf Basis einer Radioanuklidbatterie” funktieoniert, dann kann da alles und nichts drin sein. Von ganz ekelhaften Sachen, bis hin zu nahezu harmlosem Krempel.

    Wenn Greenpeace sagt, es wären für “mehrere Stunden” 2µSv an Strahlung gewesen, dann ist das annähern gar nichts. Ich habe öfter 8h Arbeitstage mit 10µSv/h verbracht und das ist immer noch keine nennenswerte Dosis. … und Greenpeace benutze ich gerne als Quelle, weil man sich recht sicher sein kann, dass die die Zahlen nicht irgendwie kleinrechnen werden.

    Das geht natürlich davon aus, dass keine Radionuklide irgendwie in die Umwelt entkommen sind oder so.

  194. #196 Peter
    12. August 2019

    Und was für eine Reichweite hätten entkommene Radionuklide?

  195. #197 Beobachter
    12. August 2019

    Da wird es wohl drauf ankommen, wohin sie entkommen – ob ins Wasser, in die Luft, …
    Und dann entsprechend auf Meeresströmungen, Luftströmungen, Wetterbedingungen, …
    Oder?

  196. #198 Beobachter
    12. August 2019

    Etwas Aktuelleres hab ich nicht gefunden:

    https://www.welt.de/vermischtes/article198328633/Atomarer-Unfall-in-Russland-Behoerden-geben-Details-preis.html

    Also bisher: Nix Genaues weiß man nicht …

  197. #199 Beobachter
    13. August 2019

    https://www.arte.tv/de/afp/neuigkeiten/rosatom-entwicklung-neuer-waffen-wird-auch-nach-atom-unfall-fortgefuehrt

    Es geht hier auch noch um atomare Aufrüstung, nicht “nur” um die “friedliche Nutzung der Kernenergie” – beides ist eh kaum zu trennen.
    Was ist waffenfähig und was nicht?
    Wer kann das überall kontrollieren?

    Es sind Menschen bei diesem Unfall gestorben, “wahre Helden”; informationsmäßig wird gemauert und verharmlost – und “abgeschirmt”.

    Der “zivilisatorische Fortschritt der Menschheit” –
    die Atombombenabwürfe in Nagasaki, Hiroshima, schon vergessen?
    Die Menschen dort leiden bis heute darunter.

    Die Katastrophen von Tschernobyl und Fukushima werden bis heute verharmlost – und das auch noch auf zynischste Art und Weise.

    Wie kann es da noch irgendwen wundern, dass jeder vernünftige Mensch nur immer wieder sagen kann:
    “Atomkraft, nein danke!”

    Wer heute noch FÜR Kernkraft ist, unterliegt einem blinden “Fortschritts”- und Technikglauben und befürwortet ein “grenzenloses Wirtschaftswachstum”, das es nicht geben kann.

  198. #200 Beobachter
    13. August 2019

    https://www.dw.com/de/raketentests-gehen-nach-atomunfall-in-russland-weiter/a-49998995

    ” … “Stolz der Atombranche”

    Für die umgekommenen Rüstungsexperten wurde am Montag in der Stadt Sarow, die fast vollständig von der Öffentlichkeit abgeschirmt ist, eine Trauerzeremonie veranstaltet. Der frühere Rosatom-Chef Sergej Kirienko würdigte die Getöteten als “wahre Helden” und “der Stolz der Atombranche”.
    Bereits Anfang Juli hatte ein Kreml-Sprecher die Hintergründe eines verheerenden Brandes an Bord eines russischen U-Boots, bei dem 14 Marine-Soldaten ums Leben gekommen waren, zum “Staatsgeheimnis” erklärt. Der Brand wurde nach Angaben des Verteidigungsministeriums in Moskau unter Kontrolle gebracht, bevor der Atomreaktor des U-Boots beschädigt wurde.”

  199. #201 Beobachter
    13. August 2019

    Nun auch im SPIEGEL:

    https://www.spiegel.de/politik/ausland/russland-und-der-skyfall-unfall-wladimir-putins-wunderwaffe-faellt-ins-wasser-a-1281709.html

    “Atomunfall in Russland
    Putins fatale Wunderwaffe
    Donald Trump stichelt, Russland trauert: Fünf Tage nach dem rätselhaften atomaren Unfall auf einem Testgelände am Weißen Meer kommen langsam neue Details ans Licht. … ”

    “… ,Wladimir Putin habe sie bereits für staatliche Auszeichnungen vorgeschlagen. In einem Jahr soll ein Denkmal für die Toten eröffnet werden. “Die Fähigkeit, eine Großmacht zu sein, wird auch mit Opfern errungen”, sagte der Regionalgouverneur Gleb Nikitin.
    Fraglich ist, was mit der nuklearen Energiequelle – ob Kleinreaktor oder Atombatterie – passiert ist. “Dass die Radioaktivität in Sewerodwinsk schnell wieder gesunken ist, lässt mich vermuten, dass die Vorrichtung ins Wasser gefallen ist”, sagte der Atomexperte Alexander Nikitin von der Umweltorganisation Bellona dem SPIEGEL.”

    Der Gipfel des Zynismus und das Protobeispiel von “transparenter Risiko-Kommunikation”, wenn es um Atomunfälle geht !

  200. #202 Tobias Cronert
    13. August 2019

    OK, wir können ja mal versuchen Fakten aufzuzählen.

    Die Toten sind an einem “normalen” Unfall gestorben. Sprich mechanisch oder chemisch. Wäre dies durch Radioaktivität geschehen, dann hätte Greenpeace nicht von 2µSv/h Dosisleistung gesprochen.

    2 µSv/h Dosisleistung entspricht einem normalen Pechblende Stein aus dem Erzgebirge, den man beim Wandern aufsammeln kann.

    Eine Radionuklidbatterie hat viel viel mehr Radioaktives Material, als besagter Stein aus dem Erzgebirge. Wenn also nur 2µSv/h entstanden sind und wirklich eine Radionuklidbatterie beteiligt gewesen ist, dann heißt das, dass vielleicht mal die äußere Abgekung geöffnet gewesen ist. Ein Aufbrechen des Behälters, in dem die Radionuklide lagern, hätte SEHR viel mehr Radioaktivität freigesetzt.

  201. #203 tomtoo
    13. August 2019

    Nun hat sich ja auch der POTUS gemeldet.
    Um was immer es auch geht, die USA haben was besseres(hüstel).

  202. #204 Tobias Cronert
    13. August 2019

    Na das machts bestimmt besser.

  203. #205 Peter
    14. August 2019

    Aktuelle Artikel warten mit Informationen auf, denenzufolge immer noch nicht ganz klar ist, was genau explodiert ist, dass aber kurze Zeit später in der mehrere Kilometer weit entfernten größeren Stadt in der Luft das 20fache der natürlichen Radioaktivität gemessen wurde (und das für wohl mehr als eine Stunde).
    Dies lässt doch den Schluss zu, dass es eine radioaktive Wolke gab bzw. gibt? Folglich stehen weitere Fragen im Raum: in welche Windrichtung wehte die Wolke? Ist sie in Richtung Deutschland unterwegs (gewesen)? Wann kommt/kam sie an? Ist sie dann immer noch als gefährlich einzuschätzen?
    Das 20fache klingt hoch?

  204. #206 Beobachter
    15. August 2019

    zu # 205:

    https://www.tagesspiegel.de/politik/explosion-auf-raketen-testgelaende-was-ueber-den-mysterioesen-atom-unfall-in-russland-bekannt-ist/24899424.html

    ” … Im Umgang mit nuklearen Zwischenfällen ist Moskau berüchtigt. Jetzt sorgt eine weitere Panne für Unruhe, nachdem es vor knapp einer Woche zu einer Explosion auf einem Raketen-Testgelände in Nordrussland gekommen ist. Informationen darüber, was genau geschah, sind – wie in solchen Fällen üblich – spärlich. Erst am Samstag hatten Behörden den atomaren Charakter des Unglücks eingeräumt. Doch nach und nach gelangen immer mehr Details an die Öffentlichkeit. Experten sind besorgt. … ”

    ” … Die Umweltorganisation Greenpeace sprach unter Berufung auf die Stadt von 2,0 Mikrosievert pro Stunde. Deren Experten hielten den Wert „an sich für nicht dramatisch“. Es komme vielmehr darauf an, welche strahlenden Stoffe freigesetzt worden sein. Dazu gebe es aber keine offiziellen Angaben. … ”

    ” … Kritik aus dem Ausland
    Im Ausland gibt es jedoch die Befürchtung, dass die russischen Behörden – wie in der Vergangenheit – nicht über das wahre Ausmaß des Unfalls informiert hätten. … ”

    Außerdem:
    Wenn man nicht an “unbezwingbaren” Nuklearwaffen bauen/sie testen würde, käme es auch gar nicht zu den entsprechenden Atomunfällen mit Toten, Strahlenopfern und Langzeitgeschädigten, zur Verseuchung der Umwelt.
    Es ist alles vermeidbar !
    Oder glaubt irgendwer (immer noch), dass sich Konflikte mit (Nuklear)Waffengewalt lösen lassen ?!

    (Anm./Frage für/an Stamm-Kommentator/-Mitleser und Dipl.-Chemiker RPGNo 1, besonders im OLT (“Offenen Laber Thread”):
    Ist dieser Kommentar wieder “moralinsauer” ?!)

  205. #207 Captain E.
    15. August 2019

    @Beobachter:

    […]

    Oder glaubt irgendwer (immer noch), dass sich Konflikte mit (Nuklear)Waffengewalt lösen lassen ?!

    […]

    Zugegeben, das ist schon ziemlich makaber, aber ja: Das ist möglich. Der Kalte Krieg konnte doch nur deshalb stattfinden, weil beide Seiten (viel zu?) viele Atomwaffen hatten. Ohne diese Dinger hätten wir vielleicht schon in den 50er Jahren den Dritten Weltkrieg ausgekämpft. Wir werden es niemals wissen.

  206. #208 Captain E.
    15. August 2019

    @Peter:

    Aktuelle Artikel warten mit Informationen auf, denenzufolge immer noch nicht ganz klar ist, was genau explodiert ist, dass aber kurze Zeit später in der mehrere Kilometer weit entfernten größeren Stadt in der Luft das 20fache der natürlichen Radioaktivität gemessen wurde (und das für wohl mehr als eine Stunde).

    So ganz habe ich noch nicht verstanden, wie dieser Marschflugkörper funktioniert. Er soll mittels Radionuklidbatterie ewig und drei Tage in der Luft bleiben können. Für den Antrieb sollte so eine Radionuklidbatterie allerdings ein bisschen zu schwach sein. Vielleicht versorgt sie den Flugkörper auch nur mit Strom und der Treibstoff ist ein konventioneller, und der könnte dann explodiert sein. Die vagen Informationen könnten ja darauf hindeuten, dass das RTG bei der Explosion beschädigt und durch die Luft geschleudert wurde. Der Austritt des strahlenden Materials hätte dann entweder mit der schnellen Bergung geendet oder wäre mit dem Versinken im Wasser unter die Nachweisgrneze verdünnt worden.

    Übrigens, mit thermoelektrischen, thermionischen oder thermophotovoltaischen Generatoren, Betavoltaics, alkalimetall-thermisch-elektrischen Wandlern und Stirlingmotoren stehen theoretisch bzw. praktisch eine Reihe von Umwandlern zur Verfügung. Besonders explosionsgefährdet erscheint mir dabei der alkalimetall-thermisch-elektrische Wandler zu sein, der mit flüssigem Natrium arbeitet. Vielleicht haben die Russen so etwas verwendet.

    https://de.wikipedia.org/wiki/Radionuklidbatterie

    Dies lässt doch den Schluss zu, dass es eine radioaktive Wolke gab bzw. gibt? Folglich stehen weitere Fragen im Raum: in welche Windrichtung wehte die Wolke? Ist sie in Richtung Deutschland unterwegs (gewesen)? Wann kommt/kam sie an? Ist sie dann immer noch als gefährlich einzuschätzen?

    Ich würde mal vermuten, dass man in Deutschland nichts mehr wird nachweisen können oder allenfalls mit extrem empfindlichen Messgeräten. Wirklich gefährlich ist die Wolke aber wohl selbst am Orte des Geschehens nicht, und bei uns schon gleich gar nicht.

    Das 20fache klingt hoch?

    Du hast es ja gelesen: Das zwanzigfache ist noch nicht wirklich dramatisch.

  207. #209 Beobachter
    15. August 2019

    @ Captain E.:

    Das ist mehr als makaber.
    Und es werden immer wieder “diese (beiden) Dinger”, die Atombomben, die man auf Japan im 2. Weltkrieg abgeworfen hat, “vergessen”.
    Reichen die immer noch nicht ?!
    Und wir wissen, dass “diese Dinger” den 2. Weltkrieg NICHT verhindert haben !

    Es genügt das Drücken eines (symbolischen) Roten Knopfes, um eine militärische nukleare Katastrophe auszulösen –
    und man weiß ja, welche Leute heutzutage am (auch atomaren) Drücker bzw. in Machtpositionen sitzen !
    Und sich drüber streiten/damit prahlen, wer den dicksten (roten Knopf) hat – kein Witz … ! :

    https://www.welt.de/politik/ausland/article172118267/Nuklear-Vergleich-Trump-hat-einen-viel-groesseren-Atomknopf-als-Kim.html

  208. #210 tomtoo
    15. August 2019

    @Beobachter
    Es macht halt keinen Sinn Kernwaffen und Kernkraftwerke in einen Topf zu schmeisen.

  209. #211 Captain E.
    15. August 2019

    @Beobachter:

    Das ist mehr als makaber.

    Mehr als makaber? Wie willst du so etwas denn sonst nennen?

    Und es werden immer wieder “diese (beiden) Dinger”, die Atombomben, die man auf Japan im 2. Weltkrieg abgeworfen hat, “vergessen”.
    Reichen die immer noch nicht ?!

    Das halte ich für eine ziemlich steile These, dass irgendjemand die Atombomben vergessen könnten, die den Zweiten Weltkrieg beendet haben.

    Aber ja, sie haben dazu gereicht, die Japaner kapitulieren zu lassen. Andernfalls wären etliche Millionen Japaner mehr in diesem Krieg gestorben, und vielleicht auch einige hunderttausend amerikanische Soldaten.

    Und sie reichen nach wie vor aus, dass sich seither nie wieder jemand getraut hat, sie gegen einen Feind einzusetzen. Das ist in der Geschichte der Waffenentwicklung schon etwas ganz Besonderes.

    Und wir wissen, dass “diese Dinger” den 2. Weltkrieg NICHT verhindert haben !

    Wie hätten sie das auch tun können? Der Zweite Weltkrieg begann 1935, 1936 oder 1941, je nachdem wen man fragt. Trinity, die erste Testzündung, erfolgte aber erst 1945. Der Krieg war nach den nächsten beiden Detonationen in diesem Jahr bekanntlich ziemlich schnell vorbei.

    Es genügt das Drücken eines (symbolischen) Roten Knopfes, um eine militärische nukleare Katastrophe auszulösen –
    und man weiß ja, welche Leute heutzutage am (auch atomaren) Drücker bzw. in Machtpositionen sitzen !

    Du meinst Menschen, die sich noch nie getraut haben, diesen (symbolischen) Roten Knopf zu drücken? Und das hoffentlich sich auch nicht trauen werden bzw. müssen?

    Und sich drüber streiten/damit prahlen, wer den dicksten (roten Knopf) hat – kein Witz … !

    https://www.welt.de/politik/ausland/article172118267/Nuklear-Vergleich-Trump-hat-einen-viel-groesseren-Atomknopf-als-Kim.html:

    Lass ihnen doch diesen kleinen Spaß! Hauptsache ist doch, dass sie es beim Reden belassen.

  210. #212 Beobachter
    15. August 2019

    @ tomtoo, # 210:

    Doch, es macht sogar großen Sinn:

    Die Technologie ist die gleiche (ob waffenfähige Produkte oder nicht, ist eh nicht kontrollierbar), bei der Produktion/Testung von Kernwaffen und beim Betrieb von AKWs passieren Unfälle bis Katastrophen.

    Und beim Thema “Kernenergie” kann man vom Irrsinn der Kernwaffen nicht einfach “absehen” !

    So, und jetzt habe ich weder Zeit noch Lust, diese fruchtlose Im-Kreis-Diskussion fortzusetzen …

    Bleibt halt AKW- und Kernwaffen-Fans –
    solange, bis euch höchstselbst “der Himmel auf den Kopf fällt” … ! 🙂
    “Skyfall … “:

    https://www.spiegel.de/politik/ausland/russland-und-der-skyfall-unfall-wladimir-putins-wunderwaffe-faellt-ins-wasser-a-1281709.html

  211. #213 Captain E.
    15. August 2019

    @Beobachter:

    Doch, es macht sogar großen Sinn:

    Die Technologie ist die gleiche (ob waffenfähige Produkte oder nicht, ist eh nicht kontrollierbar), bei der Produktion/Testung von Kernwaffen und beim Betrieb von AKWs passieren Unfälle bis Katastrophen.

    Das ist aber schon eine ziemliche Vereinfachung, nicht wahr? In einem Kernkraftwerk detonieren keine Kernwaffen, und es gibt auch keine vorstellbare Katastrophe, in der sich ein Kernkraftwerke wie eine solche Waffe verhalten könnte. Die Brennstäbe können mit Plutonium betrieben werden, aber zumeist nimmt man angereichertes Uran (oder verwendet schweres Wasser). Kernwaffen sind heutzutage (fast) alle Fusionswaffen, die mit einer eher kleinen Fissionsbombe gezündet wird. Die wiederum funktioniert ausschließlich mit Plutonium. Gut, eine Drei-Stufen-Bombe mit abgereichertem Uran in der äußeren Hülle wird dann wieder eine echte Schweinerei und sollte noch weniger zur Anwendung gelangen als die “normale” Wasserstoffbombe.

    Klar ist aber, dass Kernkraftwerke mehr oder minder gut darin sind, waffenfähiges Plutonium zu erbrüten. Es bringt aber trotzdem nichts, auf die zivile Nutzung zu verzichten, wenn die Atommächte weiterhin spezielle Reaktoren betreiben, um an ihr Waffenplutonium zu gelangen.

    Und beim Thema “Kernenergie” kann man vom Irrsinn der Kernwaffen nicht einfach “absehen” !

    Sicher kann man das. Letztlich ist unsere gesamte Existenz ohne Kernenergie überhaupt nicht vorstellbar. Die Nutzung in militärischer Form als Waffe ist da nur ein Aspekt, die zivile (und auch militärische) als ergiebiger Energiequelle ein anderer.

    So, und jetzt habe ich weder Zeit noch Lust, diese fruchtlose Im-Kreis-Diskussion fortzusetzen …

    Bleibt halt AKW- und Kernwaffen-Fans –
    solange, bis euch höchstselbst “der Himmel auf den Kopf fällt” … !

    Wie kommst du denn zu dem Schluss, wir seien Kernwaffenfans? Oder dass wir allesamt dermaßen glühende Anhänger von Kernkraftwerken seien, dass die Bezeichnung “Fans” gerechtfertigt sei?

    “Skyfall … “:
    https://www.spiegel.de/politik/ausland/russland-und-der-skyfall-unfall-wladimir-putins-wunderwaffe-faellt-ins-wasser-a-1281709.html

    Aber das sollte dir doch gefallen, oder? Ein nuklear betriebener und mit an Sicherheit grenzender Wahrscheinlichkeit nuklear bestückbarer Flugkörper, der lange und tief fliegen und seine tödliche Fracht überraschend praktisch überall auf der Erde ans Ziel bringen kann, funktioniert nicht allzu gut.

    Aber jetzt stell dir doch selber mal eine Frage: Solange Russland über derartige Waffen verfügt, möchtest du da wirklich, dass Verbündeten deines Heimatlandes ihre eigenen Waffen verschrotten?

    Und ja, auch das ist ein ziemlich makabrer Aspekt der ganzen Sache.

  212. #214 Tobias Cronert
    15. August 2019

    So erstmal muss ich hier den Diskussionen einen kleinen Riegel vorschieben. Ich könnt in allen meinen anderen Artikeln gerne über politischen Zweck und Implikationen von Atomwaffen, Kraftwerken etc. pp. diskutieren. Wie z.B. gerade bei “der Horrokarte von Fukushima”.

    Aber bitte nicht hier im “Fragen an den Autor”-Artikel. Denn hier wird das Lesen einfach zu schwierig, wenn es viele Kommentare mit Meinungen und Gegenmeinungen gibt.

    Hier sollen einfach nur Fragen gestellt und beantwortet werden, mit maximal ein paar Nachfragen zu dem entsprechenden Thema. Also Back to Topic bitte.

    Dies lässt doch den Schluss zu, dass es eine radioaktive Wolke gab bzw. gibt? Folglich stehen weitere Fragen im Raum: in welche Windrichtung wehte die Wolke? Ist sie in Richtung Deutschland unterwegs (gewesen)? Wann kommt/kam sie an? Ist sie dann immer noch als gefährlich einzuschätzen?
    Das 20fache klingt hoch?

    Das 20 fache ist sehr niedrig. Die Radioaktivität ist z.B. im Erzgebirge oft 100fach höher, als in Hannover. Das ist vollkommen normal.

    Eine “radioaktive Wolke” von 2µSv/h produziere ich ganz einfach in dem ich eine handvoll Sand aus dem Erzgebirge in die Luft werfe.

    Was mich dabei allerdings etwas verwirrt ist, dass dieses Ereignis überhaupt gemessen wurde. Weil einen Erhöhung für 2µSv für mehrere Stunden ist eigentlich so gering, dass es in der Messgenauigkeit untergehen würde. Wenn das in Deutschland passieren würde, dann würde da kein Hahn nach krähen

    Es sei denn…
    es wurde nicht integral gemessen, sondern für bestimmte Radionuklide aufgeschlüsselt. Wenn für ein einzelnes Radionuklid plötzlich die Dosisleistung um 2µSv/h in der Luft steigen würde, dann würde man sich schon fragen: “Wo zum Geier kommt das her?” Vor allem, wenn es ein exotischeres Nuklid ist, dass in der Natur nicht vorkommt.

    Das ist in Deutschland auch schon öfter vorgekommen, dass man einen Anstieg für ein exotischen Nuklid in der Luft gefunden hat und dann direkt fragte: “Was zum Geier machen die Russen denn da gerade wieder?”

    Das ist halt der Nebeneffekt, dass man Radiaktivität so gut und präzise messen kann.

  213. #215 zimtspinne
    15. August 2019

    Tztz,
    Herr Tobi,
    ich hatte nach den Namen der Katzen gefragt (kurz und knapp) und bisher keine Antwort!?
    Oder fällt das unter streng geheim zum kätzischen Datenschutz?
    Dann ziehe ich die Frage natürlich schnell zurück. Will die Fellträger ja nicht in Schwierigkeiten bringen.

  214. #216 Tobias Cronert
    15. August 2019

    Oh, sorry. Ich bitte vielmals um Entschuldigung.

    Der Dicke heißt Zorro und seine kleine Schwester Chibi. Wir hatten nichts mit der Namenswahl zu tun *g*

  215. #217 Captain E.
    15. August 2019

    Ganz andere Frage:

    Da stürmen die Russen nach der Zerstörung ihrer Nuklearrakete die Apotheken auf der Suche nach Jodtabletten. Das hilft aber nur gegen radioaktives Jod, wie es bei der Kernspaltung anfällt. Macht das also irgendeinen Sinn?

  216. #218 Tobias Cronert
    15. August 2019

    Nö, gegen Sachen aus einer Radionuklidbatterie nützen Jodtabletten absolut gar nichts.

    Menschen haben halt Angst und Panik und reagieren dann irrational.
    Ist allerdings in gewisser Weise auch verständlich, wenn man in Russland lebt, die Informationspolitik dort gewohnt ist, aber trotzdem das Internet zur Verfügung hat.

    https://scienceblogs.de/nucular/2016/04/14/kurz-notiert-jodtabletten-gegen-strahlenkrankheit/

    https://scienceblogs.de/nucular/2017/10/02/ausgabe-von-jodtabletten-in-aachen-und-umgebung-fuer-den-atomaren-notfall/

  217. #219 zimtspinne
    15. August 2019

    och, sind die toll!
    und beim Dicken ein bisschen Norweger oder anderes Halblanghaar dabei?
    Also bei beiden, aber bei ihm erkennt man es mehr. Ach ja, und dann gibts natürlich auch manches Mal Spermienkriege, so dass die Gesschister eines Wurfes tatsächlich auch mal unterschiedliche Väter haben können 😉

    Wie alt sind die denn und sind das deine oder haben sie die ganze WG unter Kontrolle?

  218. #220 Tobias Cronert
    15. August 2019

    Bei dem Dicken haben wir stark den Verdacht, dass da ein gutes Stück Maine coon (oder ähnliches) mit reingeraten ist, auch vom Verhalten her und das sie zwei verschiedenen Väter haben.

    Die beiden gehören zur WG und auch nach deutschem Recht nicht “zu mir”. Sie selber haben auch eine klare Hirarchie ihrer bevorzugten Zweibeiner/Dosenöffner und ich bin da eher am unteren Ende der Nahrungskette.

  219. #221 Peter
    15. August 2019

    Vielen Dank an alle Disskusionsteilnehmer für die Beiträge zu den von mir in den (Chat-)Raum geworfenen Fragen (vgl. #205), insbesondere für den ausführlichen Beitrag von dir, Tobias (#214)!

    Wenn erstmal alle Informationen durchgesickert sind, verhält es sich hoffentlich immer noch so, dass aktuell nicht von allzuviel Gefahr auszugehen ist!

    Wenn die bisherigen Messungen so niedrig sind und dich – Tobias – verwundern, könnte dies dann vielleicht auch daran liegen, dass sie geschönt sind??

  220. #222 Tobias Cronert
    16. August 2019

    Naja, der Wert von 2µSv/h über mehrere Stunden ist ja auch von Seiten von Greenpeace nur indirekt und daher wenig belastbar.

    Ich meine eine vernünftige öffentliche Auskunft würde beinhalten, was genau gemessen wurde (Luft?), Wann und Wo, Messmethode (Gamma?, Energieaufgelöst?) und idealerweise auch noch mit welchem Messgerät.

    Das was wir aktuell so an Informationen haben ist von einer “vernünftig staatlich dokumentierten Messung”, wie ich sie z.B. vom Bundesamt für Strahlenschutz erwarten würde, meilenweit entfernt.

    Vielleicht kommt da irgendwann ja noch was nach.

  221. #223 Peter
    17. August 2019

    Danke für die Einblicke!
    Man kann nur hoffen, dass etwaige Nachreichungen von Daten harmloser Natur sein werden. Aber Skepsis ist wohl leider nach wie vor angebracht.

  222. #224 demolog
    17. August 2019

    @ rolak
    27. Mai 2019 #161

    @ Tobias Cronert
    27. Mai 2019 #162

    Zitat:
    Aber der Dünger wird halt auch nicht gefährlicher sein, als Erde aus dem Erzgebirge oder den Alpen.

    -> Tja, wenn alles so liegen bleibt, wie es sich auf der Erde so abgelagert hat, dann dürfte es da kein Problem geben.

    Aber was ist mit landwirtschaftlichen Flächen, wo regelmäßig Dünger auftragen wird?

    Reichert sich da nicht das Zeugs an? Oder wird damit nicht das Problem verstärkt, wenn man Dünger mit hohem Anteil an radioaktivem Material aufträgt?

    Zum Beispiel auf flächen des Tabakanbaus: Hier werden Pflanzen meistens auf die maximal wachstgumsfördernde Weise gepflanzt und gezüchtet. Daraus ergäbe sich, dass im Boden durch den übermäßígen Eintrag von Dünger (und damit auch radioaktive Elemente) auch mehr radioaktive Elemente angereichert werden.
    Und die gelangen dann im ungünstigen Fall auch in die Pflanze.

    Übrigens scheint mir das mit dem Uran im Dünger nicht zu stimmen. Hier steht etwas anderres dazu:
    https://de.wikipedia.org/wiki/Tabakrauchen#Radioaktivit%C3%A4t

    Dort ist es Polonium210. Was mich etwas verwundert.

    Weil es einst hiess, dieses Element käme auf der Erde natürlich nahezu nicht vor. Und nun soll es ausgerechnet im Phosphordünger signifikant höher vorkommen?

    Und den Tabak also verseuchen?

    Waren der Lidvenjenkow und der Arafat etwa starke Raucher, dass sie das Element in sich trugen, wie Sand am Meer?

  223. #225 demolog
    17. August 2019

    @ Tobias Cronert
    15. August 2019

    Tja,. russische “Informationspolitik”…wenn sie das schon so nennen, dann wird wohl was dahinter stecken, nicht wahr?

    Aber was?

    2 mykrosivert/stunde ist nicht viel mehr, als Umgebungsstrahlung. Aber wo ist das gemessen worden?

    Sie erklärten ja selbst, wie spärlich hier die Informationen sind.

    Aber nach der Wetterkarte (Winde) an dem Datum und darauf muß man doch was in Finland gemessen haben. Es gab neben der Hauptwindströhmung nach Osten auch eine langsame, aber stetig niedrige Windströhmung nach Westen.

    Das kann man hier ablesen:
    https://www.meteoblue.com/de/wetter/vorhersage/windmap/europa_brasilien_3664248

    Und Datum vorher einstellen.

    Wieso aber von dort keine Informationen kommen, scheint mir wieder seltsam zu sein.

    Dabei müssten ja die ganzen Institutionen und Messstellen geradezu darauf erpicht sein, genau zu wissen, was da in der Atmosphäre ist…sie wissen schon: wegen de “neuen Waffe” und so. Geheimdienste der Welt müssten Schlangestehen bei den Messtellen. Und vor allem eigene Messungen machen (Partikelmessungen, um herrauszufinden, was da in der Luft sei).

    Die “Informationspolitik” wäre demnach im Westen auch “zweifelhaft”…

  224. #226 Anonymous
    17. August 2019

    Zitat:
    Russlands Präsident Wladimir Putin hatte im März 2018 eine neue Generation von Marschflugkörpern angekündigt. Sie seien bereits 2017 getestet worden, hätten eine unbegrenzte Reichweite und seien unangreifbar für alle existierenden Raketenabwehrsysteme.

    -> Das kann man mehr oder weniger gleich nahezu überall lesen. Eine Aneinanderreihung von Superlativen.

    Aber das ist nach bestem Wissen der Physik nahezu unmöglich, was da angekündfigt wird.

    Ausserdem:
    Ein Atomantrieb existiert nicht und wird auch nicht funktionieren. Atomare Reaktionen lassen es nicht zu, dass man damit bessere Antriebe, als die chemischen Reaktionsporzesse heute als Antrieb leisten, bauen kann.
    Das ist nur Panikmache auf subtiler Art.

    Mit dem Atomthema kann man noch heute allerbestens Panik machen.
    Ausserdem kann man damit politisch wirksam herrausfinden, wie gerade die Lage in der Population ist. Diese Atom-Szenerien sprechen immer auch unser unbewusstes an. Und an der Reaktion der Population merkt man dann, wie es gegenwärtig mit der Szenerie des “unbewussten” gerade steht.

    Im Moment sieht es so aus, als ob sich da nicht viel tut. Niemand beschwehrt sich wirklich, dass mal wieder ein atomarer Unfall geschehen ist.
    Man nimmt es offenbar einfach hin. Scheinbar ist eine
    fortschreitende Resignation gegenüber der Atomprolbematik eingetreten.

    Wenn morgen ein Atomkraftwerk in eine Kernschmelze gehen würde, täte sich kaum jemand effektiv darüber beschwehren wollen. Denn da ist kein Oppositionspotential mehr, dass sich darüber aufregen könnte. Die Menschen sind abgenutzt und verschlissen.

    Wie macht man also Politik über unsere Köpfe hinweg? Man lässt eine Katastrophe nach der anderen geschehen und zermürbt damit das Souzveränitätspotenzial des Volkes.

    Stoisch wird alles hingenommen.

    Die sogenannten “höheren Gewalt” (sogenannter Zufall oder Naturkatastrophen) kann also dazu ausgenutzt werden, dass man ganze Populationen wirksam politisch verschleisst und zermürbt. Übrig bleibt der willenlose Bürger, weil all seine Ideale durch solche Katastrophen unmöglich gemacht wurden.

    Und da haben wir noch gar nicht über die Wirkung der schweren Metalle gesüroichen, die in uns gelangen, wenn sie derart in Mengen freigesetzt werden.

    Und die kontraintuitive Erkenntnis, dass mehr Schwermetalle in gewisser Weise politischen Nutzen erzeugen kann, trägt dazu bei, dass man in eienr zwickmühle steckt, aus der man unter den Umständen gar nicht entkommen kann.

  225. #227 demolog
    17. August 2019

    Wie interessant: Die Plattform interessiert sich nicht mehr für den Absender. Er wird einfach zum “Anonymous” gemacht, wenn man den Namen vergisst einzutragen.

  226. #228 Tobias Cronert
    19. August 2019

    @Dünger: Da kommt es auf die entsprechenden Elemente an. Schwermetalle u.ä. werden sich im Boden ablagern, während Kalium 40 in wasserlöslicher Form überall hin geht.

    Die Idee Flugzeuge und Marschflugkörper mit einem nucularen Antrieb zu versehen gabs schon immer. War halt meistens nicht praktikabel, weil die Dinger zu schwer sind. Aber wenn ich das jemandem zutrauen würde, dann sind es die Russen. Die haben da immer Ideen, die sich die anderen nicht trauen.

    Für eine neutronenquelle hatte ich mich mal mit einem russischen Wissenschaftler über ein Kopnzept unterhalten, wo der dann nur lapidar meinte “Ja, dann haben wir da 50kg Neptunium im subkritischen Bereiche …”

    Woraufhin meine Reaktion war: “50kg Neptunium???? WTF …

  227. #229 demolog
    19. August 2019

    Warum der Entwicklungsversuch des Atomantriebes für Raketenantriebe damals in den USA (Orion-Projekt) aufgegeben wurde, lag vermutlich daran, dass atomare Kettenreaktionen, wie bei der Atombombe, ausserhalb der Erdgravitation nicht so effektiv ablaufen, dass daraus ein effektiver Antrieb entstehen würde.

    Und einen Raketenantrieb für innerhalb der Atmosphäre müsste aus einem Gemisch hochinstabiler Elemente und einem hochabschirmendem Moderator bestehen. Denn sonst würde die Rakete in einem Knall abbrennen und keinen kontinuierlichen Antrieb darstellen können.

    Und ob atomarer Zerfall sich dazu eignet, eine gerichtete Kraft auszuüben, bezweifle ich auch mal so aufs Blaue hinaus, weil mir keine Möglichkeit bekannt ist, die Zerfallsprodukte in eine Richtung fliegen zu lassen.
    Und Fusionsantriebe? Na, solange es noch keine stabile Kernfusion auf der Erde gibt, kann man das auch nicht weiterentwickeln, oder?

    Aber was weiß ich schon.

    Ausserdem:
    Polonium…Polonium…Polonium…210.

    Dazu wollen sie nix sagen? Warum ist auf Wikipedia von Polonium210 die Rede und hier von Uran?

    Wenn ich über Jahrzehnte lang auf einer Fläche belasteten Dünger ausbringe, dann reichert sich das Zeug dort an. Und wenn die Pflanze dann auch noch solche Elemente aufnimmt, dann entsteht das, was wir mit instabilen Elementen nicht wünschen: sie werden “mobil”.

    Ich gehe mal davon aus, dass der Wikipedia-Artikel nicht plausibel ist.
    Schon die Aussage, die Pflanze würde das Schwehrmetall über die Bläteter aufnehmen, ist doch eher hahnebüchener Blödsinn. Jedenfalls, was den hohen Gehalt der Pflanze angeht, der sicher nicht über die Atmosphöäre und die Blätter in die Pflanze kommt.

    Mutmaßlich wird das ganze Zeug nicht auf natürlichen Weg ion die Pflanze kommen, sondern durch vorsättzliche Kontamination.

    Und der russische Wissenschaftler hat sich wohl einen Spaß mit ihnen gemacht. Wahrscheinlich. Sie wissen schon: ehemals “eiserner Vorhang” und so….alles ist so geheimnisvoll und da macht man gerne mal Späße mit leichtgläubigen “Wessis”.

  228. #230 Captain E.
    20. August 2019

    Kann man, abgesehen von Orion, überhaupt einen nuklearen Raketenantrieb bauen ohne zusätzliche Stützmasse?

  229. #231 tomtoo
    20. August 2019

    @Captain E.
    Ein Raketenantrieb wohl nicht. Aber die Gerüchte drehen sich ja um ein cruise missile
    dass innerhalb der Atmosphäre bleibt. Auch wenn die Russen irgentwie schmerzbefreit erscheinen, kann ich mir nicht vorstellen das die einen so heißen Reaktor fahren. Das ganze bleibt für Aussenstehende geheimnissvoll.

  230. #232 tomtoo
    20. August 2019

    @demolog
    Was bitte sollte eine Kettenreaktion mit der Erdgravitation zu tun haben? Meines wissens funktionieren Atomwaffen ganz prima ohne Gravitation.

  231. #233 Tobias Cronert
    20. August 2019

    Ja, wie schon gesagt. Auf Basis einer Radionukligbatterie soll es sein.
    Aber ohne weiter Informationen ist das alles nur Spekulation.

    @Dünger: Ich habe absolut keine Ahnung, wieviel Polonium in Dünger drin ist und wie das biologisch vom Boden und Pflanzen behandelt wird. Ist halt ein Alpha Strahler und daher notorisch schwer zu messen – siehe Litvinienko.

    Das mit dem Neptunium war schon vollkommen erst gemeint von denen. Die haben das vernünftig simuliert und eben auf einer Fachkonferenz präsentiert. Die Zuhörer waren aus ausreichend beeindruckt, es war nur klar, dass das außer Russland niemals jemand bauen würde.

  232. #234 Captain E.
    21. August 2019

    Ich hatte da übersehen, dass es weiter oben um einen Raketenantrieb “innerhalb” der Atmosphäre ging. Das ist natürlich noch einmal etwas anderes.

    Ich formuliere dazu jetzt einmal ein paar Fragen:

    Lässt sich im Weltraum mit einer Radionuklidbatterie ein Ionentriebwerk (Stützmasse: Zumeist Xenon) betreiben? Braucht ein reaktorgestütztes Nukleartriebwerk nicht Wasser oder etwas ähnliches als Stützmasse?

    Zu dem Marschflugkörper: Soweit man hört, ist der Flüssigtreibstoff explodiert. Wenn wir von der Annahme ausgehen, dass dieser nur oder zumindest in erster Linie für den Start verwendet wird, ließe sich mittels Radionuklidbatterie der Flugkörper danach in der Luft halten?

    Und was den Tabak angeht: Könnte die Polonium-Anreicherung in Tabakpflanzen etwas mit dem Edelgas Radon zu tun haben?

  233. #235 demolog
    22. August 2019

    “Aber ohne weiter Informationen ist das alles nur Spekulation”

    Richtig.

    Keine Spekulation soll ja aber das Polonium im Tabak (oder Dünger) sein.
    Das können sie doch leicht mal eben testen, oder? Tabak kostet das Päckchen 5 Euro.

    @ tomtoo
    20. August 2019

    @demolog
    Was bitte sollte eine Kettenreaktion mit der Erdgravitation zu tun haben? Meines wissens funktionieren Atomwaffen ganz prima ohne Gravitation.

    -> Woher wissen sie das?

    Haben sie eine im erdfernen Orbit gezündet?

  234. #236 tomtoo
    23. August 2019

    @demolog
    Muss ich vom Hochhaus springen um zu wissen was passiert ?

    Welchen Einfluss sollte denn die Gravitation auf eine Kernwaffe haben? Ich quetsche eine gewisse Menge Metall mit einer Sprengladung zusammen. Auf dass es möglichst lange überkritisch ist ohne frühzeitig zu verpuffen. Wo sollte da Gravitation eine Rolle spielen?

  235. #237 Tobias Cronert
    23. August 2019

    @ Polonium: Da Po ein reiner Alpha-Strahler ist, ist es recht schwer nachzuweisen. Das war z.B. ein klassisches Problem bei Litvinienko. Mit dem Equipment auf der Arbeit kann ich das. Mit dem Zeug, was ich hier zu Hause habe nicht.

    @ Graviatition: Auswirkugen auf die Explosion einer A-Bombe hat Gravitation nicht, aber doch (teilweise) deutlich auf die Ausbreitung und und das “Fernfeld”. Bei den entsprechenden Simulationsprogrammen kann man mit einem Befehl die Gravitation aus und ein schalten und die Simulationene dann vergleichen. Interessanterweise fallen Neutronen und Protonen im Gravitationsfeld “nach unten”. Je schneller die sind, desto weniger spielt das eine Rolle, aber grundsätzlich ist der Effekt schon vorhanden.

    @Antrieb: Ja grundsätzlich machen Reaktoren und Radionuklidbatterien erst mal nur Wärme. Da muss man dann schon noch was mit machen um mit der Wärme etwas anzutreiben. Wie gesagt gibt es viele Ideen, aber nichts, was sich bislang als wirklich praktikabel durchgesetzt hätte. Ich mein selbst die Radionuklidbatterien in der Raumfahrt sind ja eher die absoluten Exoten.

  236. #238 tomtoo
    23. August 2019

    @Tobias
    “..Interessanterweise fallen Neutronen und Protonen im Gravitationsfeld “nach unten”..”

    Den rahm ich mir ein!
    Physiker Spruch des Jahrhunderts! ; )

  237. #239 tomtoo
    23. August 2019

    @Tobi
    Evtl. fallen auch Äpfel deswegen? ; )

  238. #240 Tobias Cronert
    24. August 2019

    Hm, da muss ich mal diesen komigen Englänger fragen. 😉

    Na bei einer Atombombenexplosion (oder einer Spallations-Neutronenquelle) fliegen eine Menge Teilchen erst mal direkt nach oben in die Atmosphäre. Die Massereichen Teilchen, wie Neutronen und Protonen fallen dann unter der Graviation wieder zurück auf die Erde (wie eine Pistolenkugel), während Gammas in den Weltraum entkommen.

    Also irgendwie hat das immer meiner Intuition von Elementarteilchen widersprochen.

  239. #241 tomtoo
    24. August 2019

    @Tobi
    “..Also irgendwie hat das immer meiner Intuition von Elementarteilchen widersproc hen..”

    Quantenphysik Verseuchung ??!! Da sind die Teilchen mal ganz lieb und folgen der alten Physik…und….und?..auch wieder nicht recht ; )

  240. #242 tomtoo
    24. August 2019

    @Tobi
    Eigentlich faszinierend. Als der Olle mit dem Apfel, seine Gleichungen aufgestellt hatt, hatt er bestimmt an vieles gedacht, aber Neutronen? Aber selbst die Biester folgen(im großen und ganzen)seinen Gleichungen.

  241. #243 Captain E.
    26. August 2019

    Die Frage ist allerdings auch, wie viele dieser Nukleonen zurückfallen und wie viele trotzdem in den Weltraum entkommen.

    Das ist doch alles eine Frage der jeweiligen Energie, oder?

  242. #244 tomtoo
    26. August 2019

    @Captain E.
    Bestimmt eine klasse Frage um arme Physikstudenten zu ärgern.
    Die gebräuchliche A-Bombe aus dem Supermarkt nebenan enthält 10Kg Plutonium. 2% der Plutonium Atome werden gespalten. Die Bombe zündet in 100m Höhe, wieviele Neutronen entkommen der Erdgravitation? ; )

  243. #245 Tobias Cronert
    26. August 2019

    Gute Frage um Physik Studenten zu ärgern? Ja und Nein

    Erst mal ist das ganze recht einfach. Eine Atombombe ist auch “nur” eine Kernspaltung, d.h. die Neutronen haben eine max. Energie von ca. 10MeV. Das entspricht 13000km/s. Die Fluchtgeschwindigkeit der Erde ist 11km/s, also müssten alle entfliehen können.

    Simulationen für die ESS z.B. haben allerdings gezeigt, dass mindestens die Hälfte dennoch zurück fallen. Ein Effekt, der Skyshine genannt wird. Der Atmosphäre sei Dank ?!
    Das ist bei einer Atombombe halt auch so.

    Das kann man halt nicht mehr einfach berechnen, sondern muss man simulieren. Es sei denn man würde evtl. ein sehr vereinfachtes Modell für die Atmosphäre annehmen … hm

  244. #246 tomtoo
    26. August 2019

    @Tobi
    Spannend ist ja das bei dem Effekt in der Wiki ausgerechnet Jülich erwähnt wird.

  245. #247 Captain E.
    26. August 2019

    @Tobias Cronert:

    Gute Frage um Physik Studenten zu ärgern? Ja und Nein

    Erst mal ist das ganze recht einfach. Eine Atombombe ist auch “nur” eine Kernspaltung, d.h. die Neutronen haben eine max. Energie von ca. 10MeV. Das entspricht 13000km/s. Die Fluchtgeschwindigkeit der Erde ist 11km/s, also müssten alle entfliehen können.

    Simulationen für die ESS z.B. haben allerdings gezeigt, dass mindestens die Hälfte dennoch zurück fallen. Ein Effekt, der Skyshine genannt wird. Der Atmosphäre sei Dank ?!
    Das ist bei einer Atombombe halt auch so.

    Das kann man halt nicht mehr einfach berechnen, sondern muss man simulieren. Es sei denn man würde evtl. ein sehr vereinfachtes Modell für die Atmosphäre annehmen … hm

    Tja, irgendwie stehe ich jetzt auf dem Schlauch!

    Ist das nicht ohnehin zu erwarten, dass die Hälfte der bei einer Kernwaffendetonation freiwerdenden Nukleonen Richtung Erdboden fliegen? Die dürften doch schließlich gleichmäßig in alle Richtungen abgestrahlt werden, nicht wahr? Also auch zum Erdboden hin?

  246. #248 tomtoo
    26. August 2019

    @Captain E.
    Stell dich nicht so an. Ist ja klar das die hälfte derer gemeint ist die nicht eh direkt Richtung Erde fliegen.

  247. #249 Captain E.
    26. August 2019

    @tomtoo:

    Ok, wenn das so zu verstehen war…

    Da man meines Wissens nach nie eine nukleare Detonation über einem atmosphärelosen Himmelskörper durchgeführt hat, ist es also nicht ganz klar, ob auf der Erde die Gravitation oder die Reaktionen mit den Molekülen der Lufthülle für das “Umdrehen” verantwortlich sind?

    Indes ergeben sich da weitere Fragen: Wenn es zu “Skyshine” aufgrund der Reaktion mit der Luft kommt wie etwa in Jülich beim dortige Kugelhaufenreaktor, ist dann eine einmalige Reflexion um ca. 180° das Ereignis mit der höchsten Wahrscheinlichkeit? Und was ist mit einer Luftdetonation? Müsste nicht auch ein Teil auf dem Weg Richtung Erdboden reflektiert werden? (Vermutlich reicht danach die Energie für ein Entweichen nicht mehr aus). Und wie war das beim Test Starfish Prime, beim dem der Sprengsatz in 400 km Höhe detoniert ist? Besonders dicht ist die Atmosphäre dort oben nun nicht mehr.

  248. #250 tomtoo
    27. August 2019

    Auf jeden Fall ist mir jetzt klar,.das beim Weggesprenkten Dach in Tschernobyl der unsicherste Platz nicht unbeding der direkt an der Mauer war. ; )

  249. #251 Tobias Cronert
    27. August 2019

    Äh, ja sorry. Ich meinte natürlich 50% der Neutronen, die “nach oben” fliegen fallen auch wieder zurück.

    Wie oben schon angedeutet ist das ganze für mich immer etwas contraintuitiv. Bei schnellen Protonen oder Neutronen gehe ich eigentlich davon aus, dass der Einfluss der Graviation vernachlässigt werden kann. Einfach weil die so schnell sind, dass die mikroskopische Abweichung von einer gerade Flugbahn (von sagen wir mal Gamma-Quanten) nicht ins Gewicht fällt.

    Aber zusammen mit der Atmosphäre reicht die schwache Graviation aus um echt viele Teilchen wieder zu Boden zu bringen. Wie das jetzt für Atombomben aussieht, die man in verschiedenen Höhen zündet kann ich nicht sagen, denn ich kenne nur die Simulationen für Neutronenquellen und die sind alle auf dem Boden.

    Neutronen verlieren relativ schnell viel Energie. Bei jedem kinetischen Stoß in der Atmosphäre ca. die Hälfte. Für Protonen und geladenen Teilchen geht es sogar noch wesenlich schneller. Daher glaube ich, dass die Atmosphäre jetzt nicht soooo dicht sein muss, aber das ist echt nur geraten. Da habe ich keine Zahlen um das zu unterstützen.

  250. #252 zimtspinne
    28. August 2019

    Bei diesen Erläuterungen….
    Es gibt Menschen mit Sinn für Totes und es gibt Menschen für Lebendiges (die Menschen mit Sinn für Untotes lass ich mal der Einfachheit halber weg).

    Ich kann mir solche Geschichten einfach nicht vorstellen. Erinnere mich noch, wie ich versucht hab, die Unterschiede zwischen Atom und Molekül zu verstehen…. verstehen tu ich es ja, aber mit der Vorstellungskraft des Ganzen hapert es dann schon wieder.

    Wobei Chemiekrams noch eher ok ist als Physikkrams. Das ist ja ein Bindeglied zwischen Nichtleben und Leben, das geht dann schon eher in meinen Kopf rein.
    Außerdem ist es spannend zu beobachten, wie was miteinander/aufeinander reagiert. Fast wie bei menschlichem Leben *g*

    die Katze als solches könnte man ja auch als Versuch werten, Physikkrams mit Lebenskrams zusammenzubringen….. ich hoffe mal, da steckte kein Katzenhass dahinter.

  251. #253 tomtoo
    29. August 2019

    @Zimti
    Eigentlich wäre doch Physik klasse zu verstehen, gäbe es da nicht die Katzen..also die vom Schrödinger. Die machen Physik irgentwie sau schwer zu verstehen. Als Hundemensch könnte man jetzt motzen und sagen, Katzen raus aus der Physik, ihr seid einfach shyse. Denk an das Berühmte physikalische Problem einer Katze einen Marmeladentoast auf den Rücken zu binden und sie vom Hochhaus zu schmeisen. Welche Seite landet jetzt unten? Letzlich ist sowas theoretisch kaum zu lösen, und es wird noch vieler Versuche bedürfen solch Fundamentale Fragen zu beantworten. ; )

  252. #254 Peter
    31. August 2019

    Seit einer Weile kursieren die Namen der freigesetzten Isotope (beim Atomunfall am Weißen Meer): Strontium, Barium sowie Lanthan. Was lässt sich hieraus – gespiegelt im Prisma der Physik – für mich als Laien erschließen? Ist dies nun besonders gefährlich oder eben nicht? Oder lässt sich diesbezüglich nichts Konkretes sagen?

  253. #255 Captain E.
    2. September 2019

    Irre ich mich, oder deutet das auf Kernspaltung hin? Aber fehlen dann nicht noch Iod und Caesium?

  254. #256 tomtoo
    2. September 2019
  255. #257 Captain E.
    2. September 2019

    Habt ihr die weiteren Links gesehen? Ein Toter soll ein schwer verstrahlter Taucher gewesen sein. Ebenso hat es Nuklearexperten erwischt, und das Gerät vor Ort, darunter zwei schwer beschädigte Pontons, deuten auf einen Bergungsversuch unter Wasser hin.

    Ein FK-Start war dagegen wohl nicht vorgesehen, denn es gab keine Sperrung des Luftraumes.

  256. #258 tomtoo
    2. September 2019

    @Captain E.
    Jo, gelesen. Aber so richtig schlau werde ich aus dem Allem nicht.

  257. #259 Captain E.
    3. September 2019

    @tomtoo:

    Wusstest du, dass die USA bereits in den 50er Jahren über nukleare Antriebe nachgedacht haben? Da gab es seinerzeit das NEPA-Programm (Nuclear Energy for the Propulsion of Aircraft).

    https://de.wikipedia.org/wiki/Fl%C3%BCssigsalzreaktor#Aircraft_Reactor_Experiment

    Die Dinger wurden offensichtlich mit Natrium-, Zirconium- und Uranfluoriden befüllt. Da frage ich mich (und euch) doch: Wie wahrscheinlich ist es, dass bei diesem Unfall gerade ein ähnlicher Reaktor befüllt wurde, als irgendetwas schief gegangen ist? Die nächste Frage wäre nur: Hat dieser hypothetische Reaktor ungeplant Kritikalität erreicht oder lag das Problem tatsächlich, wie offiziell verlautbart, beim konventionellen Flüssigtreibstoffantrieb? Das Vorhandensein von Spaltprodukten weist zumindest darauf hin, dass da ein Reaktor kritisch geworden und danach leck geschlagen ist, oder? Und nach erfolgreich abgeschlossener Befüllung wären doch die Nuklearexperten bereits wieder in sicherer Entfernung gewesen?

  258. #260 tomtoo
    3. September 2019

    @Captain E.
    Ja, aber ich hab ja schon gesagt, es ist für mich nur schwer vorzustellen das die Russen tatsächlich an sowas verrückt heißem arbeiten. Sowas macht doch nur Sinn wenn es dann auch wirklich Nuklear bestückt ist. Das Ziel ist dann eh verseucht. Was will man mit so einer Doomsday Maschine? Evtl. interessant für dich?

    https://www.google.de/url?sa=i&source=web&cd=&ved=2ahUKEwia972yj7TkAhVDEVAKHeDOCHEQzPwBegQIARAC&url=https%3A%2F%2Farstechnica.com%2Fscience%2F2018%2F03%2Fbest-bad-idea-ever-why-putins-nuclear-powered-missile-is-possible-and-awful%2F3%2F&psig=AOvVaw1O_WHpySejBfnD9dlIkLZZ&ust=1567581796761019

  259. #261 Tobias Cronert
    3. September 2019

    Also wenn Spaltprodukte nachgewiesen worden sind, dann wäre es eben keine Radionuklidbatterie, sondern eine kritische Anordnung.
    Wobei man sich da im Hinterkopf behalten muss, dass die Russen da auch gerne mal kreativ werden und reine Plutonium oder gar Neptunium kritische Anordnungen …. zumindest hypothetisch in den Raum werfen.

    Da muss ich noch mal schwer auf die Suche nach verlässlichen Informationen gehen.

  260. #262 Captain E.
    3. September 2019

    @tomtoo:

    Mir stellt sich eher die Sinnfrage. Wozu braucht man einen nuklear angetriebenen Flugkörper?

    Taktisch ist das Ding ein Alptraum. Allerhöchstens könnte man über seinen Einsatz nachdenken, wenn es denn auch die Fähigkeit hätte, wieder zu landen. Denn ansonsten hieße, falls sich die aktuelle Krise, derentwegen er gestartet wurde, doch wieder beruhigen sollte: “Beschießen wir unseren Gegner jetzt doch noch, wo das Gerät schon einmal in der Luft ist, oder in welchem Meer versenken wir es gleich, samt Antrieb und Sprengkopf?”

    Die Betankung dürfte auch nicht ohne sein, und vermutlich wäre sein Einsatz daher strategisch zu sehen. Wenn es sowieso gerade überall knallt, schickt man das Ding auch noch los, damit es sich ein paar noch heil gebliebene Ziele beim Gegner sucht und die auch noch atomisiert. Einen entscheidenden Vorteil gegenüber Interkontinentalraketen sehe ich da aber nicht, und deren Entwicklung hat die nuklear angetriebenen Flugzeuge und Marschflugkörper schon vor ihrer Serienreife obsolet gemacht. Man frage sich überhaupt, ob man in einem gerade stattfindenden weltweiten Atomkrieg noch die Experten zusammenbekommt, um den Reaktor zu befüllen. Oder will man die auch dauerhaft schichtweise verbunkern?

    Mir erscheint dieser Marschflugkörper eher wie ein Ausdruck der russischen Inkompetenz. Laut Putin sollen sie die US-Raketenabwehr aushebeln. Fein, nur ist diese vor allem gegen schwache Gegner entwickelt worden. Sollte Russland so viele nuklear bestückte Raketen und nukleare Fliegerbomben samt Trägerflugzeugen besitzen wie es von sich selbst behauptet, könnte die NATO diese im Leben nicht abwehren.

    Sind diese “Wunderwaffen” also vielleicht nichts anderes als das heimliche Eingeständnis Russlands, dass der allergrößte Teil ihrer nuklearen Bewaffnung aus nicht einsatzfähigem Schrott besteht?

  261. #263 tomtoo
    3. September 2019

    @Captain E.
    Naja evtl. ist es mit den Wunderwaffen so wie es schon mal war, und sie dienen am ehesten den Innenpolitischen wundern? ; )

  262. #264 Tobias Cronert
    3. September 2019

    Nuculartechnik ist eine der weigen technischen Zweige, in denen Russland noch eine Vorreiterrolle in der Welt hat.

    Wenn man einen Hammer hat, dann sucht man sich eben ein Problem, was so aussieht, wie ein Nagel?

  263. #265 Captain E.
    4. September 2019

    @Tobias Cronert:

    Was genau soll das denn bedeuten, diese “Vorreiterrolle”?

  264. #266 Tobias Cronert
    4. September 2019

    Naja, OK, Vorreiterrolle ist wahrscheinlich das falsche Wort, weil sie einen Evolution in eine bestimmte Richtung impliziert.

    Was ich damit sagen wollte, ist, dass die Russen in den meisten Nuklearen Themen besser sind, als als der Rest der Welt. Das ist eine ihrer letzten verbleibenden großen Stärken (wie sonst z.B. die Raumfahrt) und daher ist es nicht verwunderlich, wenn sie diese Stärke ausspielen wollen.

  265. #267 Captain E.
    4. September 2019

    @Tobias Cronert:

    Ist das deine Meinung als Experte?

    Ich finde nämlich deinen Vergleich mit der russischen Raumfahrt interessant. Innovativ ist in der nämlich nur die PowerPoint-Abteilung (oder was auch immer die Russen benutzen). Das, was wirklich fliegt, ist ein klein wenig weiter entwickelter alter Schrabbel. Das “Verdienst” der russischen Raumfahrt wäre höchstens, dass man immer weiter gemacht hat. Die US-Raumfahrt steckt bekanntlich gerade mal wieder in einer “Gap”, was den bemannten Teil angeht.

    Was den unbemannten Teil angeht: Wann hat irgendetwas russisches/sowjetisches das letzte Mal die Erdumlaufbahn verlassen? Das muss schon ziemlich lange her sein. Zur Venus? In den 80ern? Oder zum Mars etwas später (zwei Fehlschläge)? Man könnte den ExoMars Trace Gas Orbiter nennen, aber da war Roskomos für ein paar Instrumente und die Trägerrakete verantwortlich, der Rest stammte aber von der ESA.

  266. #268 Tobias Cronert
    4. September 2019

    Ich bin jetzt echt nicht up to date, was Raumfahrt angeht, aber meine Meinung deckt sich schon sehr stark mit dem, was du beschrieben hast.

    Die Russen haben richtig tolle Experten aus den 80er Jahren und früher, die die aktuelle Generation auf einem sehr hohen Niveau ausgebildet haben. Sie haben viele tolle Maschinen, die zwar alt sind, aber weltweit noch ihresgleichen suchen (z.B. der IBR 2 Reaktor in Dubna).

    Eigentlich haben sie alle Voraussetzungen (von Wissenschaftsseite aus) um auch die zukünftige nächste Generation zu bauen, werden aber leider von politischen … sagen wir Schwierigkeiten zurückgehalten. Wie z.B. der PIK-Reaktor, der zwar nominell doppelt so viel Power hat, wie alle anderen Neutronenquellen weltweil, diese aber nicht nutzen kann, weil die versprochene europäische Unterstzützung wegen der Krim-Krise auf Eis liegt. Daher dümpelt die nominell weltweit stärkste Quelle eher irgendwo auf Platz 5-8 herum, was Performance angeht … wenn überhaupt.

  267. #269 demolog
    4. September 2019

    https://www.nzz.ch/international/atomunfall-in-russland-missglueckte-bergungsaktion-als-erklaerung-ld.1505646

    -> NZZ als einzige Quelle?

    Mich bestätigt, was man so zu lesen bekommt.
    Es gibt ein Problem bei der Nutzung von atomarer Energieträger und die effiziente Funktion in höheren Atmosphärenschichten oder gar ausserhalb des Erdsystems.
    Warum auch hier jeder davon ausgeht, dass es sich um eine überkritische Menge an radioaktiven Elementmischung handeln soll, wäre ja fast ein Beweis dafür, dass die Notwendigkeit deswegen gegeben ist, weil die auf der Erdoberfläche konfigurierten Geräte “auf Kannte” oder eben “überkritisch” konfiguriert werden müssen, damit sie ausserhalb des Erdeinflusses überhaupt noch funktionieren. Flüssigsalz liesse sich da ja theoretisch leicht mischen, um die Kritikalität zu konfigurieren.

    Die Starfish-Tests scheinen mir anhand der Videos und Infos auch eher ineffizient explodiert zu sein. Aber das ist kaum hinreichend sicher zu erklären, da es absolut keine Informationen zu den Filmsituationen gibt (wie weit entfernt, welcher Winkel auf die Explosion).

    Müsste aber nicht eine atomare Explosion den ganzen Himmel ausfüllen, bei den Druckverhältnissen da oben (400 Km)?

    Irritierend ist, dass der Taucher offenbar nach der Explosion noch gelebt hat und ins Krankenhaus gefahren wurde.
    Obwohl der mutmaßlich der Rakete (oder was immer das war) am nächsten war. Die auf der Plattform befindlichen Leute aber sofort gestorben sein sollen?

    Eine Interkontinentalrakete auf Ramjet-Basis hätte das Problem mit der erdsystemabhängigen Kritikalität ja nicht, wenn sie gar nicht die Erde verlassen soll.

    Ich erinnere mich noch an Reagons Weltraum-Abwehrsystem.
    Wenn man also eine Rakete im All dramatisch überkritisch konfigurieren würde, könnte man vielleicht die Kritikalität der Konfiguration eine Weile stabil halten, bis sie auf die Erdoberfläche ankommt. Ebenso, wie es bei Meteoriten oder anderen einschlagenden Objekten der Fall ist.
    Das aber wäre die falsche technologische Entwicklungsrichtung.

  268. #270 Beobachter
    27. September 2019

    Kürzlich hat mal ein Kommentator hier im Blog erwähnt, es hätte in der ehemaligen DDR eine Meldepflicht für Krebserkrankungen gegeben und auch ein entsprechendes Melderegister.
    Und er fragte, ob das jemals ausgewertet worden wäre.
    Er hat darauf, wenn ich mich recht erinnere, keine Antwort bekommen.
    Weißt du da etwas drüber, Tobias?
    Oder ein Mitleser?

    Leider finde ich den entsprechenden Kommentar auf die Schnelle jetzt nicht mehr – muss aber erst vor ein paar Tagen gewesen sein.

  269. #271 RainerO
    27. September 2019

    Leider finde ich den entsprechenden Kommentar auf die Schnelle jetzt nicht mehr…

    Es war hier.
    Das Krebsregister der DDR wurde offenbar bearbeitet, gefunden habe ich z.B. das hier”.
    Zu diesem Thema wäre sicher auch Joseph Kuhn eine gute Anlaufstation.

  270. #272 RainerO
    27. September 2019

    Oops, Link verhunzt, sorry: Das ist der richtige Link zum gesuchten Kommentar”.

  271. #273 Beobachter
    27. September 2019

    @ RainerO:

    Danke für deine Auskünfte – es geht also doch auch konstruktiv.
    Warum nicht immer so … ?

    Jedenfalls sind diese Register eine gute und große Datenbasis – da ließe sich bestimmt viel daraus machen.
    Vielleicht hat man es auch schon getan, nur weshalb ist es dann offenbar kaum bekannt?
    Eine gute Anregung, sich näher damit zu beschäftigen.

  272. #274 Captain E.
    27. September 2019

    @Beobachter:

    Danke für deine Auskünfte – es geht also doch auch konstruktiv.
    Warum nicht immer so … ?

    […]

    Nun, diese eine Mal hast du tatsächlich eine ganz normale Frage gestellt und natürlich eine Antwort bekommen. Leider kommst du sonst fast immer völlig überzogen rüber. Wie wir dir schon einmal gesagt haben: “Wie man in den Wald hinein ruft, so schallt es heraus.” Kommunizier mit uns zivilisiert und vernünftig, und wir tun das auch mit dir. Du merkst ja selber gerade den Unterschied. Es geht also!

  273. #275 Beobachter
    27. September 2019

    @ Captain E.:

    Lass bitte zumindest hier dein Geschwätz !
    Und mich in Ruhe !

  274. #276 Captain E.
    27. September 2019

    @Beobachter:

    Lass bitte zumindest hier dein Geschwätz !
    Und mich in Ruhe !

    Ich sehe schon: Deine zivilisierte Phase war kurz – und ist schon wieder vorüber. Das finde ich sehr schade.

    Aber ich erkläre es dir gerne noch einmal. Wenn du mit uns ordentlich umgehst, tun wir das auch mit dir. Sprühst du dagegen aus jedem Satz Gift und Galle wie du es gerade eben wieder getan hat, dann wird unser Umgangston mit dir ganz folgerichtig auch härter.

    Also, meine Liebe: Wie hart möchtest du es denn haben? Du bestimmst ganz alleine den Grad an Härte und Schärfe im Umgangston.

    Übrigens, denk daran: Hier bitte nur Fragen stellen. Das entspricht dem ausdrücklichen Wunsch von Tobias.

  275. #277 Beobachter
    27. September 2019

    Nachtrag:

    Und warum hat Kommentator “Leser” (schon am 20. Sept. !) auf seine “ganz normale Frage” von euch keine Antwort bekommen?
    Weil es euch gar nicht um die Sache geht und ihr viel zu sehr damit beschäftigt seid, “Gegner” (die ihr euch nach Bedarf ausguckt) oder jemanden, der nicht eurer Meinung ist, persönlich anzugreifen und zu beleidigen.

    Und wenn einer von euch Pöblern das Wort “zivilisiert” in den Mund nimmt und gar für sich beansprucht, ist das der GWAZ (Größte Witz Aller Zeiten).
    So, das musste sein.

    Ansonsten gilt (s. o.):

    Lass bitte zumindest hier dein Geschwätz !
    Und mich in Ruhe !

  276. #278 RainerO
    27. September 2019

    @ Beobachter
    Ich habe die Frage nach der Auswertung des Krebsregisters der DDR bei Joseph Kuhn in einem passenden Thread gestellt.
    Auch wenn du aus in #275 und #277 ersichtlichen Gründen dort nicht mehr schreiben kannst, lesen geht ja noch.

  277. #279 Beobachter
    27. September 2019

    @ RainerO, # 278:

    Ja, prima – soll er als Gesundheitsdaten-Fachmann da mal weitermachen.

    OT:
    Dass und weshalb ich bei JK gesperrt bin, ist tatsächlich dort ersichtlich – nämlich weil ich gut begründeter, anderer Meinung war.
    Wer so mit “unbequemen” Kommentatoren umgeht, hat keine fruchtbare Debatte zur Sache in seinem Blog verdient.
    Insofern bin ich sogar sehr froh drüber, dass ich dort “raus” bin und auch nicht mehr in “Kommentar-Versuchung” geraten kann … 🙂

  278. #280 Captain E.
    27. September 2019

    @Beobachter:

    Nachtrag:

    Und warum hat Kommentator “Leser” (schon am 20. Sept. !) auf seine “ganz normale Frage” von euch keine Antwort bekommen?

    Nun, wir sind hier nicht die Hausherren und springen höchstens einmal ein, und vermutlich fühlte sich niemand kompetent genug für eine Antwort. Der Gesundheitszustand von Tobias ist dir bekannt?

    Weil es euch gar nicht um die Sache geht und ihr viel zu sehr damit beschäftigt seid, “Gegner” (die ihr euch nach Bedarf ausguckt) oder jemanden, der nicht eurer Meinung ist, persönlich anzugreifen und zu beleidigen.

    Ist deine Ansicht, uns ginge es nicht um die Sache, falsch?

    Ist deine Ansicht, wir könnten keine anderen Meinungen akzeptieren, falsch?

    Ist deine Ansicht, wir griffen andere Kommentatoren an und beleidigten sie, falsch?

    Und wenn einer von euch Pöblern das Wort “zivilisiert” in den Mund nimmt und gar für sich beansprucht, ist das der GWAZ (Größte Witz Aller Zeiten).

    Trifft es zu, das auf diesem Blog zurzeit der Kommentator “Beobachter” dadurch auffällt, dass er oder sie regelmäßig über andere Kommentatoren herzieht und sie anpöbelt?

    So, das musste sein.

    Ansonsten gilt (s. o.):

    Lass bitte zumindest hier dein Geschwätz !
    Und mich in Ruhe !

    Darf ich deine Anweisung als Frage interpretieren?

    Dann erfolgt hiermit die Antwort: Benimm dich anständig, und du kannst mit uns ganz normal diskutieren.

  279. #281 zimtspinne
    27. September 2019

    Sorry, aber ich hab eben einem Lachanfall nachgeben müssen, bei den Anweisungen an den Captain 😀
    Es hat ja auch was von comedy…. das müsst ihr zugeben, bei aller Brisanz und Dramatik.

    Nun hab ich Herrin Beobachter vor meinen gedanklichen Augen, wie sie nudelholz- und pfannenschwingend mit gnadenloser Anweiser-Hand und Stimme für Disziplin und Ordnung sorgt.

  280. #282 Beobachter
    27. September 2019

    Ich habe in # 270 eine Frage gestellt, habe dann eine Antwort erhalten und mich dann bedankt (# 273).
    Punkt. Und gut ist es.

    Warum lasst ihr mich dann nicht in Ruhe?
    Das ist ja schon krankhafte Streitsucht bei euch und/oder ein pathologisches Mobbingbedürfnis, das ständig befriedigt werden muss, oder tödliche Langeweile oder weiß Gott was …

  281. #283 Captain E.
    27. September 2019

    @Beobachter:

    Warum lasst ihr mich dann nicht in Ruhe?

    Das ist ganz einfach zu beantworten: Du bist es, der/die uns immer angreift.

    Das ist ja schon krankhafte Streitsucht bei euch und/oder ein pathologisches Mobbingbedürfnis, das ständig befriedigt werden muss, oder tödliche Langeweile oder weiß Gott was …

    Nein, das ist reiner Selbstbehauptungswille. Wer so angegangen wird wie wir immer von dir angegangen werden, der kuscht oder der wehrt sich. Was sollten wir denn deiner Meinung nach sonst tun?

    Gegenfrage: Was treibt dich denn dazu, ständig mit einer solchen Aggressivität auf alles und jeden loszugehen?

  282. #284 zimtspinne
    27. September 2019

    Ach komm, sei doch nicht so krätzig und unknusper an einem Wochenendanfang, Beobachter….

    Bade doch nicht schon wieder in Unschuld, wo du es warst, die unbedingt noch nachtreten musste und wadenbeißen.

  283. #285 RainerO
    27. September 2019

    @ Beobachter
    Ich wollte mich eigentlich nicht am neuerlichen Gemetzel beteiligen, aber das…

    mich dann bedankt (# 273).
    Punkt. Und gut ist es.

    … muss ich kommentieren.
    Ja, du hast dich bedankt, aber du konntest natürlich trotzdem nicht deine Füße stillhalten und musstest unbedingt sticheln. Was hast du daraufhin erwartet?

    Ich habe übrigens lange überlegt, ob ich überhaupt auf deine Frage in #270 antworten soll, weil ich als Antwort ein hingerotztes “Googeln kann ich selber” erwartet habe. Ich wurde leider nur teilweise eines Besseren belehrt.

    Joseph Kuhn hat übrigens bereits geantwortet.

  284. #286 Tobias Cronert
    27. September 2019

    Hallo Leute,

    ich bin gerade leider nicht fit genug um vernünftig zu diskutieren, deshalb nur mal kurz und knapp:

    Ihr könnt euch gerne in den anderen Artikeln angiften und die Kommentarzeilen vollschreiben. Aber bitte nicht hier in dem “Fragen an den Autor”-Artikel. Hier sind die Leser darauf angewiesen, dass man in den Kommentaren halbwegs vernünftig lesen kann. Daher werde ich in Zukunft alle Kommentare hier löschen, die keine Fragen oder knappe Antworten sind.

    Danke

  285. #287 Miri
    1. Oktober 2019

    Ich hab da nochmal eine Frage zu Radon.
    Neulich las ich, dass sich das in innenräumen im Hausstaub und auch an Gegenständen usw. anlagern kann.
    Aber wie muß ich mir das dann vorstellen ? Sind dann alle meine Gegenstände kontaminiert und geben Strahlung ab ?
    Meine Waschmaschine zB. steht im Keller und dort trockne ich immer meine Wäsche..wenn sich das dann da auch anlagern kann dann hätte man das ja auf der Kleidung und überall und ständig an-und um sich ?
    Vorausgesetzt natürlich in meinem Keller gibt es eine erhöhte Konzentration davon.
    Aber wenn das so wäre kann es ja so schlimm auch nicht sein, denn dann ist man dem ja ohnehin immer ausgesetzt ?
    Oder verflüchtigt sich das Ganze dann trotzdem recht schnell bzw. was haben denn die Radonterfallsprodukte für eine Halbwertszeit ?
    Lg

  286. #288 Miri
    2. Oktober 2019

    Ist zwar keine Frage oder knappe Antwort aber an der Stelle wünsche ich Dir Tobias, dass es Dir schnell wieder besser geht und einfach alles Gute für Dich !!
    LG

  287. #289 Tobias Cronert
    2. Oktober 2019

    Ui Danke.

    Radon ist ein Gas und schwerer als Luft. Das heißt in geschlossenen Räumen lagert es sich am Boden in Senken und dergleichen an.
    Erfahrungsgemäß reicht hier aber Stoßlüften durchaus aus um in Regelmäßigen Abständen das Radon loszuwerden.

    Kontamination über das Radon selber hinaus kann man eigentlich vernachlässigen. Da ist das Radon rel. dankbar.

  288. #290 Tuxic
    Nähe Hannover
    10. November 2019

    Hallo Tobias,

    vielen Dank für Deine Artikel zum Thema. Neben den vielen Sachinformationen gefällt mir der Grundtenor von
    “Don’t panic!” sehr…gerade in diesem Themenbereich ist das sehr erholsam 🙂
    Vielleicht ist dieser Detektor für Dich interessant für ein Review (den gibt es als Bausatz und als Fertiggerät) — er enstand als eine Art “Volksdetektor” im Zuge der Fukushima-Katastrophe:
    Hersteller:
    https://www.netiodev.com/
    Forum:
    https://www.netiodev.com/bb/
    Der Vertrieb erfolgt über EBay.
    Ich bin weder verwandt noch verschwägert mit dem Hersteller sondern nur ein “Happy customer”.
    Gruß
    Tuxic

    Die beiden dort

  289. #291 Tobias Cronert
    10. November 2019

    Hallo Tuxic,

    es freu mich sehr, dass dir mein Geschreibsel hier gefällt.
    Die interaktive Strahlung-Karte von Japan und im Allgmeinen alles rund um http://www.safecast.org steht definitiv noch ganz hoch auf meiner 2do Liste. Unter anderem, da ich auch durchaus eine gewisse Affinität (und Verbindung) zu Japan habe.

    Da passt netiodev natürlich super rein. Vielen Dank für den Link, den ich so auch noch nicht kannte.
    Momentan wird es leider erst mal keine Reviews oder Test geben, da ich dahingehend noch recht stark eingeschränkt bin. Aber das wird sich sicher ändern und dann steht der Detektor ganz oben auf der Liste.

    vielen Dank und liebe Grüße
    Tobi

  290. #292 Hans
    20. Januar 2020

    So ruhig hier. Wie geht es dir?

  291. #293 Tobias Cronert
    22. Januar 2020

    Okok, hier ein aktuelles Update *g*
    https://scienceblogs.de/nucular/2020/01/22/strahlenphysiker-hat-jetzt-leukaemie-blog-36-tag-445/

    Na, mal ersnthaft. Bei mir ist momentan recht viel los und da komme ich kaum zum Schreiben. Sobald ich wieder im Krankenhaus eingesperrt bin wird es da wahrscheinlich besser werden. Dann wird mir erfahrungsgemäß schnell langweilig, wovon Scienceblogs dann profitieren kann 😉

  292. #294 Hans
    22. Januar 2020

    Danke, das ging ja schnell. 🙂 Freut mich wieder von dir zu lesen, auch wenn der Inhalt nicht besonders erfreulich ist.
    Alles erdenklich Gute!

  293. #295 Sepp
    12. April 2020

    Guten Tag Herr Cronert,

    ich habe eine Frage bzgl der Verarbeitung von durch die Katastrophe von Tschernobyl verstrahltem Holz. Behaupten wir z. B. einmal, dass das Papier eines Buches aus Holz gewonnen wurde, welches wiederum aus belasteten polnischen Wäldern stammt. Bestände hier eine Gefahr für den Endverbraucher?

    Ihnen wünsche ich gesundheitlich weiterhin alles Gute, lassen Sie sich nicht unterkriegen.

  294. #296 Captain E
    14. April 2020

    Vermutlich nur, wenn man das Papier mit Wasser aus dem Karatschai-See herstellte. Aber der ist verlandet und Großteils verfüllt, und liegt ohnehin in Russland.

  295. #297 Tobias Cronert
    16. April 2020

    Nein, bei so Sachen, wie Büchern und anderen festen Gegenständen, die nicht gegessen oder eingeatmet werden sind harmlos.

    Wahrscheinlich wäre ein Buch aus Bayrischen Wädern wahrscheinlich sogar schlimmer belastet, als ein Polnisches … aber das kommt sehr stark darauf an, wo der Baum gestanden hat und wie alt er ist.

  296. #298 Claudia
    17. April 2020

    Gibt es Tritiumhältiges Wasser bzw. was passiert mit Tritium im Wasser? Kann Tritium in Bäumen enthalten sein? Ist es dort meßbar?
    Danke für eine Antwort
    Please notify me of your answer.
    CM

  297. #299 Captain E.
    17. April 2020

    Im Vorgriff auf den Hausherrn:

    Ja, es gibt tritiumhaltiges Wasser. Genau gesagt, ist es sogar ziemlich schwierig, tritiumfreies Wasser herzustellen!

    Dieses Tritium zerfällt über Betazerfall zu Helium (He-3) und strahlt dabei Beta- und vermutlich Gammastrahlung ab. Die Halbwertszeit beträgt ungefähr 12 Jahre. Und natürlich ist Tritium in Bäumen enthalten. Das mit der Messbarkeit dürfte etwas schwieriger sein, da es naturgemäß von den anderen Wasserstoffisotopen nur schwer zu unterscheiden ist. Außerdem ist der Tritiumanteil in normalem Wasser ziemlich niedrig.

    Aber natürlich gilt: der natürliche Tritiumgehalt ist dermaßen niedrig, dass er völlig unbedenklich ist. Wer davon krank werden will, muss es schon so machen wie die Leuchtzeigermalerinnen, die eine tritiumhaltige Farbe auf Uhrenzeige aufgetragen haben und dabei immer die Pinsel ablecken mussten. Das war tatsächlich nicht sehr gesund.

  298. #300 Tobias Cronert
    17. April 2020

    Ja, was der Captain sagt.

    tritium ist nicht nur in Wasser, sondern auch in allem, was Wasserstoff enthält. D.h. alles, was Kohlenhydrate enthält, also Brot, Fleisch etc. pp.

    Also kurz gesagt alles, was lebt, hat Tritium.

    Grundsätzlich kann man das auch messen. Es gibt mehrere Techniken, von denen die Flüssigszintillation https://scienceblogs.de/nucular/2015/02/17/tritium-was-sie-schon-immer-wissen-wollten/ https://de.wikipedia.org/wiki/Szintillationsz%C3%A4hler#Fl.C3.BCssigszintillationsz.C3.A4hler eine übliche und oft angewendete.

    Aber in Büchern und ähnlichen Dingen macht das C14 viel Mehr Strahlung, als Tritium.

  299. #301 Sepp
    19. April 2020

    Danke an Herrn Cronert und Captain E bezüglich meiner Frage (#295).
    Sie sagten, dass Radioaktivität “in” festen Gegenständen harmlos ist. Soweit ist das für mich verständlich. Wie würde es sich denn verhalten, wenn ich z. B. ein Buch, welches aus belastetem Holz herstellt wurde, anfasse und mir danach mit den Fingern ins Gesicht/den Mund fasse?

  300. #302 Tobias Cronert
    20. April 2020

    Also diese Menge (Dosis) in festen Gegenständen ist harmlos, solange man sie eben nicht aufnimmt (bei Nahrungsmitteln sieht das schon wieder sehr anders aus).

    Das würde dann auch für das Buch gelten- Wenn man es nur anfasst und die Finger ableckt, dann ist es harmlos. Wenn man es in den Körper aufnehmen würde (sprich der unwahrscheinliche Fall, dass man eine Seite essen würde (Kinder z.B.) dann sähe es etwas anders aus.

    Aber im ganz konkreten Fall ist die Radioaktivität so gering, das selbst das Essen von einer Seite oder mehr völlig harmlos wäre.

  301. #303 Elli
    Bitz
    1. Mai 2020

    Hallo,

    ich habe auch eine Frage. Neulich hab ich in einem Artikel gelesen, dass Fliesen im Bad radioaktive Stahlung abgeben können. Das Kinderzimmer befindet sich in unserer Wohnung direkt neben dem Badezimmer und das Hochbett meiner Kinder steht sogar direkt an der Wand die an das Bad grenzt und von unten bis oben gefliest ist.
    Falls diese Fliesen strahlen, kann das dann auch durch die Wand dringen ?
    Danke und Lg und natürlich alles Gute Ihnen
    Elli

  302. #304 Tobias Cronert
    2. Mai 2020

    Hallo Elli,

    ja, es gibt bestimmte (wenige) Fliesen, die strahlen können. Das sind meistens diese super tollen italienischen Designerfliesen mit farblicher Glasur und ähnliche Dinge. Die normalen Fliesen aus dem Baumarkt sind idr. harmlos.

    JA, die Strahlung kann auch durch die Wand hindurch. Hier gilt auch wieder gesunder Menschenverstand, sprich, je dicker die Wand, desto weniger Strahlung. Je weiter das Bett von der Wand weg, desto weniger Strahlung etc. pp.

    Im Zweifelsfall kann man die Fliesen ja mal nachmessen, aber aus dem Handgelenk würde ich sagen, dass normale Fliesen (also 99% aller so verkauften Fleisen) harmlos sind. Wenn du ein Handyfoto schicken kannst, dann kann ich ja mal nen Blick drauf werfen.

  303. #305 rolak
    2. Mai 2020

    Handyfoto

    ^^welch schnöde Diskriminierung althergebrachter DickesTal-Knipsomaten :•P

    **daumendrückend für ‘Prädikat »harmlos«’**

  304. #306 Elli
    2. Mai 2020

    Das ist ja supernett 🙂 dankeschön..soll ich das Bild an deine E-Mail Adresse schicken ?

    LG

  305. #307 Elli
    2. Mai 2020

    ach und @Rolak: danke fürs Daumen drücken 🙂

  306. #308 Tobias Cronert
    2. Mai 2020

    Ja, per E-Mail hat ja gut funktioniert.

    Also wie in der E-Mail schon gesagt bin ich mir bei 2 Fliesen nahezu 100% sicher und bei den dritten auch annähernd. Die sind alle harmlos.

    Ich bin von der Ausbildung her zwar nur Festkörperphysiker mit ein wenig Kristallographie und kein Geologe, aber soweit man sich per Ferndiagnose sicher sein kann, so sicher bin ich mir, dass da keine nennenswerte Strahlung drin ist.

  307. #309 zimtspinne
    3. Mai 2020

    Wieso sind denn ausgerechnet italienische Designerfliesen radioaktiv!?
    Die wollte ich schon immer haben, wenn ich endlich zu Geld gekommen bin (was nicht mehr lang dauern kann!)

    Finde ich nicht nett, dass du mir die Fliesen nun madig gemacht hast und habe den Verdacht, du wolltest nur mal eine Runde auf überteuerte Designerfliesen drauf hauen.
    Man könnte ja auch welche ohne farbige Glasur nehmen, falls die das Problem ist.

  308. #310 Tobias Cronert
    4. Mai 2020

    Tja, dann investiere noch 60€ für nene Geigerzähler und nehm den mit ins Gesignerfliesenstudio um die Italiener zu messen, wenn sie all zu farbenfroh sind *g*

  309. #311 Elli
    8. Mai 2020

    Ja das frage ich mich auch gerade..werden denn heutzutage echt noch Farben mit Uran verwendet ? Ich dachte das war nur bis ca Mitte des 20. Jahrhunderts der Fall ?
    Wenn da also Uran enthalten ist, dann ist es doch natürliches Uran und ich meine, dass du mal irgendwo geschrieben hast, dass dieses nicht so gefährlich sei, da es ja nicht angereichert ist und eben auch überall vorkommt und der Körper damit klar kommt.
    ( Vielleicht habe ich es auch falsch in Erinnerung also falls ich mich hier geirrt habe bitte korrigieren 🙂 )

    Also was ich eigentlich gerne wissen würde, wäre es sehr schlimm solche Fliesen zu haben ?

    Und wie ist das mit Fliesen oder Küchenarbeitsplatten aus Granit und Schiefer ? Falls die Uran enthielten, ist das dann darin fest gebunden oder kann sich das auch durch abwischen oder drüber laufen oder so irgendwo anhaften ?
    und falls so eine Granitplatte Uran enthält, ist das dann eher eine Art partielle Verunreinigung ? Man hat doch dann nicht eine riesige Arbeitsplatte komplett aus Uran oder ?

    Und dann noch meine vorletze Frage:
    Eigentlich müsste doch dann in allen Materialien die aus dem Boden kommen Uran enthalten sein ? Also auch Erde und Sand und somit Beton aus dem unsere Hauwände gemacht werden.
    und irgendwo war die Frage zum Thema: wenn man radioaktive Partikel verschluckt und diese in den Körper gelangen.
    Und jetzt zu meiner (vorerst letzten *g) Frage, die sich daraus ableitet:
    Nehmen wir durch Gemüse das ja in der Erde angebaut wird oder durch das Wasser das ja tief aus der Erde kommt nicht ständig Uran-und andere radioaltiven Partikel auf ?
    Da es sich ja auch wieder um natürlich vorkommendes Uran handelt, dessen Kern ja nicht gespalten wird ist das wahrscheinlich auch nicht so das Problem dann oder ?

    LG Elli

  310. #312 Tobias Cronert
    8. Mai 2020

    Grundsätzlich hast du recht und in allem Gestein ist Uran und andere schwere Elemente drin, die machen dann im Gestein oft schöne Farben, in der Festkörperphysik nennt man das “dotierung”. Das ist die Natur der Erde und je älter das Gestein, desto mehr schwere Elemente. Dies sind in der Regel auch recht gleichmäßig verteilt … also immer jeweils in einer “Gesteinsschicht” (Achtung bin kein Geologe).
    Es kommt dann halt auf die Menge an. Also ich persönlich würde mir durchaus Fliesen mit Uran in die Küche bauen (OK ich würde sie natürlich vorher ausmessen, aber trotzdem). Wie ich bei meinem Artikel über die Uranmunition (https://scienceblogs.de/nucular/2017/09/12/wie-spuere-ich-uranmunition-mit-einem-geigerzaehler-von-amazon-auf-eine-anleitung/) geschrieben habe strahlt natürlich Uran ziemlich wenig … ins Schlafzimmer würde ich es mir zwar nicht stellen, aber sonst … da gibt es sehr viel schlimmere Dinge im radioaktiven Bereich. Uran hat nur einen schlechten Ruf, wegen den Atombomben und den explodierenden Reaktoren und so … als Mineral ist es erstmal ganz lieb 😉

    Wir haben schon immer in unserer Evolutionsgeschichte mit dem Essen Schwermetalle und radioaktive Substanzen (in sehr geringem Maße) aufgenommen. Darauf hat sich unser Körper eingestellt. Probleme entstehen erst dann, wenn Kontaminationen entstehen und das passiert dann meist mit künstlichen Elementen, wie Cäsium, Jod, etc.

  311. #313 Anonymous
    8. Mai 2020

    Also schließe ich jetzt daraus, dass es fest im Gestein drin ist und wenn man solche Natursteinfliesen im Haus hat, da keine Isotope rauskommen und sich überall verteilen können ?

    Also mir ist Uran trotzdem irgendwie unheimlich 😉

    Was sind denn sehr viel schlimmere Dinge im radioaktiven Bereich ? Plutonium ?

  312. #314 rolak
    8. Mai 2020

    fest im Gestein

    ‘Fest gemauert in der Erden’, altes Thema, <NichtName>, wie fest ist auch nicht die Frage: Uran ist 3.2ppm der Erdhülle; lokal mal mehr, mal weniger.
    Bei den Kacheln geht es allerdings um BearbeitungsVerfahren.

  313. #315 Elli
    9. Mai 2020

    “Uran ist 3.2ppm der Erdhülle”

    Ich weiß nicht was das bedeutet 😉

  314. #316 rolak
    9. Mai 2020

    bedeutet?

    War auch ziemlich kurz formuliert, Elli: wenn Du eine ganze Million Gramm (also eine metrische Tonne) von der Erdhülle ins Planschbecken schaufelst, sind da auch generell 3.2-parts-per-million oder konkret 3,2Gramm Uran mit drin.
    Zumindest im Mittel, also über sehr viele Planschbecken gemittelt.

    Die andersrum-Kurzformulierung wäre ‘3.2ppm der Erdhülle sind Uran’, die Wahl der Wortstellung dürfte Geschmackssache sein ;·)

  315. #317 Tobias Cronert
    9. Mai 2020

    Nein, das Uran ist ganz fest im Gestein drin eingebaut teilweise seit Millionen von Jahren.

    Die gefährlichen radioaktiven Substanzen sind die kurzlebigen, wie Caesium, Iod, Tritium, teilweise Kalium und ganz ganz böse ist Cobalt.
    … oder eben die flüchtigen, wie Radon.

    Denk immer dran je länger die Halbwertszeit, desto harmloser, je kürzer die Halbwertszeit, desto mehr strahlt es.

  316. #318 Florian
    Hamburg
    18. Mai 2020

    Guten Tag Tobias Cronert,

    vielen Dank für diese Seiten hier! Ich hätte nun tatsächlich auch eine Frage aus der profanen Welt, die wurde hier thematisch auch schon berührt. Über eine kurze Einschätzung (soweit möglich) wäre ich sehr dankbar!
    Ich habe einen Schmuckstein (ich weiß, Du bist kein Geologe) geschenkt bekommen und frage mich aufgrund seiner doch sehr kräftigen, dunklen Farbe, ob der Stein nicht vielleicht mit ionisierender Strahlung bzw. mit Neutronen behandelt/bestrahlt wurde. Da er aus dem Ausland (Russland) importiert wurde, ist die Einhaltung einer entsprechenden Abklingzeit nicht gewährleistet bzw. nicht nachvollziehbar. Die Frage dreht sich also um eine möglicherweise (noch) vorhandene Radioaktivität, wie sie bei Ihnen bereits in den FAQ vom BFS skizziert wird (siehe: https://www.bfs.de/SharedDocs/FAQs/BfS/DE/ion/ion/schmucksteine.html).

    Da ich mich da nun leider gar nicht auskenne, wüsste ich gerne, wäre ich über erste Anhaltspunkte schon sehr dankbar. Wäre in diesem Fall die Messung mit einem Geigerzähler tatsächlich eine Möglichkeit, um herauszufinden, ob der Stein in schädlicher Weise bzw. über die Grenzwerte hinaus radioaktiv ist? Sind solche Messgeräte auch ausleihbar? Und: solange Ungewissheit besteht, sollte der Stein am besten möglichst isoliert werden? Wenn ja, reicht es, ihn in einem recht dichten Behältnis wie einem Einmachglas zu lagern?

    Bei dem runden, geschliffenen Stein, in etwa der Größe des Gehäuses einer großen Weinbergschnecke, handelt es sich um einen sog. Schungit offenbar ein Gestein aus Algenkohle (s. https://de.wikipedia.org/wiki/Shungit).

    Viele Grüße!
    Flo

  317. #319 Tobias Cronert
    18. Mai 2020

    Hallo Flo,

    ich musste den Beitrag noch aus dem Spamfilter fischen, gut dass du mir noch eine Mail geschrieben hast. Das passiert eigentlich nicht so oft … komig … aber mal zur Antwort, die Frage ist schon sehr toll und spannend.

    Aaaalso die Farben kommen da her, dass in die ansonsten “perfekte” Kristallstruktur einzelne Fremdatome “implantiert” werden. Das nennt man in der Festkörperphysik dotieren und wenn man künstlich Kristalle herstellt, dann mischt man die Dotierung einfach beim Herstellungsprozess mit rein. Das ist am einfachsten. Wenn man jetzt natürliche Steine hat, dann kann man das theoretisch auch naträglich machen, in dem man sie mit ionisierender Strahlung beschießt und somit einzelne Atome aus dem ursprünglichen perfekten Kristallgitter in neue Elemente umwandelt, die dann den entsprechenden farblichen Dotierungseffekt haben.

    Das ist aufwendig und somit ziemlich teuer, weil bei Gammas oder Elektronen sehr hohe Energien benutzt werden müssen und auch nur eine sehr begrenzte “Palette” an Umwandlungen überhaupt möglich ist. Viel besser sind da dann eben Neutronen oder Protonen oder andere Teilchen, aber um die überhaupt zu bekommen braucht man einen Kernreaktor oder Teilchenbeschleuniger. Wie ich ja schon oft hier erzählt habe kostet ein Tag Neutronen ca. 15.000€ … ob sich das dann für die Schmuckherstelluing lohnt und man überhaupt die seltenen Zeit bekommt, sei mal dahingestellt.

    Wenn man das dann mit Neutronen macht, dann muss der ursprüngliche Stein möglichst klar und perfekt sein, denn je länger und intensiver die Bestrahlung, desto größer die Farbänderung.

    gleich gehts weiter es gibt Essen

  318. #320 Tobias Cronert
    19. Mai 2020

    Ob man das mit einem Geigerzähler messen kann kommt darauf über was für Elemente wir reden. Das kann man leider nicht pauschal sagen. Wenn die eine kurze Halbwertszeit haben, dann eben nicht, wenn die eine lange haben, dann schon. Ich geh mal davon aus, dass die meisten eine so lange Halbwetszeit haben, dass man das auch noch später messen könnte. Das kann man auch mit einem 60€ Messgerät von ebay machen. Ausleihen kann man einen Geigerzähler in der Regel nicht … zumindest kenne ich keine Stelle, die sowas verleihen würde.

    Bei einem dotierten Schmuckstein wäre Abschirmung bei der Lagherung wichtiger, als Luftdichtigkeit, weil die radioaktiven Elemente sowieso nicht aus der Kristallmatrix raus kommen. Also lieber eine Dose aus Metall und Abstand (also oben auf den Schrank packen).

    Bei dem Schungit ??? glaube ich nicht, dass der irgndwie behandelt worden ist. Wie gesagt braucht man dafür einen klaren durchsichtigen Stein, den mann dann leicht einfärben kann. Wenn der Stein schon von vorherein eine Farbe hat, dann funktioniert das mMn wahrscheinlich nicht… aber ich bin eben kein Geologe 😉

  319. #321 Flo
    Hamburg
    19. Mai 2020

    Lieber Tobias,

    vielen vielen Dank für deine Erklärung und Einschätzung. Das war sehr hilfreich und ich denke, dass mein Stein dann wohl eher nicht mit ionisierender Strahlung oder gar mit Neutronen oder Protonen beschossen worden ist ^^
    Wenn doch wäre das wohl vergebliche Müh gewesen und auch kein rentables Geschäft für die Verkäuferin des Steins, bei einem Durchmesser von ca. 3cm und zugleich einem Preis von offenbar unter 20 €. Und die Farbe ist auch nicht durchsichtig. Puh, Glück gehabt und wieder was dazu gelernt 😉

    Apropos. Ich bin über eine Stelle Deiner Antwort gestolpert und hätte da aus Interesse vielleicht einfach nochmal eine allgemeinere Verständnisfrage. Tatsächlich stelle ich mir die Frage auch schon etwas länger in abgewandelter Form unabhängig vom Stein. Tut mir leid, wenn ich hier nun laienhaft einiges durcheinander bringe. Ich hoffe, das hält sich in Grenzen. Wäre auf eine Antwort von Dir gespannt. Du gehst in Deiner Antwort oben auch auf die Art der Isolation des möglicherweise “verstrahlten” Steins ein (Abschirmung vs. Luftdichtigkeit). Heißt das (im Umkehrschluss), dass mit Neutronen beschossene Gegenstände ihre möglicherweise vorhandene Radioaktivität quasi nur “indirekt” über Partikel wie etwa Staub weitergeben können und dann Luftdichtigkeit wichtiger wäre? Also hängt die Frage nach einer möglichen Radioaktivität von Gegenständen und der richtigen Isolation (zum gesundheitlichen Schutz) auch immer von der Art der vorangegangen Bestrahlung ab? Mir ist die Frage auch in den Sinn gekommen, weil ich gesehen habe, dass Du woanders mal geschrieben hast, Du selbst hättest auch künstlich bestrahlte, immer eingeschweißte Proben in Form von Steinen, die Du auch nur bei Bedarf aus dem Bleischrank herausnehmen würdest. Dann ging es hier aber auch schon mal um ein Buch, dessen Seiten aus verstrahltem Holz hergestellt worden sind (#295). In diesem Fall sahst Du beim Berühren oder physischer Nähe keine konkrete Gefahr. Wo liegt da aber der Unterschied zu den Steinen im Bleischrank? Wie sollte denn die Handhabung (aus gesundheitlicher Sicht) sein, wenn z. B. mein Stein sich in einem Gebiet während einer Nuklearkatastrohphe befunden hätte und ich ihn trotzdem noch gerne behalten würde? Fand deinen Artikel “Ist Radioaktivität ansteckend?” da auch schon sehr erhellend! Aber komme auch damit gerade noch nicht weiter für meine hypothetische Frage.

    Nun muss ich erstmal was essen.
    Guten Abend noch und liebe Grüße!

  320. #322 Tobias Cronert
    20. Mai 2020

    Ja, bei 20€ wird da keiner was dran verdienen 😉

    Allein die Stromkosten für eine Bestrahlung sind ein paar hundert, wenn nicht 1000€ im Falle eines Teilchenbeschleunigers. *g*

    Was die Aufbewahrung angeht, so ist das eine sehr gute Frage mit erschreckend profaner Antwort. So ein Edelstein, wie ein Topas ist sehr hart (Moshärte 8/10 10 ist Diamant) Das heißt werdem gehen quasi keine Atome von der Oberfläche locker und fallen ab um man braucht sich keine Gedanken zu machen, dass die irgendwas anderes kontaminieren.
    Ein anderer Stein mag jetzt aber total bröckelig sein und dann muss man eben darauf aufpassen, dass kein radioaktiver Staub irgendwo abbröckelt.

    Wenn ich einen Stein bestrahle dann bestrahle ich generell den ganzen Stein und die Verteilung der enstehenden radioaktiven Isotope ist mehr oder weniger homogen, gleichverteilt über den Stein. Für niedrigenergie Röntgen oder Gamma stimmt das nicht mehr so ganz, aber auch die haben Eindringtiefen von mehreren Zentimetern.

    Radioaktivität kann theoretisch in sehr kleine Volumina gepackt werden. Das heißt, dass man theoretisch in ein Sandkorn so viel “gespeicherte Ionisierte Strahlung” reinpacken könnte um einen Menschen zu töten. Daher sollte man beim professionellen Umgang mit radioaktivem Material immer sehr vorsichtig sein und eben auch verhindern, dass Staub abbröckelt.

    Wenn man einen kontaminierten Stein hat, dann packt man ihn am besten in ein luftdichtes Plexiglas, so dass man es sich angucken kann und je nachdem, wie stark der Strahlt macht man kleinere Strahlenschutzanwendungen. Das heißt 1. Abstand:

    Lager den Stein in einem Arbeitszimmer hoch auf dem Schrank. Also da wo man sich eher wenig aufhlt mit so viel Abstand, wie Möglich.

    Wenn das nicht reicht (vorher ausmessen, dann kann man den noch in eine Metalldose packen und nur bei Bedarf herausholen, aber generell sind Abstand und wenig Einwirkzeit effektiver.

    liebe Grüße
    Tobi

  321. #323 rolak
    22. Mai 2020

    Nucular

    Keine Frage, ne GrinseHomerabilie.
    Keine Frage, sondern Erweiterung: Nukyular (nur wg Schreibweise)

  322. #324 Elli
    Bitz
    22. Mai 2020

    @Flo:
    Man kann sich den Gamma Scout ausleihen, der wird Dir dann per Post zugeschickt..
    Ich weiß nicht, ob ich die Seite hier posten darf aber findet man auch so 😉

    LG

  323. #325 Tobias Cronert
    25. Mai 2020

    Gerne posten im Zweifel muss ich sie nur aus de, Spamfilter fischen, wenn nur ein “kommerzieller” Link drin ist *g*

  324. #326 Elli
    Bitz
    26. Mai 2020

    https://geigerzaehler-mieten.de/

    Bitteschön..

    Ich hab das auch schon mal überlegt den zu leihen z.B. um verschiedene Sachen ( Fliesen, eine alte Vase usw. zu messen.

    Vor allem dann auch interessant wenn man ein Haus baut aber ich hab jetzt mehrfach gelesen, dass man Baumaterialien nicht so einfach messen kann.
    Oder doch ? oder ist die Strahlung so schwach, dass sie bei dem Gerät gar nicht ankommt ?
    So hatte ich das irgendwie im Kopf.

    um nochmal auf die italenischen Designerfliesen zurückzu kommen, diese würde der Gamma Scout schon anzeigen ?
    Dann wäre er wieder interessant

    Aber dennoch wäre ich doch auch sehr skeptisch so ein Ding auszuleihen..wer weiß wer das vor mir hatte und wo es so
    im Einsatz war 😀

  325. #327 Peter
    Moschheim
    27. Mai 2020

    Was heißt denn :

    Quellisotop ungleich CS 137

    Die Konvertierung von Impulsen pro Zeit in Dosisrate geht vom Cs 137 aus. Für den Fall, dass andere Isotope vorliegen, kann man die Impulse pro Zeit direkt anzeigen und nach eigenen Tabellen konvertieren.

    Das steht als Erklärung auf der Seite.

    Der Gammsscout enthält jetzt aber kein Cäsium oder ?

  326. #328 Tobias Cronert
    29. Mai 2020

    @Elii: Baumaterialien kann man mit dem Gamma-Scout eigentlich ganz gut ausmessen. Das “gute” an dieser Radioaktivität ist, dass sobald sie so hoch wird, dass sie gesundheitsgefärdend wird, dann kann man sie auch leicht messen. Der Umkehrschluss stimmt nicht immer, sprich wenn du nichts misst heißt das nicht zwangweise, dass es auch harmlos ist. Aber wenn gefährliche Mengen an Radioativität in Baumeterien vorhanden sind, dann kann man sie auch messen.

    @Peter: Ich bin mir jetzt auch unsicher, was die genau damit meinen. Also wie du schon gesagt hast geschieht die Umrechnung von Impulsen pro Zeit zu Dosis(leistung) durch Kalibrierungsproben. Das kann man mit CS 137 machen, ist aber eigentlich unpraktisch. Idealerweise macht man das auch mit mehreren Teststrahlern. Es könnter heißen, dass der Fehler für alle Isotope, die nidcht das Kalibrierungsisotop Cs sind soundso hoch ist … oder etwas ähnliches.

    Ich bin mir ziemlich sicher, dass der Zähler kein Cs 137 enthält. Es gibt zwar prof Geräte, die direkt einen Teststrahler enthalten, aber das ist idR. ein wesentlich langlebigerer und die beginnen auch erst in der sehr hohen Preiskathegorie.

  327. #329 Elli
    1. Juni 2020

    hmmm das gefällt mir so nicht *g
    Also mal angenommen das Gerät zeigt bei meinen Fliesen oder Steinböden oder was immer ich messe ( Baumaterialien halt) einen Wert von ca 0,1 bis ca. 0,25 µSv/h ( was glaube ich die normale Hintergrundstrahlung ist ) an kann ich dann sicher davon ausgehen, dass es harmlos ist.
    Wenn du sagst,: “der Umkehrschluss stimmt nicht immer, dass wenn man nichts misst es auch harmlos ist”
    was nützt mir dann meine Messung ? 🙁

    zu Peters Frage:

    Ist es nicht irgendwie so dass die Einheit cpm umgerechnet wird in µSv/h und für diese Umrechnung wird die Energiekurve von cs-137 verwendet.
    So ungefähr meine ich das mal gelesen zu haben.

    Es kann doch gar nicht sein, dass ein Strahlenmessgerät etwas radioaktives enthält, dann wären doch die ganzen Messergebnise falsch. Dann wären in dem Gerät die Werte ja schon höher als in der Umwelt oder verstehe ich da was nicht ?

    LG Elli

    rde das darin enthaltene Zählrohr ja höhere Werte zeigen als in der Umwelt üblich sind (0,1 bis ca. 0,25 µSv/h)

  328. #330 Elli
    1. Juni 2020

    Ja der letze Satz sollte eigentlich oben hin..genau, dass das Zählrohr eben dann höhere Werte anzeigen würde als in der Umwelt.

  329. #331 Tobias Cronert
    3. Juni 2020

    Also, wenn du Baumaterialien misst und der Zähler nichts anzeigt, dann kann du zu 95% oder noch mehr davon ausgehen, dass es sicher ist.

    Aber eine 100% Sicherheit gibt es halt niemals. Ein Restrisiko ist immer da. Warum die Sicherheit nur 95 und nicht 99,9% ist ist auch eher akademisch. Ich kann mir halt 1001 Szenarien vorstellen, in die so speziell sind, dass sie von ihrer Wahrscheinlichkeit her nie (im Lebenzeitalter des Universums) auftreten werden, aber dennoch theoretisch möglich sind. Was ist, wenn die ganze Aktivität in nur ein paar stark lokalisierter Atome konzentriert ist? Was ist, wenn die Strahlung gerichtet ist? Theoretisch ist viel möglich, was in der Praxis nie passiert.

    Die Strahlenmessgeräte mit den eingebauten Quellen sind die absoluten Profigeräte. Es gibt Varianten, wo man einen Eichstrahler intern vor den Detektor schieben kann zum eichen und andere Detektoren, die energiesensitiv sind und die konkreten Energielinien des Prüfstrahlers für eine allgemeine Messung herausfiltern. Aber mit solchen Geräten kommt der Normalbürger idR. nicht in Kontakt. Wie ich immer gerne sage, man kann Radioaktivität sehr sehr sehr präzise messen … mit den entsprechenden Detektoren, die dann auch gerne mal 30-100.000€ kosten können … oder noch mehr, wenn man in den hochwissenschaftlichen Bereich geht.

  330. #332 Elli
    9. Juni 2020

    ok, verstanden dankeschön 🙂

    Wenn ich jetzt etwas messen möchte z.B. eine Uranglasur oder wieder die Baumaterialien, kann ich mir das Ergebnis in µSv anzeigen lassen oder muß ich da CPS nehmen ?

    Der Hersteller vom GS sagt nämlich, dass man Alphastrahlen und Betastrahlen in CPS messen soll.

    Ich hab jetzt doch mal einen GS ausgeliehen und dieser zeigte bei allen Keramiken die ich gemessen habe ca 0,1 µSv an.
    Was wäre denn der Normalwert in CPS ?

  331. #333 Tobias Cronert
    10. Juni 2020

    Ja, am besten benutzt du µSv. Dem Wert kann bei einem Gammascout getrost vertrauen.

    Der Gammascout kann keine Alpha-Strahlung messen … egal, was der Hersteller sagt. Das stimmt einfach nicht und bei Bata Strahlung sollte man auch vorsichtig sein. … Aber das ist auch gar nicht nötig. Solange die Gamma Strahlung niedrig ist, ist alles bestens.

    Sv in CPM umrechnen ist immer für jedes Gerät individuell. Da kann ich spezifisch nichts zu sagen. Wenn ich jetzt z.B. die Detektorgröße verdoppeln würde, dann verdoppelt sich auch der CPS, weil doppelt so viel Ereignisse gemessen werden. Aber die Dosis und damit die Sv bleiben natürlich gleich.

  332. #334 Elli
    10. Juni 2020

    Also kann ich letztendlich keine Uranglasur messen ?

  333. #335 zimtspinne
    10. Juni 2020

    Hallo Tobi,

    hab heute auch mal eine Minifrage an dich, auch wenn die eigentlich (glaube ich) nicht so ganz in deinen Bereich fällt.
    Da du dich aber mit allem, was radioaktiv strahlt, auskennst, kannst du mir ja vielleicht trotzdem weiterhelfen.

    Und zwar gehts um Röntgengeräte.
    Ich will demnächst mal meinen dicken Kater zum Zahnarzt schicken, weil mir auffällt, dass er Rohfleisch nicht mehr so gerne frisst wie früher und dabei den Kopf extrem schief hält und nur auf einer Seite kaut.
    Äußerlich ist nichts zu sehen, Mundgeruch hat er auch nicht, ich will der Sache aber auf den Grund gehen und somit muss ein Röntgen her.

    Mein Lieblingstierarzt bietet leider nicht speziell Dentalröntgen an, er hat aber ein digitales Röntgengerät (super Sache, sollte es auch in der Humanmedizin öfter geben) und ich frage mich nun, ob das für die Zähne ausreicht….?
    Oder was kann speziell das Dentalröntgen besser?

    Bei den digitalen Geräten können die Aufnahmen wohl aufbereitet werden (Kontraste etc) – jetzt weiß ich aber nicht und habe dazu auch nichts auf die Schnelle gefunden, ob in der Zahnmedizin generell ganz andere Geräte zum Einsatz kommen, mit besserer Auflösung oder so, könnte ich mir zumindest vorstellen.

    Ich hatte selbst auch Aufnahmen gesehen, die mir sehr scharf aussahen, das waren allerdings Thoraxaufnahmen und eben keine Kiefer/Zahnwurzeln…. vielleicht kommt es ja dort noch viel mehr auf Feinheiten an, um auch jedes kleine Würzelchen auf dem Schirm zu haben.
    Kann man denn diese Bereiche nicht einfach entsprechend vergrößern?
    Ich werde auf jeden Fall auch noch dort nachfragen, wollte mich aber schon mal vorab bei dir informieren, damit ich dann dort keine völlig bekloppten Fragen stelle 😉

  334. #336 Tobias Cronert
    11. Juni 2020

    @Elli: Na klar kannst du Uranglasur messen. Einfach den Geigerzähler dranhalten und wenn der mehr als 1mSv / 365Tage/Jahr / 24h/Tag / 60 min/h / 60min/s = 0,3 µSv pro Sekunde anzeigt, dann kannst du dir Gedanken machen und wenn der weniger anzeigt, dann ist alles OK!

    Denn auch wenn der kein Alpha messen kann und Uran ein Alpha-Strahlen ist, kann der Gamma Scout trotzdem Uran messen (wie in vielen Youtube Videos gezeigt). Denn überall, wo natürliches Uran ist, sind auch die Zerfallsprodukte des Urans und diese Zerfallsprodukte emittieren Gamma Strahlung. Sprich überall, wo Uranglasur ist, ist auch Gamma Strahlung, die der GS messen kann. Also alles kein Problem, einfach dranhalten und messen, das reicht völlig aus.

    @Zimtspinne: Abuf … also obwohl ich mich mit Röntgengeräten auskenne ist das schon eine sehr spezielle Frage *g*
    Also aus dem Bauch raus, würde ich das digitale Gerät bevorzugen. Denn in der Tiermedizin braucht man sich keine Gedanken über die Dosis zu machen und da kann man die Leistung einfach super hoch aufdrehen und damit die Auflösung verbessern. Diese skaliert (annähernd) linear mit der Leistung mit dem BOnus, dass man bei einem digitalen Gerät direkt gucken kann, was man da geknipst hat, was bei “beweglichen Objekten” echt sinnvoll ist *g*.

    Spezialröntgen für Zähne reduziert wahrscheinlich die durchleutete Fläche zu Gunsten einer Dosisreduzierung und Auflösungserhöhung … ob das bei unseren felinen Freunden so sinnvoll ist wage ich mal zu bezweifeln. Macht natürlich auch noch mal weniger Streustrahlung und erhöht so die Qualität … na wie auch immer ohne die konkreten Werte der konkreten Geräte kann ich da kaum ne sinnvole pauschlale Aussage treffen.

  335. #337 zimtspinne
    11. Juni 2020

    @ Tobias

    “keine Gedanken über die Dosis zu machen” und “da kann man die Leistung einfach super hoch aufdrehen” – das geht aber doch nicht zu Lasten der Risiken der Tiere!?

    mein Kater ist ja erst knapp 10 und hat noch ein gutes Stück Leben vor sich, ich möchte ihn ja noch nicht abschießen.
    Vielleicht warte ich mit der Zahnbehandlung auch noch ein bisschen, momentan hat er ja noch keine Probleme.

  336. #338 Tobias Cronert
    11. Juni 2020

    Naja, ein Tier bekommt grundsätzlich die gleichen Problem, die es beim Menschen eben auch gibt, sprich erhöhtes Krebsrisito etc. pp.

    Aber wenn mein Tunfischschnitzel (neuer Name für unsere dicke faule Katze … lange Geschichte)) jetzt in 30 Jahren Krebs bekommt … naja das ist dann auch ziemlich egal. Da kommt es halt auf Lebenszeit an.

  337. #339 zimtspinne
    28. Juni 2020

    Ich bedanke mich erstmal für deine Antwort und Mühe und werde berichten, wie es war beim Katzenzahnarzt.
    Habe inzwischen auch Antwort erhalten – die werden wohl Unterkiefer und Oberkiefer insgesamt röntgen und nicht jeden Zahn einzeln (wie beim Dentalröntgen).

    apropos dicker Kater-
    meiner soll/muss abnehmen (wurde mir unabhängig von 2 verschiedenen TÄ beim Nierencheck erzählt und zum Schluss nochmal dran erinnert^^) und ich habe daher jetzt einen Großteil seiner Nahrung von Fertigfutter auf Rohfutter umgestellt. Damit frisst er tatsächlich sehr viel weniger – abgenommen hat er aber noch nicht wirklich dollle….. der Arme.
    Thunfisch mag er natürlich auch sehr gerne!

  338. #340 Anonymous
    29. Juni 2020

    Ich würde gerne längere Zeit nach Japan reisen und auch gerne einen Geigerzähler mitnehmen.
    In Japan ist glaube ich vor allem Plutonium freigesetzt worden, falls ich richtig informiert bin.
    Kann das ein Geigerzähler anzeigen ?

    LG
    Peter

  339. #341 Tobias Cronert
    29. Juni 2020

    Also die normalen Geräte von Amazon oder Conrad im Bereich von 100-300€ können dir anzeigen, falls irgendwas direkt gefährlich für Menschen ist. Was sie nicht können ist Radioaktivität in Nahrung messen, da musst du dich auf die Behörden und deren Profi-Messgeräte verlassen, wenn du z.B. Fisch oder Spinat aus der Sendai/Fukushima Provinz essen willst (was heutzutage aber eigentlich auch kein Problem ist).

    Wegen Plutonium braucht du dir keine Sorgen zu machen. Das gibt es in Japan nur in den Überresten der Brennstäbe in Fukushima und da auch nur in sehr geringem Maße. Da sind andere radioaktive Materialien viel viel gefährlicher.
    Die Atombombe von Nagasaki hatte auch Plutonium, aber das ist schon lange alles weg. Hiroshima war Uran und da gilt das gleiche.

    Die größte Gefahr in Japan geht aktuell von den Überbleibseln von Fukushima, also Cäsium, Tritium im Wasser, Kobald und den üblichen Verdächtigen aus … so wie bei Tschernobyl nur diesmal mit Meer und Fischen.

  340. #342 Captain E.
    30. Juni 2020

    Tja, man muss Kernkraftwerke nun nicht wirklich mögen, aber das Nichtwissen ist schon recht ausgeprägt. Klar enthält so eine Anlage Uran und Plutonium, und das auch in lebensgefährlichen Mengen.

    Nur zeigen alle bisherigen Ereignisse, dass bei steigender Distanz zum Unglücksort gerade einmal drei Isotope übrig bleiben, die mehr oder minder Ärger machen: Iod-131, Caesium-137 und Strontium-90. Ansonsten gilt: Was leicht flüchtig ist, kommt nicht weit über das Kraftwerk hin aus, alles andere bleibt dafür gleich in der Nähe. Das ist schon ein Problem, aber eben nur nahe am jeweiligen Schrottreaktor.

    Der Altmeister der deutschen Science Fiction, Wolfgang Jeschke, hat daher auch einen ganz dicken Schnitzer in seinem Buch “Das Cusanus-Spiel” eingebaut. Dort hat eine Anlage in Cattenom nach einem schweren Störfall Europa bis ins sachsen-anhaltinische und thüringesche hinein mit Pu-238 kontaminiert. Meine Güte! Pu-238? Das muss mit einem von mehreren recht aufwändigen Prozessen gezielt hergestellt werden, wenn man damit etwa Radionuklid-Batterien bestücken will. Das ist in einem normalen Reaktor kaum enthalten, und leicht flüchtig ist es als Schwermetall natürlich auch nicht.

    Und übrigens: Sollte es dieser Tage noch ein messbares Vorkommen von I-131 in Japan geben, dann stünde es um die havarierten Reaktoren wesentlich schlimmer als üblicherweise berichtet. Bei einer Halbwertszeit von 8 Tagen ist von damals nichts mehr vorhanden.

    Apropos: Tritium und Kobalt, gelangen sie durch das Kühlen in die Umwelt? Tritium dürfte leicht flüchtig sein. Wieso stellt das also auf größere Distanz kein Problem dar? Ist es vielleicht im Wasser gebunden?

  341. #343 Tobias Cronert
    1. Juli 2020

    Tritium ist halt gut wasserlöslich und dadurch entstehen da wohl (sowit ich weis) Probleme bei der Kühlung und in den Auffangbecken. Daher macht es dann halt auch Sinn das erst mal ein paar Jahre vor Ort zu lagern und dann in den Pazifik zu lassen … oder so.

    Cobalt entsteht halt da, wo Stahl zu viel aktiviert worden ist und hat halt eine sehr dumme Halbwertszeit. Aber soweit ich weis ist das eher ein sekundäres Problem dort vor Ort.

  342. #344 Elli
    1. Juli 2020

    “das Nichtwissen ist schon recht ausgeprägt”

    ja das ist es..und dafür haben wir ja euch 😀 🙂

    Also einfach ml ein großes Dankeschön und auch speziell an Dich Tobi für Deinen Fragenartikel und auch Deine Mail neulich !!

    Liebe Grüße
    Elli

  343. #345 Dalmatine
    1. Juli 2020

    Sie haben ja auch Bananen als Thema.
    Wieso haben diese Bananen denn überhaupt so viel Radioaktivität?
    Also die Frage klungt jetzt seltsam. Aber Bananen sin dja Kulturpflanzen. Und die Pflanzen stehen ja länger auf einem Feld. Müsste nicht die Pflanze irgendwann alle radioaktivität aus dem Boden aufgenommen haben, sodass dann irgendwann keine radioaktivität mehr in den Pflanzenfrüchten ist?

  344. #346 Dalmatine
    1. Juli 2020

    Sie haben ja auch Bananen als Thema.
    Wieso haben diese Bananen denn überhaupt so viel Radioaktivität?
    Also die Frage klingt jetzt seltsam. Aber Bananen sind ja Kulturpflanzen. Und die Pflanzen stehen ja länger auf einem Feld. Müsste nicht die Pflanze irgendwann alle Radioaktivität aus dem Boden aufgenommen haben, sodass dann irgendwann keine Radioaktivität mehr in den Pflanzenfrüchten gelangt?

  345. #347 Tobias Cronert
    2. Juli 2020

    @Elli: Sehr gerne. Es freut mich immer sehr, wenn ich etwas sinnvolles beitragen kann und es ist immer sehr schon zu hören, wenn man mal von Nutzen gewesen ist. Das motiviert sehr weiter zu machen.

    @ Dalmatine: Also bezgl. meiner geliebten Bananen. Die sind radioaktiv, weil sie von Natur aus relativ. viel gesundes Kalium enthalten. Also nicht, weil sie Radioaktivität aus dem Boden aufnehmen würden, sondern weil sie eben ein rel. hohes Level an dem einen Mineral haben. Sprich Bananen die auf Boden mit hoher Radioaktivität wachsen hätten (mehr oder weniger) genausoviel Radioaktivität, wie Bananen, die auf Boden mit niedriger Radioaktivität wachsen (sehr vereinfacht ausgedrückt).

    Alles natürliche Kalium überall auf der Welt besteht zu 0.01% aus dem radioaktiven Kalium 40. Das heißt alles natürliche Kalium überall ist immer ein bischen radioaktiv. Von Natur aus.

    Menschen haben auch Kalium im Körper und brauchen das Mineral auch ganz dringend um funktionieren zu können. Ich muss es z.B. aktuell sogar als Brausetablette zusätzlich nehmen. Weil ein Mensch viel größer ist, als eine Banane hat er auch mehr Kalium und ist daher (streng genommen) sogar noch radioaktiver, als eine Banane. Sprich während eine Banane “nur” mit 10 Becquerel in der Gegend herumstrahlt, strahlt ein gesunder, normaler Mensch mit 5000 Becquerel in der Gegend herum. Das ist aber OK so, denn alles strahlt immer irgendwie.

    Was für das Kalium gilt, gilt für sehr viele Mineralien. Die meisten netürlichen Materialien strahlen irgendwie ein bischen. Das ist normal. Die Dosis macht halt das Gift. Erst wenn die natürliche Radioaktivität durch den Menschen künstlich erhöht, sprich zusammengetragen wird, dann wird es gefährlich und je mehr davon auf einem Haufen gebracht wird, desto gefährlicher wird es.

    Also, meine geliebten Bananen sind erst mal unschuldig und fiese andere Dinge, wie Paranüsse sind viel böser 😉

  346. #348 Captain E.
    2. Juli 2020

    Anmerkung zum Thema Kalium: Die Bananen werden ja mit Sicherheit gedüngt. In diesem Dünger muss auch Kalium enthalten sein. Mit anderen Worten: Das Feld, auf dem die Bananenpflanzen stehen, wird von den Bauern in schöner Regelmäßigkeit frisch “kontaminiert”. Das Isotop K-40 ist mit einer Halbwertszeit von 1,248 Milliarden Jahren eben doch recht langlebig.

    Das Gute daran ist, dass langlebige Radionuklide dafür nicht allzu stark strahlen. Die Strahlung entsteht, wenn sich der Atomkern durch irgendwelche Kernprozesse verändert, sprich: zerfällt.